You are on page 1of 75

c

c
ccc
c c  cc
cc
 
c
 c c

Management control is a process of assuming that resources are obtained and used effectively and efficiently in the
accomplishment of the organization¶s objectives. It is a fundamental necessity for the success of a business and hence from
time to time the current performance of the various operations is compared to a predetermined standard or ideal performance
and in case of variance remedial measures are adopted to confirm operations to set plan or policy.
 cc c  cc !c"#!$#c%!c cc&'c

Gc !c  'c MANAGEMENT CONTROL SYSTEM is an overall process of the enterprise which aims to fit together the
separate plans for various segments as to assure that each harmonizes with the others and that the aggregate effect of
all of them on the whole enterprise is satisfactory.

Gc 
 c 
( (' MANAGEMENT CONTROL SYSTEM is built around a financial structure and all the resources and
outputs are expressed in terms of money. The results of each responsibility centre in respect to production and resources
are expressed in terms of a common denominator of money.

Gc  
 c
' It follows a definite pattern and time table. The whole operational activity is regular and rhythmic. It is
a continuous process even if the plans are changed in the light of experience or technology.

Gc " (
 (c ' It is a fully coordinated and integrated system.

Gc  ' Management control requires emphasis both on the search for planning as well as control. Both should go
hand in hand to achieve the best results.

Gc £
 
cc ) c
 ' Manager at every level as to focus towards future operational and accounting data, taking
into consideration past performance, present trends and anticipated economic and technological changes. The nature,
scope and level of control will be governed by the level of manager exercising it.

Gc  
 ccc
(c
'cMANAGEMENT CONTROL SYSTEM is not possible without predetermined goals and plans.
These two provide a link between such future anticipations and actual performance.

Gc £ & (c *
' MANAGEMENT CONTROL SYSTEM is on the basis of evaluation of past performance that the future
plans or guidelines can be laid down. Management Control involves managing the overall activity of the enterprise for the
future. It prevents deviations in operational goals.

Gc "

c  ' It is a continuous process over the human and material resources. It demands vigilance at every
step. Deciding, planning and regulating the activities of people associated in the common task of attaining the objectives
of the organization is a the primary aim of MANAGEMENT CONTROL SYSTEM.

Gc è  c 
 (' It is the managers, engineers and operators which implement the ideas and objectives of the
management. The coordination of the main division of an organization helps in smoother operations and less friction
which results in the achievement of the predetermined objectives.
c
 cc
 c
c
MANAGEMENT CONTROL SYSTEM is an important process in which accounting information is used to accomplish the
organizations objectives. Therefore the scope of control is very wide which covers a very wide range of management
activities.

Gc è  c
 ' Success if a business depends on formulation of sound policies and their proper implementation.

Gc "
 c) c 
+ 
' It involves designing and organizing the various departments for the smooth running of the
business. It attempts to remove the causes of such friction and rationalizes the organizational structure as and when the
need arises.

Gc "
 c) c 

' Anything that the business accomplishes is the result of the action of those people who work in
the organization. It is the people, and not the figures, that get things done.

Gc "
 c ) c ' The cost accountant is responsible to control cost sets, cost standards, labour material and over
heads. He makes comparisons of actual cost data with standard cost. Cost control is a delicate task and is supplemented
by budgetary control systems.

Gc "
 c) c 
, ' It involves the use of best methods and techniques so as to eliminate all wastages in time,
energy and material. The task is accomplished by periodic analysis and checking of activities of each department with a
view to avoid an eliminate all non-essential motions, functions and methods.

Gc "
 c ) c  c 
(  ' Capital budget is prepared for the whole concern. Every project is evaluated in
terms if the advantage it accrues to the firm. For this purpose capital budgeting, project analysis, study of cost of capital
etc are carried out.

Gc #) c
 ' A master plan is prepared for overall control and all the departments of the concern are involved in this
procedure.
-c .c c  c 
 c c  c c &c c  (c c  
+ 
c 
cc
cc   cc 
c

We define net cash flow as net income plus non cash adjustment which typically means net income plus depreciation though
that cash flows cannot be maintained over time unless depreciated fixed assets are replaced. So management is not
completely free to use its cash flows however it chooses. Therefore we define the term free cash flows.

c
£ cc& is the cash flow actually available for distribution to investor after the company has made all the investment
in fixed assets and working capital necessary to sustain ongoing operation. When we studied income statement in accounting
the emphasis was probably on the firm¶s net income, which is accounting profit. However the value of company¶s operation is
determined by the stream of cash flows that the operations will generate now and in the future. To be more specific, the
value of operation depends on all the future expected free cash flows, defined as after- tax operating profit minus the amount
of new investment in working capital and fixed assets necessary to sustain the business. Therefore the way for managers to
make their companies more valuable is to increase their free cash flow.
/ cc£"£'
 Pay interest to debt holders, keeping in mind that the net cost to the company is the after tax interest expense.
- Repay debt holders, that is, pay off some of debt.
0ccc Pay dividends to shareholders.
1ccc Repurchase stock from shareholders.
2 Buy marketable securities or other non operating assets.

In practice, most companies combine these five uses in such a way that the net total is equal to FCF. For example, a company
might pay interest and dividends, issue new debts, also sell some of its marketable securities. Some of these activities are
cash outflows (paying interest and dividends) and some are cash inflows (issuing debt and selling marketable securities), but
the net cash flow from these five activities is equal to free cash flows.
c
" 
cc cc&'c
'c
Suppose the company had a 2001 NOPAT of $170.3million and depreciation is only the non cash charge which is $100million
then its operating cash flow in 2001 would be NOPAT plus any non cash adjustment on the statement of cash flows.

Operating cash flow =NOPAT +depreciation (non cash adjustment)


= $17.03 + $100
= $270.3

Company has $1,455million operating assets, at the end of 2000, but $1,800 at the end of 2001.it made a net investment in
operating assets of

Net investment in operating assets = $18, 00 - $1,455 = $345million

If net fixed assets rose from $870million to $1000million however company reported $100million of depreciation. So its
gross investment in fixed assets would be

Gross investment = net investment + depreciation


= $130 + $100 = $230million

Company free cash flows in 2001 was

FCF = operating cash flow ± gross investment in operating assets


= $270.3 - $445
= - $174.7million

An algebraically equivalent equation is

FCF = NOPAT - Net investment in operating assets


= $170.3- $345
= - $174.7million

Even though company had a positive NOPAT, its very high investment in operating assets resulted in a negative free cash
flow. Because free cash flow is what is available for distribution to investor, not only was there nothing for investors, but
investor actually had to provide additional money to keep the business ongoing. A negative current FCF not necessarily bad
provided it is due to the high growth or to support the growth. There is nothing wrong with profitable growth; even it causes
negative free cash flow in the short term
c

0c .c c 3


 c   (c .c c  c   c c  
 
c 
(c (   c
cc

cc  
 
c

The 3
 (c   ( (BSC) is a performance management tool which began as a concept for measuring whether the
smaller-scale operational activities of a company are aligned with its larger-scale objectives in terms of vision and strategy.

By focusing not only on financial outcomes but also on the operational, marketing and developmental inputs to these, the
Balanced Scorecard helps provide a more comprehensive view of a business, which in turn helps organizations act in their
best long-term interests.
c c
The underlying rationale is that organizations cannot ˜  influence financial outcomes, as these are "lag" measures, and
that the use of financial measures alone to inform the strategic control of the firm is unwise. Organizations should instead
also measure those areas where direct management intervention is possible. In so doing, the early versions of the Balanced
Scorecard helped organizations achieve a degree of "balance" in selection of performance measures. In practice, early
Scorecards achieved this balance by encouraging managers to select measures from three additional categories or
perspectives: "Customer," "Internal Business Processes" and "Learning and Growth."

The balance scorecard suggests that we view the organization from


four perspectives, and to develop metrics, collect data and
analyze it relative to each of these perspectives:
c ! c 

c
(c &c   ) ' ³To achieve our
vision, how will we sustain our ability to change and
improve?´

c ! c 
c   c    ) c ' ³To satisfy our
shareholders and customers what business processes must
we excel at?´

c ! c  c    ) c ' ³To achieve our vision, how


should we appear to our customer?´

c ! c 

 c    ) c ' ³To succeed financially, how
should we appear to our shareholders?´
c
 

cc3
 (c  (cc

We can summarize the implantation of a balanced scorecard in four general steps;


1.c Define strategy.
2.c Define measure of strategy.
3.c Integrate measures into the management system.
4.c Review measures and result frequently.
c
Each of these steps is iterative, requiring the participation of senior executive and employees throughout the organization

§c  
c  c
The balance scorecard builds a link between strategy and operational action. As a result it is necessary to begin the process of
defining a balanced scorecard by defining the organization goals are explicit and what that targets have been developed.

§c  
c  cc  c
The next step is to develop measures in support of the articulate strategy. It is imperative that the organization focuses on a
few critical measures at this point; otherwise management will be overloaded with measures. Also, it is important that the
individual measures be linked with each other in a cause effect manner

§c 
   (c  c
c c
 
c c
The balanced scorecard must be integrated with the organization formal and informal structure, its culture, and its human
resources practice. While the balanced Scorecard gives some means for balancing measures, the measures can still become
unbalanced by others system in the organization such as compensation policies that compensate the manager strictly based on
financial performance.

§c $ ) &c  c


(c c£ ,
c
Once the balance scorecard is up and running it must be consistently reviewed by senior management. The organization should
be looking for the following

§c How do the outcome measures say the organization is doing?


§c How do the driver measures say the organization is doing?
§c How has the organization¶s strategy changed since the last review?
§c How has the scorecard measures changed?

The most important aspects of these reviews are as follows;

§c They tell management whether the strategy is being implemented correctly and how successfully the strategy is
working.
§c They show that management is serious about the importance of these measures.
§c They maintain alignment of measure to ever changing strategies.
c
Gc    c
c  

c3
 (c  (c

The following problems unless suitably dealt with, could limit the usefulness of the balanced scorecard approach:
c Poor correlation between nonfinancial measures and result.
c Fixation on financial result. No mechanism for improvement.
c No mechanism for improvement.
c Measures overload.

c c
Gc è c"  
c &
c


 c  c
(c c
Simply put there is no guarantee that future profitably will allow targets achievement in any nonfinancial area. This is probably
the biggest problem with the balanced scorecard because there is an inherent assumption that future profitability does follow
from achieving the scorecard measures, identifying the cause effect relationships among the different measures is easier said
than done.

This will be a problem with any system that is trying to develop proxy measures for future performance. While this does not
mean that the balanced Scorecard should be abandoned it is imp that comp adopting such a system understand that the links
between nonfinancial measures and financial performance are still poorly understood.
c
Gc £  
c


 c$ c
As previously discussed not only are most senior managers well trained and very adept with financial measures but they also
most keenly feel pressure regarding the financial performance of their comp. Shareholder are vocal and the board of directors
often applies pressure on the stakeholders behalf .this pressure often overwhelms the long term uncertain payback of the
nonfinancial measures.
c
Gc 
c 
c c ) 
c
One of the most overlooked pitfalls of the balanced scorecard is that a company cannot achieve Stretch goals if the Company
has no mechanism for improvement .Unfortunately achieving many of these goals require complete shifts in the way that
business is done yet the company often does not have mechanism to make those shifts . The mechanism available takes
additional resource and requires a changed in the company culture. These changes do not happen overnight nor do they
respond automatically to a new stretch targets. Inertia often works against the company employees are accustomed to a self
limited cycle of setting targets, missing those targets and readjusting the targets to reflect what was actually achieved.
Without a method for making improvement, improvements are unlikely to consistently happen no matter how good the stretch
goal sound.
c
Gc   
c) (c
How many critical measures can one manager track at one time without losing? Unfortunately there is no right answer to this
question except it is more than 1 and less than 50. It too few then the manager is ignoring measures that are critical to
creating success. If it too many then the manager may risk losing focus and trying to do too many things at once.

23"c(c4"5-667c c

è   c  )  
c8c4$c  )  
c%c4$c
ROI 28% 26%
Sales 100 Lacs 500 lacs
Investment 25 lacs 100 Lacs
EBIT 7 Lacs 26 lacs
c

+ c
(c
c
c  
 cc c c( )  
c
Solution:

 )  
c8c
ROI = (Profit / investment)* 100
Profit = (28/100)*25lacs
= 7lacs

Profit margin = (Profit/sales)*100


= (7/100)*100
= 7lacs

Turnover of investments = (Sales/investment)*100


= (100/25)*100
= 4 times

 )  
c%c
ROI = (Profit / investment)* 100
Profit = (26/100)*100lacs
= 26lacs

Profit margin = (Profit/sales)*100


= (26/500)*100
= 5.2lacs

Turnover of investments = (Sales/investment)*100


= (500/100)*100
= 5 times

Profit margin of X is better than profit margin of division Y. Turnover of investment of division Y is better than Division X.

Hence cost management of Division X is better than Division Y.

c c
 c-c

c "c (  
c 
c (  c &  c 
 c   c c ) c  c   c   
 c 
(c  c

) 
c  
 c c    c cè$!c  c c c c    c9c"#! ?

ANS. There should be different measures used for evaluating profit performance and capital investment performance as
needed.The goal of performance measurement systems is to implement strategy. In setting up such systems, senior
management selects measures that best represent the company's strategy. These measures can be seen as current and
future critical success factors; if they are improved, the company has implemented its strategy. The strategy's success
depends on its soundness. A performance measurement system is simply a mechanism that improves the likelihood the
organisation will implement its strategy successfully.

Gc  
cè    

There are two types of profitability measurements used in evaluating a profit center, just as there are in evaluating an
organization as a whole. First, there is a measure of management performance, which focuses on how well the manager is
doing. This measure is used for planning, coordinating, and controlling the profit center's day-to-day activities and as a device
for providing the proper motivation for its manager. Second, there is the measure of economic performance, which focuses on
how well the profit center is doing as an economic entity. The messages conveyed by these two measures may be quite
different from each other. For example, the management performance report for a branch store may show that the store's
manager is doing an excellent job under the circumstances, while the economic performance report may indicate that because
of economic and competitive conditions in its area the store is a losing proposition and should be closed.

The necessary information for both purposes usually cannot be obtained from a single set of data. Because the management
report is used frequently, while the economic report is prepared only on those occasions when economic decisions must be
made, considerations relating to management performance measurement have first priority in systems design-that is, the
system should be designed to measure management performance routinely, with economic information being derived from
these performance reports as well as from other sources.

Gc " c 
) 
c   
 : Most proposals require significant new capital. Techniques for analyzing
capital investment proposals attempt to find either (a) The net present value of the project, that is, the excess of the
present value of the estimated cash inflows over the amount of investment required, or

(b) The internal rate of return implicit in the relationship between inflows and outflows. An important point is that these
techniques are used in only about half the situations in which, conceptually, they are applicable.

! c cc c c 


c c
c
c 
c) c 
, c
c
+
cc cc

1)c The proposal may be so obviously attractive that a calculation of its net present value is unnecessary. A newly
developed machine that reduces costs so substantially that it will pay for itself in a year is an example.
2)c The estimates involved in the proposal are so uncertain that making present value calculations is believed to be not
worth the effort-one can't draw a reliable conclusion from unreliable data. This situation is common when the results
are heavily dependent on estimates of sales volume of new products for which no good market data exist. In these
situations, the "payback period" criterion is used frequently.
3)c The rationale for the proposal is something other than increased profitability. The present value approach assumes
that the "objective function" is to increase profits, but many proposed investments win approval on the grounds that
they improve employee morale, the company's image, or safety.

4)c There is no feasible alternative to adoption. Environmental laws may require investment in a new program,as an
example. The management control system should provide an orderly way of deciding on proposals that cannot be analyzed by
quantitative techniques. Systems that attempt to rank non-quantifiable projects in order of profitability won't work. Many
projects do not fit into a mechanical ranking scheme.
c c
c -c .c  c  c ( 
c  (c c   c   c c c   c 
 c
c  
+ 
c . c ( 
c

  c c cc  c c * cc


) cc
 

c
' When financial performance in a responsibility center is measured in terms of profit, which is the difference between
the revenues and expenses, the responsibility center is called a profit center.Profit as a measure of performance is especially
useful since it enables senior management to use one comprehensive measure instead of several measures that often point to
different directions.

There are two types of profitability measurements in a profit center, just as there are for the organization as a whole. There
is, first, a measure of 
 
c  
 :cin which the focus is on how well the manager is doing. This measure is
used for planning, coordinating and controlling the day-to-day activities of the profit center. Second, there is a measure of

 c   
 , in which the focus is on how well the profit center is doing as an economic entity. The message
given by these two measures may be quite different.

Gc ! ccè    c  'c

In order to evaluate the economic performance of a profit center, one must use net income after allocating all costs. However,
in evaluating the performance of manager, any of five different measures of profitability can be used.

1)c "
  
c  
'c The logic behind using contribution margin as a measure is that fixed expenses are not
controllable by the manager, and therefore he should focus on maximizing the spread between revenue and
expenses. But the problem with this is that some fixed costs are controllable and all fixed costs are partially
controllable. A focus on the contribution margin tends to direct attention away from this responsibility.
2)c  cè  ' This measure shows the amount that the profit center contributes to the general overhead and profit
of the corporation. It incorporates all expenses incurred in or directly traced to the profit center, regardless of
whether these items are entirely controllable by the profit center manager. A weakness of this measure is that it
does not recognize the motivational benefit of charging headquarters costs.
3)c "
   cè  ' Headquarters expenses are divided into two categories: controllable and non-controllable. The
controllable expenses are controlled by business unit manager. Consequently, if these costs are included in the
management system, the profit will be after the deduction of all expenses that are influenced by profit center
manager.
4)c 
 c  c! ' In this measure, all corporate overhead is allocated to profit centers. The basis of allocation
reflects the relative amount of expense that is incurred for each profit center. If corporate overheads are allocated to
profit centers, budgeted costs, not actual costs, should be allocated. Then the performance report will show an
identical amount in the ³budget´ and ³actual´ columns for such overheads.
5)c  c 
 ' Here, companies measure performance of domestic profit centers at the bottom line, the amount of
net income after income tax. There are two arguments 1) Income after tax is constant percentage of the pretax
income, so there is no advantage in incorporating income taxes 2) many decisions that have impact on income taxes
are made at headquarters, and it is believed that profit center manager should not be judged by the consequences of
these decisions.
0'c 
c   c    c c    
c  
+ 
c & c c 
 
c "
 c .c  c


  ) c
 c

  c"   cccè   


c# 
+ 
'c

c c

Acgoal of a manufacturing company is to earn a satisfactory profit specially a satisfaction profit, specially a satisfactory return
on assets its principle assets is the skill of its professional staff which doesn¶t appear on its balance sheet .return on assets
employed therefore is essential meaningless in such organization .their financial goal is to provide adequate compensation to
the professional.

c c
-cè   
c

Professional organization is labour intensive and the labour is of a special type. Research and development organization use in
setting selling price and for other management purposes .standard cost system ,separation of fixed and variable cost and
analyses of variance were built on the foundation are example of organization whose product are professional service.
Professional tends to give in adequate weight to the financial implication of their decision they want to do the best job they can
regardless of its cost.

Because profession are the organization most important resource some authors have advocated that the value of these
profession should be counted as assets the system that does this is called human resource accounting .in the 1970¶s many
books and articles were written on this subject but few comp actually such a system and we do not know of any that one
current .the problem of measuring the value of human assets is intractable

0c#c
(c
c  
c

The output of a profession organisation cannot be measured in physical terms, use in setting selling price and for other
management purposes .standard cost system, seperstion of fixed and variable cost and analyses of variance were built on the
foundation. We can measures the number of patient a physician treats n a day and even classify these visit by type of
complaint but this is by no means equivalent to measuring the amt or quality earned is one measures of output in some
professional organization but these monetary amts at most relate to the quantity of service rendered not to their quality.c

Some profession notably scientist engineer, and professional are reluctant to keep track of how they spend their time and this
complicate the track of measuring performance .this reluctant seems to have its root in tradition usually it can be overcome if
senior management is willing to put appropriate emphasis on the necessity for accurate time reporting .nevertheless difficult
problem arise in deciding how time should be charged to clients .if the normal work week is 40 hrs should a job be charged for
1/40th of a week compensation for each other spent on it? If so how should work done on evening and weekend be counted
how to account for time spent reading literature ,going to meeting ,and otherwise keeping up to date?

1cc + c

With a few exception such as some law firm and accounting firms ,professional organisations are relatively small and operate
at a single location .senior management in such organisations can personally observe what is going on and personally motivate
employee .thus there is less need for a sophisticated management control system ,with profit centres and formal performance
reports nevertheless even a small organisations need a budget a regular comparison of performance against budget ,and a
way relating compensation to performance.

2c * 
cc

In a manufacturing company there is a dividing line between marketing activities and production activities only senior
management is concerned with both .such a clean separation does not exist in most Professional organisation, however their
time and this complicate the track of measuring performance .this reluctant seems to have its root in tradition usually it can be
overcome if senior management is willing to put appropriate emphasis on the necessity for accurate time reporting.
Nevertheless difficult problem arise in deciding how time should be charged to clients .if the normal work. These marketing
activities are conducted by professional usually by professional, usually by professional who spend much of their time in
production work that is working for clients.

In such situation it is difficult to assign appropriate credit to the person responsible for selling a new customer; in a consulting
firm for example a new engagement may result from a conversation between a member of the firm or from the reputation of
one of the firm professional as an outgrowth of speeches or articles. Moreover the profession al who is responsible for
obtaining the engagement may not personally involved in carrying it out .until fairly recently these marketing contribution
were rewarded subjectively ±that is they were taken into account in promotion and compensation decisions .some
organisation now give explicit credit, perhaps as a percentage of the project revenue, if the person revenue, if the person who
hold sold the project can be identified.

c c
Gc .c c
  ) c"
 cc

Interactive control alerts management of strategic uncertainties either trouble or opportunities that become the basis for
manager to adapt to a rapidly changing environments by thinking about new strategies.

1.cA subset of the management control information that has a bearing on the strategic uncertainties facing the buss
becomes the focal point.
2.cSenior executive take such information seriously.
3.cManagers at all levels of the org focus attention on the information produced by the system.

2'c
( c(c4"5-667c 

Shandilya Ltd. has adopted Economic Value Added (EVA) technique for the appraisal of performance of its three divisions A,B
and C. Company charges 6% for current assets and 8 % for Fixed Assets, while computing EVA relevant data are given below
:-

è   c  )cc  )c3c  )c"c !c

3(  (c c 3(  (c c 3(  (c c 3(  (c c

Profit 360 320 220 240 200 200 780 760

Current Assets 400 360 800 760 1200 1400 2400 2520

Fixed Assets 1600 1600 1600 1800 2000 2200 5200 5600

Solution:

è   c  )cc  )c3c  )c"c !c

3(  (c c 3(  (c c 3(  (c c 3(  (c c

ROA 18% 16% 9% 9% 6% 6% 10% 9%

EVA 208 170.4 44 50.4 -32 -60 220 160.8

c"
c
c c (:c  (c
c c

There are three apparent benefits of an ROA measure. First, it is a comprehensive measure in that anything that effects the
financial statements is reflected in this ratio. Secondly, ROA is easy to calculate, easy to understand, and meaningful in
absolute sense. Finally, it is a common denominator that may be applied to any organizational units responsible for
profitability, no matter what its size or what business it practices. The performance of different units may be compared
directly to each other. Also, ROI data is available for competitors that can be used as a basis for comparison. Nevertheless,
the EVA approach has some inherent advantages over ROA.

There are three compelling reasons to use EVA over ROI. £ , with EVA all business units have the same profit objective for
comparable investments. The ROI approach, on the other hand, provides different incentives for investment across business
units. For example, a business unit that is currently achieving 30% ROA would be most reluctant to expand unless it is able to
earn a ROI of 30% or more on additional assets. Second, decision that increase a centre¶s ROI may decrease its overall
profits. Third advantage of EVA is that different interest rates may be used for different types of assets. For example, a
relatively low rate May be used for inventories while a higher rate may be used for different types of fixed assets.

c c
!c0c

c  c( 
 c
c ( 
c   ) cc

(c
(c(   
 c 
 c
 c
c

 c
 'cc
Expenses center are responsibility centers for which input or expenses are measured in monetary terms, but for
which outputs are not measured in monetary terms. There are two general types: engineered expense center and
discretionary expense center. They correspond to two types of costs.. Engineered costs are elements of cost for
which the right or proper amount of costs that should be incurred can be estimated with a reasonable degree of
reliability. Costs incurred in factory for direct labour direct material component supplies and utilities are examples.
c
-

(c 
 c
 'c
Engineered expense center have the following characteristics:c
1.c Their inputs can be measured in monetary terms.
2.c Their output can be measured in physical terms.
3.c The optimal dollar amount of input required to produce one unit of output can be established

Engineered expense center usually are found in manufacturing operations. Warehousing, distribution, trucking and
similar units in the marketing organization also may be engineered expense center and so many certain
responsibility center within administrative and support department. Examples are accounts receivable account
payable and payroll section in the controller department personnel record and cafeteria in the human resource
department shareholder record in the corporate secretary department and the company motor pool. Such units
perform repetitive task for which standard cost can be developed

In an engineered expense center the output multiplied by the standard cost or each unit produced represents what
the finished product should have cost. When this cost is compared to actual costs, the difference between the two
represents the efficiency of the organization unit being measured.

We emphasize that engineered expense centers have other important tasks not measured by cast alone. The
effectiveness of these aspects of performance should be controlled. For example expenses center supervisor are
responsible for the quality of good and for the volume of production in addition to their responsibility for cost
efficiency. Therefore the type and amount of production is prescribed and specific quality standards are set so that
manufacturing costs are not minimized at the expense of quality. Moreover manager of engineered expense center
may be responsible for activities such a training that are not related to current production judgment about their
performance should include an appraisal of how well they carry out these responsibilities.

There are few if any responsibility center in which all cost items are engineered. Even in highly automated
production department the amount of indirect labour and of various services used can vary with management
discretion.

Thus, the term engineered costs center refers to responsibility center in which engineered cost predominate but it
does not imply that valid engineering estimates can be made for each and every cost item.
c
0   
 c 
 c
 'c
The output of discretionary expenses center cannot be measured in monetary terms. They include administration
and support units research and development organization and most marketing activities.

The term discretionary does not mean that management judgments are capricious or haphazard. Management has
decided on certain policies that should govern the operation of the company. One company may have a small
headquarter staff another company of similar size and in the same industry may have a staff that is 10 time as
large the management of both companies may be concerned that they made the correct decision on staff size but
there is no objective way judging which decision was actually better manager are hired and paid to make such
decision after such a drastic change the level of discretionary expenses generally has a similar pattern from one
year to the next.

The difference between budgeted and actual expense is not a measure of efficiency in a discretionary expense
centre it is simply the difference between the budgeted input and the actual input. It in no way measures the
value of the output, if actual expense do not exceed the budget amount, the manager has µlived within the budget
µ however ,because by definition the budget does not purport to measure the optimum amount of spending we
cannot say that living within the budget is efficient performance .

c c
1 
 c
c ( 
c   ) cc

(c
(c(   
 c 
 c
 c
c
§c 3( c   
c
The decision that management make about a discretionary expense budget are different from the decisions that it
makes about the budget for an engineered expense center. For the latter, management decides whether the
proposed operating budget represent the cost of performing task efficiently for the coming period. Management is
not so much concerned with the magnitude of the task because this is largely determined by the actions of other
responsibility centers, such as the marketing departments¶ ability to generate sales. In formulating the budget for
a discretionary expense center, however management principal task is to decide on the magnitude of the job that
should be done.
c
§c 
 
c ( 
'c
Here the current level expenses in a discretionary expense center is taken as a starting points this amount is
adjusted for inflation for anticipated changes in the workload of continuing tasks for special tasks and if the data
are readily available for the cost of comparable work in similar units.
There are two drawbacks to incremental budgeting. First because managers of these centers typically want to
provide more service they tend to request additional resources in the budgeting process and if they make a
sufficiently strong case these request will be granted. This tendency is expressed in Parkinson¶s second law:
overhead costs tend to increase period. There is ample evidence that not all this upward creep in cost is
necessary.

This problem is especially compounded by the fact that the current level of expenditure in the discretionary
expenses center is taken for granted and is not re-examined during the budget preparation process. Second when
a company faces a crises or when a new management takes over overhead costs are sometimes drastically
reduced without any adverse consequences.

Despite this limitation most budgeting in discretionary expense centers is incremental. Time does not permit the
more thorough analysis described in the next section.
c â c  (c ) &'c
An alternative approach is to make a thorough analysis of each discretionary expense center on a schedule that
will cover all of them over a period of five year or so. That analysis provides a new base. There is a likelihood that
expenses will creep up gradually over the next five years and this is tolerated at the end of five years, another
new base is established. Such an analysis is often called a zero base review.

In contrast with incremental budgeting which takes the current level of spending as the starting point this more
intensive review attempts to build up de now the resources that actually are needed by the activity. Basic question
are raised;(1) should use customer?(2) what should the quality level be ?are we doing too much(3)should the
function be performed in this way (4) how much should it cost?
c
§c "c)   'cc
In discretionary expense center costs tend to vary with volume from one year to the next but they tend not to
vary with short run fluctuation in volume within a given year. By contrast costs in engineering expense center are
expected to vary with short run changes in volume. In part this reflect the fact that volume changes do have an
impact throughout the company even though their actual impact cannot be measures the ; in part this reflect the
fact that volume changes do have an impact throughout the company even though their actual impact cannot be
measured in part this result from a management personnel and personnel related costs are by far the largest
expense item in most discretionary expense center the annual budget for these center tend to be a constant
percentage of budgeted sales volume.

Gc -c
c c c& cc

 cc
 
c c
(c c   
cc cc

 
c c
c
 
c
 c
c
The management control function in an organization is influenced by the style of senior management. The style of
the chief executive officer affects the management control process in the entire organization. Similarly, the style
of the business unit manager affects the unit's management control process, and the style of functional
department managers affects the management control process in their functional areas.
c
§c  
 c
c
 
c 
Managers manage differently. Some rely heavily on reports and certain formal documents; others prefer
conversations and informal contacts. Some are analytical; others use trial and error. Some are risk takers; others
are risk averse. Some are process oriented; others are results oriented. Some are long-term oriented; others are
short-term oriented. Some emphasize monetary rewards; others emphasize a broader set of rewards.

c  c
Management style is influenced by the manager's background and personality. Background includes things like
age, formal education, and experience in a given function, such as manufacturing, technology, marketing, or
finance. Personality characteristics include such variables as the manager's willingness to take risks and his or her
tolerance for ambiguity.
c
§c   
c c
 
c"
 
The various dimensions of management style significantly influence the operation of the control systems. Even if
the same reports with the same set of data go with the same frequency to the CEO, two CEOs with different styles
would use these reports very differently to manage the business units.

Style affects the management control process ± how the CEO prefers to use the information, conducts performance
review meetings, and so on ± which in turn affects how the control system actually operates, even if the formal
structure does not change under a new CEO. In fact, when CEOs change, subordinates typically infer what the new
CEO really wants based on how he or she interacts during the management control process.
c
c è 
c) c 
c"
 c
Presence of personal versus impersonal controls in organizations is an aspect of managerial style. Managers differ
on how much importance they attach to formal budgets and reports as well as informal conversations and other
personal contacts. Some managers are "numbers oriented"; they want a large flow of quantitative information, and
they spend much time analyzing this information and deriving tentative conclusions from it. Other managers are
"people oriented"; they look at a few numbers, but they usually arrive at their conclusions by talking with people,
judging the relevance and importance of what they learn partly on their appraisal of the other person. They visit
various locations and spend time talking with both supervisors and staff to get a sense of how well things are
going.

Managers' attitudes toward formal reports affect the amount of detail they want, the frequency of these reports,
and even their preference for graphs rather than tables of numbers, and whether they want numerical reports
supplemented with written comments. Designers of management control systems need to identify these
preferences and accommodate them.
c
§c ! c) c c"
 
A manager's style affects the degreec of tight versus loose control in any situation. The manager of a routine
production responsibility center can be controlled relatively tightly or loosely, and the actual control reflects the
style of the manager's superior. Thus, the degree of tightness or looseness often is not revealed by the content of
the forms or aspects of the formal control documents, rules, or procedures. It is a factor of how these formal
devices are used. The degree of looseness tends to increase at successively higher levels in the organization
hierarchy: higher-level managers typically tend to pay less attention to details and more to overall results.

The style of the CEO has a profound impact on management control. If a new senior manager with a different style
takes over, the system tends to change correspondingly. It might happen that the manager's style is not a good fit
with the organization's management control requirements. If the manager recognizes this incongruity and adapts
his or her style accordingly, the problem disappears. If, however, the manager is unwilling or unable to change,
the organization will experience performance problems. The solution in this case might be to change the manager.
c
0c
c()
 c
(c( ()
 cc&c c 
 c 
c
(c  c
c (c

§c ! 
 c 
'cIf two or more profit center is jointly responsible for product development manufacturing
and marketing each should share in the revenue that is generated when the product is finally sold. The
transfer price is not primarily an accounting tool; rather, it is a behavioral tool that motivates manager to
make the right decisions. In particular the transfer price should be designed so that it accomplishes the
following  ;  ) ' It should provide each segment with the relevant information required to determine the
optimum tradeoff between company cost and revenues It should induce goal congruent decisions that is the
system should be so designed that decision improve business unit to earn more profit It should help
measure the economic performance of the individual profit centerc
c
§c !&c c 
' First, a charge is made for each unit sold that is equal to the standard variable
cost of production. Second a periodic charge is made for the buying unit. One or both of these
components should include a profit margin. The two step pricing method correct this problem by
transferring variable cost on a per unit basis, and transferring fixed cost and profit on a lump sum
basis under this method the transfer price for product A would be 5$ for each unit that unit Y
purchases plus $20000 per month for fixed cost. Plus $10000 per month for profit: if transfer of
product A in a certain month are at the expected amount 5000 units then under the two step
method unit y will pay the variable cost of $25000 plus $30000 for the fixed cost and profit a total
of $55000 .this is the same amount as the amount it would pay unit x if the transfer price is less
than 5000 units say 4000unoits.unit y would pay $50000 under the two step methods compared
with the $44000 it would pay if the transfer price were $11 per unit.

c c
§c The difference is their transfer prices were for not using a portion of unit X capacity that it has
reserved. Note that under two step method the company variable cost for product A is identifiable
to unit Y variable cost for the product, and unit Y will make the correct short term marketing
decisions. Unit Y also has information on upstream fixed costs and profit related to product A and
it can use these data for long term decision.The fixed cost calculation in the two step pricing
method is based on the capacity that is reserved for the production of product A that is sold to
unit Y the investment represented by this capacity is allocated to product A. The return on
investment that unit X earns on competitive product is calculated and multiplied by the investment
assigned to the product. In the example we calculated the profit allowance as a fixed monthly
amount. It would be appropriate under some circumstance to divide the investment into variable
and fixed component. Then, a profit allowance based on a return on investment on variable assets
would be added to the standard variable cost for each unit sold.

c è  c
'cIf the two step pricing system just described is not feasible, a profit sharing system might be
used to ensure congruence of business unit interest with company interest. This system operates somewhat as
follows.

1.c The product is transferred to the marketing unit at standard variable cost.
2.c After the product is sold, the business units share the contribution earned which is selling price minus the
variable manufacturing and marketing costs.

This method of pricing may be appropriate if the demand for the manufactured product is not steady enough to
warrant the permanent assignment of facilities as in the two step method. In general, this method accomplished
the purpose of making the marketing unit¶s interest congruent with the companies. There are several practical
problems in implementing such profit sharing system. First, there can be arguments over the way contribution is
divided between the two profit centers. Which is costly, time consuming and work against basic reason for
decentralization namely autonomy of the business units mangers. Second, arbitrarily divided up the profit between
units does not give valid information on the profitability of each segment of the organization.

Third since the contribution is not allocated until after the sale has been made the manufacturing units
contribution depends upon the marketing unit¶s ability to sell and on the actual selling price. Manufacturing units
may perceive this situation to be unfair
c
§c !&c c c   'c in this method, the manufacturing unit¶s revenue is credited at the outside sales price,
and the buying unit is charged the total standard costs. The difference is changed to a headquarter
account and eliminated when the business unit statement are consolidated, this transfer pricing method is
sometimes used when there are frequent conflict between the buying and selling units that cannot be
resolved by one of the other method both the buying and selling
c
§c ! c  c ) c ( ()
 c c  c  c c )
c &c  c c  
 
c   :c & ) c
the sum of the business unit profit is greater than overall company profits, senior management must be
aware of this situation in approved budget for the business units and in subsequent evaluation of
performance against these budget. Also, this system create an illusion feeling that business units are
making money while in fact the overall company might be losing after taking account of the debits to
headquarter. Further this system might motivate business unit to concentrate more on internal transfers
at the expense of outside sales
c
§c ! ccc c
 c &
c c 
c
c&(c c 
(c
( c c c(c
c ) & (c c c & *
 Sometime, it is better for the headquarter to be aware of the conflict arising
out of transfer prices because such conflict may signal problem in either the organizational structure or In
other management systems. Under the two sets of prices method these conflicts are smoothed over
thereby not alerting senior management to these problems.

1c  c   c 


 c  (c
c 
 
c $c <c c  )  c 
(c   c 
 
c

  ) c

§c ! c c 

 c 
 'c The financial control exercised in a discretionary expense center is quite
different from that in engineered center the latter attempts to minimize operating cost by setting a
standard and reporting actual costs against this standards. The main purpose of a discretionary expense
budget on the other hand is to allow the manager to control Cost for particular in the planning. Costs are
controlled primarily by deciding what task should be undertaken and what level of effort is appropriate for
each. Thus in a discretionary expense center financial control is primary exercised at the planning stage
before the amount are incurred.c
c

c c
§c   
c c   
 'c c The primary job of the manager of a discretionary expense center is to
accomplish the desired output spending an amount that is on budget is satisfactory. This is in contrast
with the report in an engineered expense center which helps higher management to evaluate the manger
efficiency. If these two types of responsibility center are carefully distinguished management may treat
the performance report for the discretionary expense center as if it were an indication of efficiency Control
over spending can be exercised by requiring that the manger approved be obtain before the budget is over
sometimes a certain percentage of overrun is permitted without additional approval if the budget really set
forth the best estimate of actual cost there is 50 percent probability that it will overrun and this is the
reason that some latitude is often permitted.c
c
§c "
 c    'c The control of R & D centers, which are also discretionary expense center is difficult
for the following at least a semi tangible output reasons.c
c
c
1.c Results are difficult to measure quantitatively. As contrasted with administrative activities, R&D usually has at
least a semi tangible output in patent, new products, or new processes. Nevertheless, the relationship of these
outputs to inputs is difficult to measure and appraise. A complete product of an R&D group may require
several year of effort; consequently input as stated in an annual budget may be unrelated to outputs. Even if
an output can be identified a reliable estimate of its value often cannot be made. Even if the value of the
output can be calculated, it is usually not possible for management to evaluate the efficiency of the R&D effort
because of its technical nature. A brilliant effort may come up against an insuperable obstacle, whereas a
mediocre effort may, by luck result in a bonanza.

2.c The goal congruence problem in R&D center is similar to that in administrative centers. The research managers
typically want to build the best research organization that money can buy, even though this is more expensive
than the company can afford. A further problem is that research people often may not have sufficient
knowledge of the business to determine the optimum direction of the research efforts.

3.c Research and development can seldom be controlled effectively on an annual basis. A research project may
take year s to reach fruition, and the organization must be built up slowly over a long time period. The
principal cost is for the work force obtaining highly skilled scientific talented is often difficult, and short term
fluctuation in the work force are in efficient. It is not reasonable, therefore to reduce R&D costs in years when
profits are low and increase them in year when profits are high. R&D should be looked at as a long term
investment not as an activity that varies with short run corporate profitability.
c
§c ! c $<c 

'Activities conducted by R&D organization lie along a continuum. At one extreme
is basic research; the other extreme is product testing. Basic research has two characteristics: first, it is
unplanned management at most can specify the general area that is to be explored second there is
often a very long time lag before basic research result in successful new product introductions.cFinancial
control system has little value in managing basic research activities. In some companies, basic research
in included as a lump sum in the research program and budget. In others, no specific allowance is made
for basic research as such; there is an understanding that scientists and engineers can devote part of
their time to explorations in whatever direction they find most interesting, subject only to informal
agreement with their supervisor.For product testing projects, on the other hand, the time and financial
requirement can be estimated, not as accurately as production activities.c

2c
c   c (c
c 
c   c )  c ) ( (cc 
c
c 
+ (ccè  c
"
 c
Services are intangible in nature. This characteristic of services makes it difficult for pricing. Charging business
units for services furnished by corporate staff units becomes challenging work due to intangibility of services.
While pricing corporate services, we exclude the cost of central service staff units over which business units have
no control (e.g., central accounting, public relations, and administration). If these costs are charged at all, they
are allocated, and the allocations do not include a profit component. The allocations are not transfer prices.
c
Gc . c
(cc
 ( c&c cc 
 'c
c
¢c For central services that the receiving unit must accept but can at least partially control the
amount used.
¢c For central services that the business unit can decide whether or not to use.

Business units may be required to use company staffs for services such as information technology and research
and development. In these situations, the business unit manager cannot control the efficiency with which these
activities are performed but can control the amount of the service received. There are three schools of thought
about such services.

c c
One school holds that a business unit should pay the standard variable cost of the discretionary services. If it pays
less than this, it will be motivated to use more of the service than is economically justified. On the other hand, if
business unit managers are required to pay more than the variable cost, they might not elect to use certain
services that senior management believes worthwhile from the company's viewpoint. This possibility is most likely
when senior management introduces a new service, such as a new project analysis program. The low price is
analogous to the introductory price that companies sometimes use for new products.

A second school of thought advocates a price equal to the standard variable cost plus a fair share of the standard
fixed costs-that is, the full cost. Proponents argue that if the business units do not believe the services are worth
at least this amount, something is wrong with either the quality or the efficiency of the service unit. A third school
advocates a price that is equivalent to the market price, or to standard full cost plus a profit margin. The market
price would be used if available (e.g., costs charged by a computer service bureau); if not, the price would be full
cost plus a return on investment. The rationale for this position is that the capital employed by service units
should earn a return just as the capital employed by manufacturing units does. Also, the business units would
incur the investment if they provided their own service.
c
Gc c# 
c/ cc )  cc
In some cases, management may decide that business units can choose whether to use central service units.
Business units may procure the service from outside, develop their own capability, or choose not to use the
service at all. This type of arrangement is most often found for such activities as information technology, internal
consulting groups, and maintenance work. These service centers are independent; they must stand on their own
feet. If the internal services are not competitive with outside providers, the scope of their activity will be
contracted or their services may be outsourced completely.

For example, Commodore Business Machines outsourced one of its central service activities-customer service-to
Federal Express. James Reeder, Commodore's vice president of customer satisfaction, said, "At that time we didn't
have the greatest reputation for customer service and satisfaction. But this was FedEx's specialty, handling more
than 300,000 calls for service each day. Commodore arranged for FedEx to handle the entire telephone customer
service operation from FedEx's hub in Memphis.

After losing $29 million online the previous year, Borders Group turned to rival Amazon.com to manage its online
sales. Borders get to maintain an Internet sales channel and gains the operational effectiveness provided by
Amazon.com while being able to focus on the growth of its bricks and mortar business.
In this situation, business unit managers control both the amount and the efficiency of the central services. Under
these conditions, these central groups are profit centers. Their transfer prices should be based on the same
considerations as those governing other transfer prices.
4 c"c9c-661c
c
Division B of Shayana company contracted to buy from Div. A, 20,000 units of a components which goes into the
final product made by Div. B. The transfer price for this internal transaction was set at Rs. 120 per unit by mutual
agreement. This comprises of (per unit) Direct and Variable labour cost of Rs. 20; Material Cost of Rs.60; Fixed
overheads of Rs.20 (lumpsum Rs.4 lacs) and Rs.20 lacs that Div. A would require for this additional activity.
During the year, actual off take of Div. B from Div. A was 19,600 units. Div. A was able to reduce material
consumption by 5% but its budgeted investment overshot by 10%.
a)c As Financial controller of Div. A, compare Actual Vs Budgetred Performance
b)c Its implications for Management Control?
c
 
'c
è   c 3(  (cc 3(  (c ccccccccccccccc ccccc
4$cè c 4!c
c$c 4$cè c/
c 4!c
c$c
/
c
£ 
 £   

 c
(c 20 4,00,000 20 3,92,000
=   c  c
"c
 c"c 60 12,00,000 57 11,17,200
£  (c#)  (c 20 4,00,000 4,00,000
!c"c 100 20,00,000 19,09,200
! 
 cè  c 120 24,00,000 119.86 23,49,200
è  c 20 4,00,000 4,40,000

) 
c 20 20,00,000 22,00,000
$#c>c 20% 20%
è  ?
) 
c

Despite of increase in investment by 10%, there is negligible difference in transfer price. Also the sales have
decreased by 400 units. Therefore we can say that additional investment has not achieved any positive results.

c c
c
!51c

c
c
c c
 cc$#c.c c c()
 

Return on investment (ROI) is the ratio of profit before tax to the gross investment.

ROI is calculated with the help of the following formula:

ROI = (Pre-Tax Profit/Sales) X (Sales/Net Assets) or (Pre-Tax Profits/Net Assets)

The numerator is profit before tax as reported in the P&L account. The profit should include only the profits arising out of the
normal activities of the division. Unusual items of receipts and expenses should be excluded from the profit figure. One should
also ignore windfalls and income from investments not related to the operations of the division. Tax is excluded from the
numerator because the marginal of the SBU is not responsible for or in control of the tax paid.

Capital employed can be ascertained from the balance sheet by including fixed and current assets. Assets not currently put to
divisional use should be excluded from the investment base. One also needs to exclude their relative earnings if any. The
company should also exclude intangible assets like goodwill, deferred revenue expenses, preliminary expenses, etc.

$#c
c c  ) (c 'c

Oc Increasing the profit margin on sales.

Oc Increasing the capital turnover

Oc Increasing both profit margin and capital turnover.

Oc Reducing cost as that adds to the total earnings of the firm.

Oc Increasing the profits by expanding present operations or developing new product line, increasing market share, etc.

Oc Diversifying, introducing productivity imporevement measures, expansion, replacement of old equipments

()
 cc$#c

Oc ROI relates return to the level of investment and not sales as the rate of return is more realistic.

Oc ROI can be decomposed into other variables as shown. These variables have tremendous analytical value.

Oc ROI is an effective tool for inter-firm comparison.

  
cc4 'c


c  c  (c=ccc
 c  cc$#c
(c c
c 
c :c=c( c
c(c;  cc c
) 
cc
) 
c
 c
c c


c& cc   
c

EVA does not solve all the problems of measuring profitability in an investment center. In particular, it does not solve the
problem of accounting for fixed assets discussed above unless annuity depreciation is also used, and this is rarely done in
practice. If gross book value is used, a business unit can increase its EVA by taking actions contrary to the interests of the
company, as shown in Exhibit 7-3. If net book value is used, EVA will increase simply due to the passage of time.
Furthermore, EVA will be temporarily depressed by new investments because of the high net book value in the early years.
EVA does solve the problem created by differing profit potentials. All business units, regardless of profitability, will be
motivated to increase investments if the rate of return from a potential investment exceeds the required rate prescribed by
the measurement system.

Moreover, some assets may be undervalued when they are capitalized, and others when they are expensed. Although the
purchase cost of fixed assets is ordinarily capitalized, a substantial amount of investment in start-up costs, new product
development, dealer organization, and so forth may be written off as expenses, and, therefore, not appear in the investment
base. This situation applies especially in marketing units. In these units the investment amount may be limited to inventories,
receivables, and office furniture and equipment. When a group of units with varying degrees of marketing responsibility are
ranked, the unit with the relatively larger marketing operations will tend to have the highest EVA.

c c
In view of all these problems, some companies have decided to exclude fixed assets from the investment base. These
companies make an interest charge for controllable assets only, and they control fixed assets by separate devices. Con-
trollable assets are, essentially, receivables and inventory.

Business unit management can make day-to-day decisions that affect the level of these assets. If these decisions are wrong,
serious consequences can occur-quickly. For example, if inventories are too high, unnecessary capital is tied up, and the risk
of obsolescence is increased; whereas, if inventories are too low, production interruptions or lost customer business can result
from the stockouts. To focus attention on these important controllable items, some companies, such as Quaker Oats, 17
include a capital charge for the items as an element of cost in the business unit income statement. This acts both to motivate
business unit management properly and also to measure the real cost of resources committed to these items.

Investments in fixed assets are controlled by the capital budgeting process before the fact and by post completion audits to
determine whether the anticipated cash flows, in fact, materialized. This is far from being completely satisfactory because
actual savings or revenues from a fixed asset acquisition may not be identifiable. For example, if a new machine produces a
variety of products, the cost accounting system usually will not identify the savings attributable to each product.

The argument for evaluating profits and capital investments separately is that this often is consistent with what senior
management wants the business unit manager to accomplish; namely, to obtain the maximum long-run cash flow from the
capital investments the business unit manager controls and to add capital investments only when they will provide a net
return in excess of the company's cost of funding that investment. Investment decisions, then, are controlled at the point
where these decisions are made. Consequently, the capital investment analysis procedure is of primary importance in
investment control. Once the investment has been made, it is largely a sunk cost and should not influence future decisions.
Nevertheless, management wants to know when capital investment decisions have been made incorrectly, not only because
some action may be appropriate with respect to the person responsible for the mistakes but also because safeguards to
prevent a recurrence may be appropriate.

-c.c c c( 
c (cc ) c c  
 cc
c
) 
c
 c c c c
(c
(  cc c. c (c&(cc 
(c

The following techniques are useful in evaluating the performance of an investment centre:

c$ 
c
c
) 
c4$#'c

The rate of return on investment is determined by dividing net profit or income by the capital employed or investment made
to achieve that profit.

ROI = Profit / Invested capital * 100

ROI consists of two components viz.

Profit margin

Investment turnover

ROI = Net profit / Investment

= (Net profit / Sales) * (Sales / Investment in assets)

It will be seen from the above formula that ROI can be improved by increasing one or both of its components viz. the profit
margin and the investment turnover in any of the following ways:

Increasing the profit margin

Increasing the investment turnover

Increasing both profit margin and investment turnover

Capital employed is taken to be the total of shareholders funds, loans etc

The profit figure used is in calculating ROI is usually taken from the profit and loss account, profit arising out of the normal
activities of the company should only be taken.

c c
Capital employed for the company as a whole can be arrived at as follows:

Share capital of the company xxx

Reserves and surplus xxx

Loans (secured/unsecured) xxx

------

xxx

Less: a. Investment outside the business xxx

b. Preliminary expenses xxx

c. Debit balance of P & L A/c xxx xxx

-------

xxxx

Merits:

Return on investment analysis provides a strong incentive for optimum utilization of the assets of the company. This
encourages managers to obtain assets that will provide a satisfactory return on investment and to dispose off assets that are
not providing an acceptable return. In selecting amongst alternative long-term investment proposals, ROI provides a suitable
measure for assessment of profitability of each proposal.

Demerits:

ROI analysis is not very suitable for short-term projects and performances. In the initial stages a new investment may yield a
small ROI which may mislead the management. Most likely the rate would improve in course of time when the initial
difficulties are overcome.

The book value of assets decline due to depreciation, the investment base will continuously decrease in value, causing the
rate of return to increase.

2. Residual income:

Residual income can be defined as the operating profit (or income) of the company less the imputed interest on the assets
used by the company. In other words, interest on the capital invested in the company is treated as a cost and any surplus is
the residual income. Residual income is profit minus notional interest charge on capital employed.

Residual income is affected by the size of the organization and therefore will not provide a basis for evaluation of
organizational performance. This is probably the main reason why the management continues to make use of ROI which is
relative measure.

Not all projects start off with positive or sufficiently large positive profits in the early years of a project to produce a positive
increment to residual income.

It has been argued that a more suitable measure of performance for investment centres, which could encourage managers to
be more willing to undertake marginally profitable projects, is residual income.

We recommend RI as a method of evaluating performance of an investment centre. Because when RI is adopted for
evaluation purposes, emphasis is placed on marginal profit amount above the cost of capital rather than on the rate itself.

c c
0c.c c c ;  ) cc! 
 cè 
cc

Transfer price if designed appropriately has the following objectives:

It should provide each segment with the relevant information required to determine the optimum trade-off between company
costs and revenues.It should induce goal congruent decisions-i.e. the system should be so designed that decisions that
improve business unit profits will also improve company profits. It should help measure the economic performance of the
individual profit centers. The system should be simple to understand and easy to administer.

Èc What is ideal transfer price in the situations of Limited Market Shortage of Capacity in the industry

The ideal transfer price in the situations of Limited Market

By limited market it means that the markets for buying and selling profit centers may be limited.

Even in case of limited market the transfer price that is ideal or satisfies the requirement of a profit center system is the
competitive price. In case if a company is not buying or selling its product in an outside market there are some ways to find
the competitive price. They are as follows:

If published market prices are available, they can be used to establish transfer prices. However, these should be prices
actually paid in the market-place and the conditions that exist in the outside market should be consistent with those existing
within the company.

For example, market prices that are applicable to relatively small purchases are not valid in this case.

Market prices are set by bids. This generally can be done only if the low bidder has a reasonable chance of obtaining the
business. One company accomplishes this by buying about one-half of a particular group of products outside the company
and one-half inside the company. The company then puts all of the products out to bid, but selects one-half to stay inside.
The company obtains valid bids, because low bidders can expect to get some of the business. By contrast, if a company
requests bids solely to obtain a competitive price and does not award the contracts to the low bidder, it will soon find that
either no one bids or that the bids are of questionable value.

If the production profit center sells similar products in outside markets, it is often possible to replicate a competitive price on
the basis of the outside price. If the buying profit center purchases similar products from the outside market, it may be
possible to replicate competitive prices for its proprietary products. This can be done by calculating the cost of the difference
in design and other conditions of sale between the competitive products and the proprietary products.

Shortage of Capacity in the industry

In this case, the output of the buying profit center is constrained and again company profits may not be optimum. Some
companies allow either buying profit center to appeal a sourcing decision to a central person or committee. In this scenario a
buying profit center could appeal a selling profit center¶s decision to sell outside.

The person/group would then make a sourcing decision on the basis of the company¶s best interests. In every case the
transfer price would be the competitive price. In other words, the profit center is appealing only the sourcing decision.

Even if there are constraints on sourcing, the market price is the best transfer price. If the market price can be approximated,
it is ideal transfer price.

Èc When do you use Cost Based Transfer Pricing?

We use cost-based transfer pricing if there is no way of approximating valid competitive price. Transfer prices may be set up
on the basis of cost plus a profit, even though such transfer prices may be complex to calculate and the results less
satisfactory than a market-based price.

Two aspects need to be considered for cost-based transfer pricing:

The cost basis: The usual basis is the standard cost. Actual costs should not be used because production inefficiencies will
then be passed on to the buying profit center. If the standard costs are used, there is a need to provide an incentive to set
tight standards and to improve standards.

The profit markup: In calculating the profit markup, there also are two decisions:

Èc What is the profit markup to be based?

c c
The simplest and most widely used base is percentage of costs. If this base is used, however, no account is taken of capital
required. A conceptually better base is a percentage of investment. But there may be a major practical problem in calculating
the investment applicable to a given product. If the historical cost of the fixed assets is used, new facilities designed to reduce
prices could actually increase costs because old assets are undervalued

Èc What is the level of profit allowed?

The second problem with the profit allowance is the amount of the profit. The conceptual solution is to base the profit
allowance on the investment required to meet the volume needed by the buying profit centers. The investment would be
calculated at a ³standard´ level, with fixed assets and inventories at current replacement costs. This solution is complicated
and, therefore, rarely used in practice.

1c4cc@! 
 cè 
c c
c
c

cAc
c& c
c   
c

If a group has subsidiaries that operate in different countries with different tax rates, manipulating the transfer prices
between the subsidiaries can scale down the overall tax bill of the group. For example the tax rate in Country A is 20% and is
50% in Country B. In the larger interest of the group, it would be advisable to show lower profits in Country B and higher
profits in Country A. For this, the group can adjust the transfer price in such a way that the profits in Country A increase and
that in Country B get reduced. For this the group should fix a very high transfer price if the Division in Country A provides
goods to the Division in Country B. This will maximize the profits in Country A and minimize the profits in Country B. The
reverse will be true if the Division in Country A acquires goods from the Division in Country B.

There is also a temptation to set up marketing subsidiaries in countries with low tax rates and transfer products to them at a
relatively low transfer price.

Transfer price is viewed as a major international tax issue. While companies indulge in all types of activities to lower their tax
liability, the tax authorities monitor transfer prices closely in an attempt to collect the full amount of tax due. For this they
enter into agreements whereby tax is paid on specific transactions in one country only. But if companies set unrealistic
transfer price to minimize their tax liabilities and the same is spotted by the tax authority, then the company is forced to pay
tax in both countries leading to double taxation.

There have been instances where companies have fixed unrealistic transfer prices. The first case relates to Hoffman La Roche
that imported two drugs Librium and Valium into UK at prices of 437 pounds and 979 pounds per kilo respectively. While the
tax authorities in UK accepted the price, the Monopolies Commission did not accept the company's argument, since the same
drugs were available from an Italian firm for 9 pounds and 28 pounds per kilo.

The company's lawyers argued the case before the Commission on two grounds viz.

1. The price was not set on cost but on what the market would bear and

2. The company had incurred an R&D cost that was included in the price.

These arguments did not go well with the Commission and the company was fined 1.85 million pounds for the manipulative
practices adopted while fixing the transfer price.

The second case is of Nissan. The company had falsely inflated freight charges by 40-60% to reduce the profits. The
manipulation helped the company to hide tax to the tune of 237 million dollars. The next year Nissan was made to pay 106
million dollars in unpaid tax in the USA because the authorities felt that part of their US marketing profits were being
transferred to Japan, as transfer prices on import of cars and trucks were too high. Interestingly the Japanese tax authorities
took a different view and returned the double tax.

With a view to avoid such cases from recurring, Organisation for Economic Cooperation and Development issued some
guidelines in 1995. These guidelines aim at encouraging world trade. They evolved what came to be known as the arm's
length price. The principle states that the transfer price would be arrived at on the basis as if the two . companies are
independent and unrelated. The price is determined through:

Comparable Price Method where the price is fixed on the basis of prices of similar products or an approximation to one.Gross
Margin Method where a gross margin is established and applied to the seller's manufacturing cost.

In spite of all these efforts, it has to be admitted that setting a fair transfer price is not easy. So the onus of proving the price
has been put on the taxpayer who is required to produce supporting documents. If the taxpayer fails to do this he is required
to pay heavy penalty. For example, in USA, failure to provide documentary evidence results in a 40% penalty on the arm's
length price. In UK the penalty is to the tune of 100% of any tax adjustment. Other countries are also in the process of
evolving tight norms for the same.Countries across the globe also allow the taxpayer to enter into an Advance Pricing
Agreement whereby dispute can be avoided and so also the costly penalty of double taxation and penalty.

c c
14c cc * cè  c c ( c 
 c  c )
c
c   (c * c"
c

By limited market it means that the markets for buying and selling profit centers may be limited.

Even in case of limited market the transfer price that is ideal or satisfies the requirement of a profit center system is the
competitive price. In case if a company is not buying or selling its product in an outside market there are some ways to find
the competitive price. They are as follows:

1. If published market prices are available, they can be used to establish transfer prices. However, these should be prices
actually paid in the market-place and the conditions that exist in the outside market should be consistent with those existing
within the company. For example, market prices that are applicable to relatively small purchases are not valid in this case.

2.Market prices are set by bids. This generally can be done only if the low bidder has a reasonable chance of obtaining the
business. One company accomplishes this by buying about one-half of a particular group of products outside the company
and one-half inside the company.

The company then puts all of the products out to bid, but selects one-half to stay inside. The company obtains valid bids,
because low bidders can expect to get some of the business. By contrast, if a company requests bids solely to obtain a
competitive price and does not award the contracts to the low bidder, it will soon find that either no one bids or that the bids
are of questionable value.

3.If the production profit center sells similar products in outside markets, it is often possible to replicate a competitive price
on the basis of the outside price.

4.If the buying profit center purchases similar products from the outside market, it may be possible to replicate competitive
prices for its proprietary products. This can be done by calculating the cost of the difference in design and other conditions of
sale between the competitive products and the proprietary products.So we see from the above arguments that market price is
ideal transfer price even in limited markets

c
c c
!c2c

c   c
(c   c 
 
 cc
c )  c
c
c
 
c
 c 


 Management control systems influence human behavior. ï ˜cmanagement control systems influence behavior in a goal
congruent manner; that is, they ensure that individual actions taken to achieve personal goals also help to achieve the
organization's goals. The concept of goal congruence, describing how it is affected both by informal actions and by formal
systems.
Senior management wants the organization to attain the organization's goals. But the individual members of the organization
have their own personal goals, and they are not necessarily consistent with those of the organization. The central purpose of
a management control system, then, is to ensure a high level of what is called "goal congruence." c a  c  c
  c
c c cc˜c cc c ˜c 
c
 c ˜c c cc c c
cc c c
c
   c
The significance of human behavior patterns in management control system can be explained with the help of 
 c

£ cc

 c c"
 
  In the informal forces both internal and external factors play a key role.


c£ c
c
External factors are norms of desirable behavior that exist in the society of which the organization is a part. These norms
include a set of attitudes, often collectively referred to as the  c 
 c which is manifested in employees' loyalty to the
organization, their diligence, their spirit, and their pride in doing a good job (rather than just putting in time). Some of these
attitudes are local that is, specific to the city or region in which the organization does its work. In encouraging companies to
locate in their city or state, chambers of commerce and other promotional organizations often claim that their locality has a
loyal, diligent workforce. Other attitudes and norms are industry-specific. Still others are national; some countries, such as
Japan and Singapore, have a reputation for excellent work ethics.



c£ c
Gc ð  
The most important internal factor is the organization's own culture-the common beliefs, shared values, norms of behavior
and assumptions that are implicitly and explicitly manifested throughout the organization. Cultural norms are extremely
important since they explain why two organizations with identical formal management control systems, may vary in terms of
actual control. A company's culture usually exists unchanged for many years. Certain practices become rituals, carried on
almost automatically because "this is the way things are done here." Others are taboo ("we just don't do that here"),
although no one may remember why. Organizational culture is also influenced strongly by the personality and policies of the
CEO, and by those of lower-level managers with respect to the areas they control. If the organization is unionized, the rules
and norms accepted by the union also have a major influence on the organization's culture. Attempts to change practices
almost always meet with resistance, and the larger and more mature the organization, the greater the resistance is.
Gc O  
The internal factor that probably has the strongest impact on management control is management style. Usually,
subordinates' attitudes reflect what they perceive their superiors' attitudes to be, and their superiors' attitudes ultimately
stem from the CEO.
Managers come in all shapes and sizes. Some are charismatic and outgoing; others are less ebullient. Some spend much time
looking and talking to people (management by walking around); others rely more heavily on written reports.   

  
The lines on an organization chart depict the formal relationships-that is, the official authority and responsibilities-of each
manager. The chart may show, for example, that the production manager of Division A reports to the general manager of
Division A. But in the course of fulfilling his or her responsibilities, the production manager of Division A actually
communicates with many other people in the organization, as well as with other managers, support units, the headquarters
staff, and people who are simply friends and acquaintances. In extreme situations, the production manager, with all these
other communication sources available, may not pay adequate attention to messages received from the general manager;

c c
this is especially likely to occur when the production manager is evaluated on production efficiency rather than on overall
performance. The realities of the management control process cannot be understood without recognizing the importance of
the relationships that constitute the informal organization.

Gc è   ð  


In working toward the goals of the organization, operating managers must know what these goals are and what actions they
are supposed to take in order to achieve them. They receive this information through various channels, both formal (e.g.,
budgets and other official documents) and informal (e.g., conversations). Despite this range of channels, it is not always clear
what senior management wants done. An organization is a complicated entity, and the actions that should be taken by
anyone part to further the common goals cannot be stated with absolute clarity even in the best of circumstances.
Moreover, the messages received from different sources may conflict with one another, or be subject to differing
interpretations. For example, the budget mechanism may convey the impression that managers are supposed to aim for the
highest profits possible in a given year, whereas senior management does not actually want them to skimp on maintenance or
employee training since such actions, although increasing current profits, might reduce future profitability. The informal
factors discussed above have a major influence on the effectiveness of an organization¶s management control. The other
major influence is the formal systems. These systems can be classified into two types: (1) the management control system
itself and (2) rules, which are described in this section.
! c£ c"
 c c
$ c
We use the word c as shorthand for all types of formal instructions and controls, including: standing instructions, job
descriptions, standard operating procedures, manuals, and ethical guidelines. Rules range from the most trivial (e.g., paper
clips will be issued only on the basis of a signed requisition) to the most important):e.g., capital expenditures of over $5
million must be approved by the board' of directors).
Some rules are guides; that is, organization members are permitted, and indeed expected, to depart from them, either under
specified circumstances or when their own best judgment indicates that a departure would be in the best interests of the
organization.
Some rules are positive requirements that certain actions be taken (e.g., fire drills at prescribed intervals). Others are
prohibitions against unethical, illegal, or other undesirable actions. Finally, there are rules that should never be broken under
any circumstances: a rule prohibiting the payment of bribes, for example, or a rule that airline pilots must never take off
without permission from the air traffic controller.
Some specific types of rules are listed below:
Gc è   ð 
Security guards, locked storerooms, vaults, computer passwords, television surveillance, and other physical controls may
be part of the control structure.
Gc O   
Much judgment is involved in deciding which rules should be written into a manual, which should be considered to be
guidelines rather than fiats, how much discretion should be allowed, and a host of other considerations. Manuals in
bureaucratic organizations are more detailed than are those in other organizations; large organizations have more
manuals and rules than small ones; centralized organizations have more than decentralized ones; and organizations with
geographically dispersed units performing similar functions (such as fast-food restaurant chains) have more than do
single-site organizations
Gc     
Various safeguards are built into the information processing system to ensure that the information flowing through the
system is accurate, and to prevent (or at least minimize) fraud of every sort. These include: cross-checking totals with
details, requiring signatures and other evidence that a transaction has been authorized, separating duties, counting cash
and other portable assets frequently, and a number of other procedures described in texts on auditing.

Gc  ð  
Task control is the process of assuring that specific tasks are carried out efficiently and effectively. Many of these tasks
are controlled by rules. If a task is automated, the automated system itself provides the control.
c

c c
-c .  c c
 c
c

c "
 cc  c
 c
c

5  c 
+ 
c

c 
 
c
 c
c c  c

c   


cc( 
  (c   c
c
 c


c "
 cc  c
 c
cè#
By law NPO are allowed to make profit but are restrained from distributing it to owners and management This way they are
non profit making organizations (from the owner's point of view). Such organizations include religious, charitable and
educational trusts. Prime goal of management control systems in such organization is enhancing the service spread first and if
possible then cost control rather and than operating efficiency. On the financial front, they enjoy many concessions from the
government such as taxes, subsidies, grants etc so also they attract special control from these assisting institutes.
Characteristics:

  
cc  c  
 c  c (cc   c
c 
c c  
cc c 
+ 
 If the
organization shows large net income it may be because that NPO may not be providing the services to the extent possible/
expected. If the organization shows net losses it may show the NPO facing risk of bankruptcy. Hence non availability of
clear-cut performance yardstick makes the problem of control worst.

- è#Bc) c
  (c cè
' NPOs do not have shareholder as its stakeholder. The capital contribution to the
business comes by way of contributions to assets such as building and equipments. Second kind of contribution could be in
the form of monetary assistance, which entitles the organization to reap the interest on it keeping the principal amount
intact.

0 # 
c c  
c c   c (c c 


c(cc(c )   And the contributed assets are
not allowed to mix up with the operating assets.
1 £
(c 

' NPO need to keep two types of financial statements one set for contributed capital and another for
operating capital. The nature of the contributed capital is beyond control of the management and therefore management
concentrates on controlling the operating assets/investments.

2c )

 ' Usually NP

of the product/services - what could be appropriate price? Usually it is set at total/full cost. The more stress expected on
allocation of scare resources. Though not stricter control, but a sense of control can be built among the managers by way of
using budgets for various O are managed by trusts, who exercise less control on operational matters. Hence performance
control is less demanding from owners' point of view and difficult from the point of view of management.

activities and expenses. Non profit basis makes performance evaluation quite impossible. But one can make the things easier
by concentrating on adherence to costs budgets, and enhancing the service base.
 
 
c
 c
c c  
Matrix organizational structure assigns multiple responsibilities to the functional heads. Evaluation of performance of such
organizational entities is very difficult. Though they offer economies of using scares functional staff, it poses problems of
casting the individual responsibility. This form of organization is very complex, from the point of view of management control
system.

At the end we must not forget that the management control system is for the organization and not the organization exists for
management control system. One has to mold and remold the management control system to suit the given organization
structure
A citation by Anthony is worth noting in this regard.

Usually in an advertisement agency, account supervisors are shifted from one


account to another on periodic basis, this practice allows the agency to look at the account from the perspectives of different
executives. However taking in to consideration the time lag of result realization in such services is quite large.
And this may pose problem of performance assessment of a particular executive. This does not mean a control system
designer should insist on abandoning the rotation system of the executives.

c c
c  
cc( 
  (c   c
c
 cc
Different corporate strategies imply the following differences in the context in which control systems need to be designed: As
firms become more diversified, corporate-level managers may not have significant knowledge of, or experience in, the
activities of the company's various business units. If so, corporate-level managers for highly diversified firms cannot expect to
control the different businesses on the basis of intimate knowledge of their activities, and performance evaluation tends to be
carried out at arm's length. Single-industry and related diversified firms possess corporatewide core competenciesc(on which
the strategies of most of the business units are based. Communication channels and transfer of competencies across business
units, therefore, are critical in such firms. In contrast, there are low levels of interdependence among the business units of
unrelated diversified firms. This implies that as firms become more diversified, it may be desirable to change the balance in
control systems from an emphasis on fostering cooperation to an emphasis on encouraging entrepreneurial spirit.

Gc    c 


' given the low level of interdependencies, conglomerates tend to use vertical strategic planning
systems-that is, business units prepare strategic plans & submit to senior management to review & approve. The
horizontal dimension might be incorporated into the strategic planning process in a number of different ways. First, a
group executive might be given the responsibility to develop a strategic plan for the group as a whole that explicitly
identifies synergies across individual business units within the group. Second, strategic plans of individual business units
could have an interdependence section, in which the general manager of the business unit identifies the focal linkages
with other business units and how those linkages will be exploited. Third, the corporate office could require joint strategic
plans for interdependent business units. Finally, strategic plansc of individual business units could be circulated to
managers of similar business units to critique and review. These methods are not mutually exclusive. In fact, several of
them could be pursued fruit. fully at the same time.
Gc 3( 
'cThe chief executives of single-industry firms may be able to control the operations of subordinates through
informal and personally oriented mechanisms, such as frequent personal interactions. This lessens the need to rely as
heavily on the budgeting system as the tool of control. On the otherchand, in a conglomerate it is nearly impossible for
the chief executive to rely on informal interpersonal interactions as a control tool; much of the communication and con-
trol has to be achieved through the formal budgeting stem. This implies the following budgeting system characteristics in
a conglomerate. Business unit managers have somewhat greater influence in developing their budgets since they, not the
corporate office, possess most of the information about their respective product/market environments. Greater emphasis
is often placed on meeting the budget since the chief executive has no other informal controls available.
Gc ! 
 c è 
'c Transfers of goods and services between business units are more frequent in single-industry and
related diversified firms than in conglomerates. The usual transfer pricing policy in a conglomerate is to give sourcing
flexibility to business units and use arm's-length market prices. However, in a single-industry or a related diversified firm,
synergies may be important, and business units may not be given the freedom to make sourcing decisions.
Gc 

 ) c"
 
'cThe incentive compensation policy tends to differ across corporate strategies in the following
ways-
Gc / c c  'c Conglomerates, in general, are more likely to use formulas to determine business unit managers'
bonuses; that is, they may base a larger portion of the bonus on quantitative, financial measures, such as X percent
bonus on actual economic value added (EVA) in excess of budgeted EVA. These formula-based bonus plans are employed
because senior management typically is not familiar with what goes on in a variety of disparate businesses. Senior
managers of single-industry and related diversified firms tend to base a larger fraction of the business unit managers¶
bonus on subjective factors. In many related diversified firms, greater degrees of interrelationships imply that one unit's
performance can be affected by the decisions and actions of other units. Therefore, for companies with highly
interdependent business units, formula-based plans that are tied strictly to financial performance criteria could be
counterproductive.

Gc è    c   'c In the case of unrelated diversified firms, the incentive bonus of the 'business unit managers
tend to be determined primarily by the profitabi1ity of that unit, rather than the profitability of the firm~ Its purpose is to
motivate managers to act as though the business unit were their own company.
c

c c
In contrast, single-industry and related diversified firms tend to base the incentive bonus of a business unit manager on
both the performance of that unit and the performance of a larger organizational unit (such as the product group to
which the business unit belongs or perhaps even .the overall corporation). When business units are interdependent, the
more the incentive bonus of general managers emphasizes the separate performance of each unit, the greater the
possibility of interunit conflict. On the other hand, basing the bonus of general managers more on the overall corporate
performance is likely to encourage greater interunit cooperation, thereby increasing managers' motivation to exploit
interdependencies rather than their individual results.
Gc 3
c/
c  'c Diversified corporations segment themselves into business units and typically assign different
strategies to the individual business units. Many chief executive officers of multi business organizations do not adopt a
standardized, uniform approach to controlling their business units; instead, they tailor the approach to each business
unit's strategy.The strategy of a business unit depends on two interrelated aspects: c(1) Its mission ("What are its overall
objectives?") and (2) its competitive advantage.c("How should the business unit compete in its industry to accomplish its
mission?"). Typicallyc business units choose from four missions: build, hold, harvest, and divest. The business unit has
two generic ways to compete and develop a sustainable competitive advantage: low cost and differentiation.c
Gc   
c The mission for existing business units could be either build, hold, or harvest. These missions constitute a
continuum, with "pure build" at one end and "pure harvest" at the other end. To implement the strategy effectively, there
should be congruence between the mission chosenc and the types of controls used. The mission of the business unit
influences the uncertainties that general managers face and the short-term versus long-term trade-offs they make.c
Management control systems can be systematically varied to help motivate the man ager to cope effectively with
uncertainty and make appropriate short-term versus long term trade-offs. Thus, different missions often require
systematically different management control systems.c
Gc   
c
(c/
 
c"Build" units tend to face greater environmental uncertainty than "harvest" units for severalc
reasons: Build strategies typically are undertaken in the growth stage of the product life cycle, whereas harvest
strategies typically are undertaken in the mature decline stage of the product life cycle. Such factors as manufacturing
process; product technology; market demand; relations with suppliers, buyers, and distribution channels; number of
competitors; and competitive structure change more rapidly and are more unpredictable in the growth stage than in the
mature/decline stage.cAn objective of a build business unit is to increase market share. Because the total market share of
all firms in an industry is 100 percent, the .battle for market share is a zero-sum game; thus, a build strategy puts a
business unit in greater conflict with its competitors than does a harvest strategy. Competitors' actions are likely to be
unpredictable, and this contributes to the uncertainty that build business units face.c On both the input side and the
output side, build managers tend to experience greater dependencies on external individuals and organizations than do
harvest managers. For instance, a build mission signifies additional capital investment (greater dependence on capital
markets), expansion of capacity (greater dependence on the technological environment), increase in market share
(greater dependence suppliers and labor markets), and so on. The greater the external dependencies a business unit
faces, the greater the uncertainty it confronts.Build business units are often in new and evolving industries; thus, build
managers are likely to have less experience in their industries. This also contributes to the greater uncertainty that
managers of build units face in dealing with external constituencies.c
Gc   
c
(c!  c
cThe choice of build versus harvest strategies has implications for short-term versus long-term
profit trade-offs. The share-building strategy includes Î
c price 'cutting, Î
c major R&D expenditures (to introduce new
products), and Î
cmajor market development expenditures. These actions are aimed at establishing market leadership,
but they depress short-term profits. Thus, many decisions that a build unit manager makes, today may not result in
profits until some future period. A harvest strategy, on the other hand, concentrates on maximizing short-term profits.c
Gc    c è


c When the environment is uncertain, the strategic planning process is especially important
management needs to think about how to cope with the uncertainties, and this usually requll1 longer-range view of
planning than is possible in the annual budget. If the environment is stable, there may be no strategic planning process
at all or only a broad-brush strategic plan. Thus, the strategic planning process is more critical and more important for
build, as compared with harvest, business units. c
Gc Nevertheless, some strategic planning of the harvest business units may be necessary because the company's overall
strategic plan must encompass all of its businesses to effectively balance cash flows.cc
c

c c
In screening capital investments and allocating resources, the system may be more quantitative and financial for harvest
units. A harvest business unit operates in a mature industry and does not offer tremendous new investment possibilities.
Hence, the required earnings rate for such a business unit may be relatively high to motivate the manager to search for
project with truly exceptional returns. Because harvest units tend to experience stable environments with predictable
products, technologies, competitors, and customers), discounted cash flow PCF) analysis often can be used more confidently.
The required information used to evaluate investments from harvest units c primarily financial.c A build unit, however, is
positioned on the growth stage of the product life cycle. Since the corporate office wants to take advantage of the
opportunities in a growing market, senior management may set a relatively low discount rate, thereby motivating build
managers to forward more investment ideas to corporate office. Given the product/market uncertainties, financial analysis of
some projects from build units may be unreliable. For such projects, nonfinancial data are more important.c
Gc 3( 
c The calculational aspects of variance analysis comparing actual results with the budget identify variances as
either favorable or unfavorable. However, a favorable variance does not necessarily imply favorable performance, nor
does an unfavorable variance imply unfavorable performance. The link between a favorable or unfavorable variance, on
the one hand, and favorable or unfavorable performance, on the other hand, depends on the strategic context of the
business unit under evaluation.c
Gc 

 ) c "
 
c  c In designing an incentive compensation package for business unit managers, the
followingcquestions need to be resolved:c
1.c 1. What should the size of incentive bonus payments be relative to the general manager's base salary? Should the
incentive bonus payments have upper limits?
2.c What measures of performance (e.g., profit, EVA, sales volume, market share, product
development) should be used when deciding the general manager's incentive bonus awards? If multiple performance
measures are employed, how should they be weighted?
3.c How much reliance should be placed on subjective judgments in deciding on the bonus amount?
4.c How frequently (semiannual, annual, biennial, etc.) should incentive awards be made?

With respect to the  c,  


, many firms use the principle that the riskier the strategy, the greater the proportion of
the general manager's compensation in bonus compared to salary (the "risk/return" principle). They maintain that because
managers in charge of more uncertain task situations should be willing.to take greater risks, they should have a higher per-
centage of their remuneration in the form of an incentive bonus. Thus, "build" managers are more likely than "harvest"
managers to rely on bonuses.
As  c the  
(c,  
, when rewards are tied to certain performance criteria, behaviour ls influenced cthe desire to
optimize performance with respect to those criteria. Some performance criteria (cost control, operating profits, and cash flow
from operations) focus more on short-term results, whereas other performance criteria (market share, new product develop-
ment, market development, and people development) focus on long-term profitability. Thus,
linking incentive bonus to short-term criteria tends to promote a short-term focus on the part of the general manager and,
similarly, linking incentive bonus to long-term criteria is likely to promote long-term focus. Considering the relative
differences in time horizons of build and harvest managers, it may not be appropriate to use a single, uniform financial
criterion, such as operating profits, to evaluate the performance of every business unit. A better idea would be louse multiple
performance criteria, with differential weights for each criterion depending on the business unit's mission.

The third question asks how much subjective judgment should affect bonus amounts. At one extreme, a manager's bonus
might be a strict formula-based plan, with the bonus tied to performance on quantifiable criteria (e.g., X percent bonus on
actual profits in excess of budgeted profits). At the other extreme, a manager's incentive bonus amounts might be based
solely on the superior's subjective judgment or discretion. Alternatively, incentive bonus amounts might also be based on a
combination of formula-based and subjective approaches. Performance on most long-term criteria (market development,
new-product development, and people development) is harder to measure objectively than is performance along most short-
run criteria (operating profits, cash flow from operations, and return on investment).As already noted, build managers- in
contrast withcharvest managers, should concentrate more on the long run, so they typically are evaluated more subjectively
than are harvest managers.

c c
As to the final question, the frequency of bonus awards does influence the time horizon of managers. More frequent bonus
awards encourage managers to concentrate on short-term performance since they have the effect of motivating managers to
focus on those facets of the business they can affect in the short run.
Gc "   ) c()
 cA business unit can choose to compete. Either as a differentiated player or as a low-cost
player, Choosing a differentiation 'approach, rather than a low-cost approach, increases uncertainty of a business unit's task
environment for three reasons.c

1.c Product innovation is more critical for differentiation business units than for low cost business units. This is partly
because a low-cost business unit, with primary emphasis on cost reduction, typically prefers to keep its product
offerings stable over time; a differentiation business unit, with its primary focus on uniqueness & exclusivity, is likely
to engage in greater product innovation.
2.c A low cost business unit typically tend to have narrow product lines to minimize the inventory carry costs as well as
to benefit from scale economies. Differentiation business units on the other hand tend to have a broader set of
products to create uniqueness.
3.c Low cost business units typically produce no-frill commodity products& these products succeed primarily because
they have lower prices than competing products. However product differentiation business units succeed if customers
perceive that the products have advantages over competing products. Since the customer perception is difficult to
learn about, & since customer loyalty is subject to change resulting from actions of competitors or other reasons, the
demand for differentiated products is typically more difficult to predict than the demand for commodities.

0c . c 
 
c 
 c    :c c & (:c
c   
 c   
c c
) 
c 
 c  c

 * cc
( cc
 
 :c
c  cc&
c c

(c 4 c( c c ccccccccccc4 c



  c c cccccccccccc4 c  (c

c 4 c"cc 4 c$  )  cc 4 c


)
 c


cIn some business units, the focus is on profit as measured by the difference between revenues and expenses. In other
business units, profit is compared with the assets employed in earning it. We refer to the latter group of responsibility centers
as investment centers.
 
c c (c
In deciding what investment base to use to evaluate investment center managers, headquarters asks two questions: First,
what practices will induce business unit managers to use their assets most efficiently and to acquire the proper amount and
kind of new assets? Presumably, when their profits are related to assets employed, business unit managers will try to improve
their performance as measured in this way. \Senior management wants the actions that they take toward this end to be in
the best interest of the whole corporation. Second, what practices best measure the performance of the unit as an economic
entity?

Gc "c

Most companies control cash centrally because central control permits use of a smaller cash balance than would be the case if
each business unit held the cash balances it needed to weather the unevenness of its cash inflows and outflows. Business unit
cash balances may well be only the "float" between daily receipts and daily disbursements. Consequently, the actual cash
balances at the business unit level tend to be much smaller than would be required if the business unit were an independent
company. Many companies therefore use a formula to calculate the cash to be included in the investment base. For example,
General Motors was reported to use 4.5 percent of annual sales; Du Pont was reported to use two months' costs of sales
minus depreciation.
One reason to include cash at a higher amount than the balance carried by a business unit is that the higher amount is
necessary to allow comparisons to outside companies. If only the actual cash were shown: by internal units would appear
abnormally high and might mislead senior management.

c c
Some companies omit cash from the investment base. These companies reason that the amount of cash approximates the
current liabilities; if this is so, the sum of accounts receivable and inventories will approximate the working capital.

Gc $  )  'c Business unit managers can influence the level of receivables, not only indirectly by their ability to
generate sales, and directly, by establishing credit terms by approving individual credit accounts and credit limits,
and by the collecting overdue amount. In the interest of simplicity, receivable included at the actual end-.of-period
balances, although the average of intraperiod balances is conceptually a better measure of the am should be related
to profits.c
Whether to include accounts receivable at selling prices or at cost of goods sold is debatable. One could argue that the
business unit's real investment in accounts receivable is only the cost of goods sold and that a satisfactory return on this
investment is probably enough. On the other hand, it is possible to argue that the business unit could reinvest the money
collected from accounts receivable, and, therefore, accounts receivable should be included at selling prices. The usual
practice is to take the simpler alternative-that is, receivables at the book amount, which is the selling price less an
allowance for bad debts.
If the business unit does not control credits and collections, receivables may be calculated on a formula basis. This formula
should be consistent with the normal payment period-for example, 30 days' sales where payment is made 30 days after
the shipment of goods.

Gc 
)
 'cInventories ordinarily are treated in a manner similar to receivables ±that is they are often recorded at end-
of-period amounts even though intraperiod averages would be preferable conceptually. If the company uses LIFO (last in
first out) for financial accounting purposes, a different valuation method usually is used for business unit profit reporting
because LIFO inventory balances tend to be unrealistically low in periods of inflation. In these circumstances, inventories
should be valued at standard or average costs, and these same costs should be used to measure cost of sales on the
business unit income statementc
If work-in-process inventory is financed by ˜ ccor by  ccfrom the customer, as is typically the
case with goods that require a long manufacturing period, these payments either are subtracted from the gross inventory
amounts or reported as liabilities.

For e.g. with manufacturing periods a year or greater, Boeing received progress payments for its airplanes and
recorded them as liabilities.

Some companies subtract  ccfrom inventory on the grounds that accounts payable represent financing of part
of the inventory by vendors, at zero cost to the business unit. The corporate capital required for inventories is only the
difference between the gross inventory amount and accounts payable. If the business unit can influence the payment period
allowed by vendors, then including accounts payable in the calculation encourages the manager to seek the most favorable
terms. In times of high interest rates or credit stringency, managers might be encouraged to consider forgoing the cash
discount to have, in effect, additional financing provided by vendors. On the other hand, delaying payments unduly to reduce
net current assets may not be in the company's best interest since this may hurt its credit rating.
Gc   (c c
Suppose the business unit whose financial statements are shown in Exhibit 1 (see page 21) sold its fixed assets for their book
value of $300,000, returned the proceeds of the sale to corporate headquarters, and then leased back the assets at a rental
rate of $60,000 per year. As Exhibit 2 (see page 21) shows, the business unit's income before taxes would decrease because
the new rental expense would be higher than the depreciation charge that was eliminated. Nevertheless, economic valued
added would increase because the higher cost would be more than offset by the decrease in the capital charge. Because of
this, business unit managers are induced to lease, rather than own, assets whenever the interest charge that is built into the
rental cost is less than the capital charge that is applied to the business unit's investment base. (Here, as elsewhere, this
generalization oversimplifies because, in the real world, the impact of income taxes must also be taken into account.)
Many leases are financing arrangements-that is, they provide an alternative way of getting to use assets that otherwise would
be acquired by funds obtained from debt and equity financing. Financial leases (i.e., long-term leases equivalent to the
present value of the stream of lease charges) are similar to debt and are so reported on the balance sheet. Financing
decisions usually are made by corporate headquarters. For these reasons, restrictions usually are placed on the business unit
manager's freedom to lease assets.

c c
Gc ( c c
If a business unit has idle asset that can be used by other units, the business unit may be permitted to exclude them from the
investment base if it classifies them as available. The purpose of this permission is to encourage business unit managers to
release underutilized assets to units that may have better use for them. However, if the fixed assets cannot be used by other
units, permitting the business unit manager to remove them from the investment base could result in dysfunctional actions
For example; it could encourage the business unit manager to idle partially utilized assets that are not earning a return equal
to the business unit's profit objective. If there is no alternative use for the equipment, ccontribution from this equipment
will improve company profits.
Gc 

  c c c
Some companies tend to be R&D intensive (e.g., pharmaceutical firms such as Novartis spend huge amounts on developing
new products); others tend to be marketing intensive (e.g., consumer products firms such as Unilever spend huge amounts
on advertising). There are advantages to capitalizing intangible assets such as R&D and marketing and then amortizing them
over a selected life. This method should change how the business unit manager views these expenditures. By accounting for
these assets as long-term investments, the business unit manager will gain less short-term benefit from reducing out lays on
such item, For instance, if R&D expenditures are expensed immediately, each dollar of R&D cut would be a dollar more in
pretax profits. On the other hand, if R&D costs are capitalized, each dollar cut will reduce the assets employed by a dollar; the
capital charge is thus reduced only by one dollar times the cost of capital, which has a much smaller positive impact on eco-
nomic valued added.

!Cc
c .c (c c 
( 
(c c c "
 
 c .c  c  c
 c  c c

 c c

 
 c

Ans: This term is used when the same goals are shared by top managers and their subordinates. This is one of the many
criteria used to judge the performance of an accounting system. The system can achieve its goal more effectively and perform
better when organizational goals can be well aligned with the personal and group goals of subordinates and superiors. The
goals of the company should be the same as the goals of the individual business segments. Corporate goals can be
communicated by budgets, organization charts, and job descriptions.
Gc c "
 
 5c  

c Individuals work in different hierarchies and handle different responsibilities & may
have different goals. But they must come together as far as Company¶s Goal is concerned (there action must speak Co¶s
language.)c
c"
 
 c
Example 1± The HR manager has devised a HR training program to enhance the skills of its sales personnel, with an objective
to enhance their productivity But if company is in strategic need of attaining a certain sales volume in a given quarter, it can
not do so on account of non availability of personnel.
Example 2± The marketing department has planned an impressive advertising campaign, which promises good returns, But
say due to cash crunch Company¶s current financial position may not let to lose the strings
Example 3 ± Production Manager may get a good applause for reducing cycle time; But at what cost? Building up the high
inventory i.e. higher investment in current assets. While doing so he just overlooked the financial interest of the company. ‡
After completing the given activity in more efficient manner the concerned manager scores the point/s on his score card. ‡
Whether his actions are leading to scoring of points on the organization¶s score card too? if it is so then only one can say the
organization is marching towards a common goal.
Every individual working in an organization has got his own motive to do the work. Individuals act in their own interest, based
on their own motivations. And it is always not necessarily consistent with the Co¶s goal. In a goal congruence process, the
actions the people are led to take in accordance with their perceived self interest are also in the best interest of the
organization i.e. Goal congruence ensures that the action of manager taken in their best interest is also in the best interest of
the organization.

Informal factors that influence goal congruence:


Informal Factors ±
External factors ± set of attitudes of the society, work ethics of the society

c c
Internal factors ± (Factors within the organization)
c Culture-Common beliefs, shared values, norms of behavior & assumptions
c Implicitly accepted and explicitly built into.
c Mgt. Style ± Informal/Formal
c The Communication Channels
c Perception and Communication ± e.g. Budget (meaning) strict profit.

Gc # 
+ 
c & c 3
c  )  
c 4è  c "
 c  c ) c   ) (c c  )  
c "
  c

 c ( ) ( (c c
c  
c c  c 
 
:c 
c c   c (
 
c D&c (c c c
  
c c ) (c 
c  cc
  c& c c c  
c

To the extent the decision are decentralized top management may lose some control. Relying on control reports is not as
effective as personal knowledge of an operation. With profit center, top management must change its approach to control.
Instead of personal direction senior management must rely to a considerable extent on management control reports.
Competent units that were once cooperating as functional units may now compete with one another dis advantageously. An
increase in one manager¶s profit may decrease those of another. This decrease in cooperation may manifest itself in a
manager unwillingness to refer sales lead to another business unit, even though that unit is better qualified to follow up on
the lead in production decision that have undesirable cost consequence on other units or in the hoarding of personnel or
equipment that from the overall company standpoint would be better off used in another units.
There may be too much emphasis on short run profitability at the expense of long run profitability. In the desire to report
high current profits, the profit center manager may skip on R&D, training, maintenance. This tendency is especially prevalent
when the turnover of profit center managers is relatively high. In these circumstances, manager may have good reason to
believe that their action may not affect profitability until after they have moved to other job.
There is no complete satisfactory system for ensuring that each profit center by optimizing its own profit , will optimize
company profits.
If headquarter management is more capable or has better information then the average profit center manager the quality of
some of the decision may be reduced.
Divisionalization may cause additional cost because it may require additional management staff personnel and recordkeeping
and may lead to redundant at each profit center.
3
c
cc  c
 'c
Business units are usually set up at profit centers. Business unit managers tend to control product development,
manufacturing, and marketing resources. They are in a position to influence revenue and cost and as such can be held
accountable for the bottom line. However as pointed out in the next section a business unit manager authority may be
constrained such constrained should be incorporated in designing and operating profit center.
Constraint on business unit authority
To realize fully the advantage of the profit center concept the business unit manger would have to be as autonomous as the
president of the independent company. As a practical matter however such autonomy is not feasible. If a company were
divided into completely independent units the organization would be giving up the advantage of size and synergism. Also
senior management authority that a board of director gives to the chief executive. Consequently business unit structure
represents trade off between business unit autonomy and corporate constraint. The effectiveness of a business units
organization is largely dependent on how well these trade off are made.
The performance of a profit center is appraised by comparing actual results for one or more orf these measures with
budgeting amounts. In addition, data on competitors and the industry provide a good cross check on the appropriate of the
budget. Data for individual companies are available from the securities and exchange commission for about key business
ratios; standard & poor computer services, Inc; Robert Morris associates annual statement studies; and annual survey
published in fortune, business week, and Forbes. Trade associations publish data for the companies in their industries.
Revenues: choosing the appropriate revenue recognition method is important. Should revenue be recognized at the time as
order is received, at the time an order is shipped, or at the time cash is received?

c  c
In addition to that decision, issues related to common revenues may need to be considered. There are some situations in
which two or more profit centers participate in the sales effort that results in a sale; ideally, each should be given appropriate
credit for its part in this transaction. Many companies have not given much attention to the solution of these common
revenue problems. They take the position that the identification of price responsibility for revenue generation is too
complicated to be practical and that sale personnel must recognize they are working not only for their own profit center but
also for the overall good of the company. They for example, may credit the business unit that takes an order for a product
handled by the another unit with the equivalent of a brokerage commission or a finder fee. In the case of a bank the branch
performing a service may be given explicit credit for that service even though the customer account is maintained in another
branch.
Role of controller
c It should publish procedure and forms for the preparation of the budget.
c It should provide assistance to budgetees in the preparation of their budget.
c It should administer the process of making budget revision during the year.
c It should coordinate the work of budget departments in lower echelons
c It should analyze reported performance against budget, interprets the result, and prepares summary report for senior
management.
Gc è ccc
 
c :c 
( c c"
4":c    (c c&
c   c
c
( c) c E2c
  c c 
(c
; (c 
c  
c  (5 c  * c  c  c c ( )  c 
 c c  :c  &c 
(c
  c "c  * c c  (c  c 
c &
(c c 
(c c &
c  

c # 
+ 
c
  c c 
 
c 
 c -66:c     :c  * c  c  c  
c è  c  c  c "
 c
 c 
(c c   c  c  * c  * c c ( c  c 
c 
*Fc c  
+ 
c  
c 
(c

 (
c"
  c"
 c cc 
c c  
cc"c  :c06Gcc (:c c
c 
c c c  ( 
c 
 (c 
c "c &
c c (c   c c ( c :c 

c
 * 
c
(c c
cc  
cc ( c
(c c
  (c
(c
c c( 
(cD&c(c
 c
c  
+ cc ) c c"
 
 c 
cè  
 c  

In a goal congruent process, the actions people are led to take in accordance with their perceived self-interest are also in the
best interest of the organization. A firm¶s strategy has a major influence on its structure. The type of structure in turn
influences the design of the organization¶s management control systems. Sundaram Shoe Company¶s (SSC) organization
structure is functional which involves the notion of a manager who brings specialized knowledge to bear on decisions related
to a specific function, vis-à-vis a general purpose manager who lacks the specialized knowledge. A skilled marketing and
production manager would be able to make better decisions in their respective fields. He would also be able to supervise
workers in the same function better than the generalist would. Thus an important advantage of the functional structure is
efficiency. A major disadvantage of this structure is that there is no unambiguous way of determining the effectiveness of the
separate functional managers because each function contributes jointly to the organization¶s final output. Therefore, there is
no way of determining how much of the profit was earned respectively by the several production departments.

Sundaram Shoe Company which was a market leader for a period of over 75 years has been losing market share, which has
impacted its profitability. Also it needs to be seen that the company outsources about 30% of its products. The company aims
to strengthen marketing, reduce costs and wants to be in a position to customize products as per the demands of the
customer. Thus, Sundaram needs to re-organize its organization structure which is functional to a Business Unit form of
organization. The benefits of the re-organization would be that the business unit or the division would be responsible for all
the functions involved in producing and marketing a specified product line. The business managers act almost as if their units
are separate companies. They are responsible for planning and co-coordinating the work of the separate functions. Their
performance is measured by the profitability of the business unit. This is a valid criterion because profit reflects the activities
of both marketing and production.

Though business unit managers exercise broad authority over their units, headquarters reserves certain key prerogatives.
Headquarters are responsible for obtaining funds for the company as a whole and allocating it to the business unit, as well as
approving budgets and judging the performance of business unit managers, setting their compensation.

c c
A major advantage of the Business unit structure of organization is that because it is close to the market for its products than
the headquarters, its manager may make sounder production and marketing decisions than headquarters might and the unit
as a whole reacts to new threats or opportunities quickly. This re-organization would help in achieving goal congruence in the
organization.

Performance Metrics are high-level measures 


 you are doing; that is, they assess your overall performance in the areas
you are measuring. They are external in nature and are most closely tied to outputs, customer requirements, and business
needs for the process.

The performance measurement system should cover the following areas at a minimum:

"/!#$cc

1.c Performance against customer requirements

2.c Customer Satisfaction

è$£#$"c#£c!$c.#$Hcè$#"cc

1.c Cycle times

2.c Product and service quality

3.c Cost performance (could be productivity measures, inventory, etc.)

/èè$cc

1.c Performance of suppliers against your requirements

£"cc

1.c Profitability (could be at the company, product line, or individual level)

2.c Market share growth and other standard financial measures

è#%cc

1.c Associate satisfaction

c c
!cEc
'cc4c  c c c& c c )  c 
+ 


'c£ c& c c )  c 
+ 
'c
Gc  
 cc
)
 c3 'cc
Goods can be held as inventory, which is a buffer that dampens the impact on production activity of fluctuations in sales
volume. Services cannot be stored. The airplane seat, hotel room, hospital operating room, or the hours of lawyers,
physicians, scientists, and other professionals that are not used today are gone forever. Thus, although a manufacturing
company can earn revenue in the future from products that are on hand today, a service company cannot do so. It must try
to minimize its unused capacity.

Moreover, the costs of many service organizations are essentially fixed in the short run. In the short run, a hotel cannot
reduce its costs substantially by closing off some of its rooms. Accounting firms, law firms, and other professional
organizations are reluctant to layoff professional personnel in times of low sales volume because of the effect on morale and
the costs of rehiring and training.
Gc   c
c"
 
c  'c
A manufacturing company can inspect its products before they are shipped to the consumer, and their quality can be
measured visually or with instruments (tolerances, purity, weight, color, and so on). A service company cannot judge product
quality until the moment the service is rendered, and then the judgments are often subjective. Restaurant management can
examine the food in the kitchen, but customer satisfaction depends to a considerable extent on the way it is served. The
quality of education is so difficult to measure that few educational organizations have a formal quality control system.
Gc   c

 ) 'c
Manufacturing companies add equipment and automate production lines, thereby replacing labor and reducing costs. Most
service companies are labor intensive and cannot do this. Hospitals do add expensive equipment, but mostly to provide better
treatment, and this increases costs. A law firm expands by adding partners and new support personnel.
Gc  5/
c# 
+ 
'c
Some service organizations operate many units in various locations; each unit relatively small. These organizations are fast-
food restaurant chains, auto rental companies, gasoline service stations, and many others. Some of the units are owned;
others operate under a franchise. The similarity of the separate units provides a common basis for analyzing budgets and
evaluating performance not available to the manufacturing company. The information for each unit can be compared with
system wide or regional averages, and high performers and low performers can be identified. However because units differ in
the mix of services they provide, in the resources that they use, and in other ways, care must be taken in making such
comparisons.c
c
'c43c
c  c   cc   
c 
+ 
c& c&(c) cc 
c
c c
 c
 c

'c  c"   ccè   
c# 
+ 
'c
Gc 'c
A dominant goal of a manufacturing company is to earn a satisfactory profit, specifically a satisfactory return on assets
employed. A professional organization has relatively few tangible assets; its principal asset is the skill of its professional staff,
which doesn't appear on its balance sheet. Return on assets employed, therefore, is essentially meaningless in such
organizations. Their financial goal is to provide adequate compensation to the professionals.c
In many organizations, a related goal is to increase their size. In part, this reflects the natural tendency to associate success
with large size. In part, it reflects economies of scale in using the efforts of a central personnel staff and units responsible for
keeping the organization up to- date. Large public accounting firms need to have enough local offices to enable them to audit
clients who have facilities located throughout the world.
Gc è   
'c
Professional organizations are labor intensive, and the labor is of a special type. Many professionals prefer to work
independently, rather than as part of a team. Professionals who are also managers tend to work only part time on
management activities; senior partners in an accounting firm participate actively in audit engagements; senior partners in law
firms have clients.

c c
Education for most professions does not include education in management, but quite naturally stresses the skills of the
profession, rather than management; for this and other reasons, professionals tend to look down on managers. Professionals
tend to give inadequate weight to the financial implications of their decisions; they want to do the best job they can, re- I
regardless of its cost. This attitude affects the attitude of support staffs and nonprofessionals in the organization; it leads to
inadequate cost control.
Gc #c
(c
c  
'c
The output of a professional organization cannot be measured in physical terms, such as units, tons, or gallons. We can
measure the number of hours a lawyer spends on a case, but this is a measure of input, not output. Output is the
effectiveness of the lawyer's work, and this is not measured by the number of pages in a brief or the number of hours in the
courtroom. We can measure the number of patients a physician treats in a day, and even classify these visits by type of
complaint; but this is by no means equivalent to measuring the amount or quality of service the physician has provided. At
most, what is measured is the physician's efficiency in treating patients, which is of some use in identifying slackers and hard
workers. Revenues earned is one measure of output in some professional organizations, but these monetary amounts, at
most, relate to the quantity of services rendered, not to their quality (although poor quality is reflected in reduced revenues in
the long run).
Furthermore, the work done by many professionals is non repetitive. No two consulting jobs or research and development
projects are quite the same. This makes it difficult to plan the time required for a task, to set reasonable standards for task
performance, and to judge how satisfactory the performance was. Some tasks are essentially repetitive: the drafting of simple
wills, deeds, sales contracts, and similar documents; the taking of a physical inventory by an auditor; and certain medical and
surgical procedures. The development of standards for such tasks may be worthwhile, although in using these standards,
unusual circumstances that affect a specific job must be taken into account.
Gc c + 'c
With a few exceptions, such as some law firms and accounting firms, professional organizations are relatively small and
operate at a single location. Senior management in such organizations can personally observe what is going on and personally
motivate employees. Thus, there is less need for a sophisticated management control system, with profit centers and formal
performance reports. Nevertheless, even a small organization needs a budget, a regular comparison of performance against
budget, and a way of relating compensation to performance.
Gc  * 
'c
In a manufacturing company there is a clear dividing line between marketing activities and production activities; only senior
management is concerned with both. Such a clean separation does not exist in most professional organizations. In some,
such as law, medicine, and accounting, the profession's ethical code limits the amount and character of overt marketing
efforts by professionals (although these restrictions have been relaxed in recent years). Marketing is an essential activity in
almost all organizations, however. If it can't be conducted openly, it takes the form of personal contacts, speeches, articles,
conversations on the golf course, and so on. These marketing activities are conducted by professionals, usually by
professionals who spend much of their time in production work-that is, working for clients.
In this situation, it is difficult to assign appropriate credit to the person responsible for "selling" a new customer. In a
consulting firm, for example, a new engagement may result from a conversation between a member of the firm and an
acquaintance in a company, or from the reputation of one of the firm's professionals as an outgrowth of speeches or articles.
Moreover, the professional who is responsible for obtaining the engagement may not be personally involved in carrying it out.
Until fairly recently, these marketing contributions were rewarded subjectively- that is, they were taken into account in
promotion and compensation decisions. Some organizations now give explicit credit, perhaps as a percentage of the project's
revenue, if the person who "sold" the project can be identified.

c c
'-c c ) c 3/c c c   c 
 c c ) c   c 
 c c
c c 3/c .c  c  c 
(  
c c (c c
 c c
c 
+ 
c
c c c
)  (c
cc   c
 c .c c c( 
c &cc  c c
  
 cc  c
 c c c  )
c c
(c(  c


'c "
(  
c  c 
c  
+ 
c c c 
)  (c
c c   c 
 'c Many management decisions involve
proposals to increase expenses with the expectation of an even greater increase in sales revenue. Such decisions are said to
involve expense/revenue trade-offs. Additional advertising expense is an example. Before it is safe to delegate such a trade-
off decision to a lower-level manager, two conditions should exist.c
c The manager should have access to the relevant information needed for making such a decision.
c There should be some way to measure the effectiveness of the trade-offs the manager has made.
A major step in creating profit centers is to determine the lowest point in an organization where these two conditions prevail.
All responsibility centers fit into a continuum ranging from those that clearly should be profit centers to those that clearly
should not. Management must decide whether the advantages of giving profit responsibility offset the disadvantages, which
are discussed below. As with all management control system design choices, there is no clear line of demarcation.
Gc .cc  cè  
 'c
There are two types of profitability measurements used in evaluating a profit center, just as there are in evaluating an
organization as a whole. First, there is the measure of management performance, which focuses on how well the manager is
doing. This measure is used for planning, coordinating, and controlling the profit center's day-to-day activities and as a device
for providing the proper motivation for its manager. Second, there is the measure of economic performance, which focuses on
how well the profit center is doing as an economic entity. The messages conveyed by these two measures may be quite
different from each other. For example, the management performance report for a branch store may show that the store's
manager is doing an excellent job under the circumstances, while the economic performance report may indicate that because
of economic and competitive conditions in its area the store is a losing proposition and should be closed. .
The necessary information for both purposes usually cannot be obtained from a single set of data. Because the management
report is used frequently, while the economic report is prepared only on those occasions when economic decisions must be
made, considerations relating to management performance measurement have first priority in systems design-that is, the
system should be designed to measure management performance routinely, with economic information being derived from
these performance reports as well as from other sources.
Gc ! ccè    c  c
A profit center's economic performance is always measured by net income (i.e., the income remaining after all costs,
including a fair share of the corporate overhead, have been allocated to the profit center). The performance of the profit
center manager, however, may be evaluated by five different measures of profitability: (1) contribution margin, (2) direct
profit, (3) controllable profit, (4) income before income taxes, or (5) net income
4c"
  
c 
'c
Contribution margin reflects the spread between revenue and variable expenses. The principal argument in favor of using it to
measure the performance of profit center managers is that since fixed expenses are beyond their control, managers should
focus their attention on maximizing contribution. The problem with this argument is that its premises are inaccurate; in fact,
almost all fixed expenses are at least partially controllable by the manager, and some are entirely controllable. Many expense
items are discretionary; that is, they can be changed at the discretion of the profit center manager. Presumably, senior
management wants the profit center to keep these discretionary expenses in line with amounts agreed on in the budget
formulation process. A focus on the contribution margin tends to direct attention away from this responsibility. Further, even
if an expense, such as administrative salaries, cannot be changed in the short run, the profit center manager is still
responsible for controlling employees' efficiency and productivity.c
4-c cè  'c
This measure reflects a profit center's contribution to the general overhead and profit of the corporation. It incorporates all
expenses either incurred by or directly traceable to the profit center, regardless of whether or not these items are within the
profit center manager's control. Expenses incurred at headquarters, however, are not included in this calculation. A weakness
of the direct profit measure is that it does not recognize the motivational benefit of charging headquarters costs.

c c
40c"
   cè  'c
Headquarters expenses can be divided into two categories: controllable and non controllable. The former category includes
expenses that are controllable, at least to a degree, by the business unit manager-information technology services, for
example. If these costs are included in the measurement system, profit will be what remains after the deduction of all
expenses that may be influenced by the profit center manager. A major disadvantage of this measure is that because it
excludes non controllable headquarters expenses it cannot be directly compared with either published data or trade
association data reporting the profits of other companies in the industry.
41c
 c  c! 'c
In this measure, all corporate overhead is allocated to profit centers based on the relative amount of expense each profit
center incurs. There are two arguments against such allocations. First, since the costs incurred by corporate staff departments
such as finance, accounting, and human resource management are not controllable by profit center managers, these
managers should not be held accountable for them. Second, it may be difficult to allocate corporate staff services in a manner
that would properly reflect the amount of costs incurred by each profit center.
There are, however, three arguments in favor of incorporating a portion of corporate overhead into the profit centers'
performance reports. First, corporate service units have a tendency to increase their power base and to enhance their own
excellence without regard to their effect on the company as a whole. Allocating corporate overhead costs to profit centers
increases the likelihood that profit center manager§ will question these costs, thus serving to keep head office spending in
check. (Some companies have actually been known to sell their corporate jets because of complaints from profit center
managers about the cost of these expensive items.) Second, the performance of each profit center will become more realistic
and more readily comparable to the performance of competitors who pay for similar services. Finally, when managers know
that their respective centers will not show a profit unless all-costs, including the allocated share of corporate overhead, are
recovered, they are motivated to make optimum long-term marketing decisions as to pricing, product mix, and so forth, that
will ultimately benefit (and even ensure the viability of) the company as a whole.
If profit centers are to be charged for a portion of corporate overhead, this item should be calculated on the basis of
budgeted, rather than actual, costs, in which case the "budget" and "actual" columns in the profit center's performance report
will show identical amounts for this particular item. This ensures that profit center managers will not complain about either
the arbitrariness of the allocation or their lack of control over these costs, since their performance reports will show no
variance in the overhead allocation. Instead, such variances would appear in the reports of the responsibility center that
actually incurred these costs. .
42c c
 'c
Here, companies measure the performance of domestic profit centers according to the bottom line, the amount of net income
after income tax. There are two principal arguments against using this measure: (1) after tax income is often a constant
percentage of the pretax income, in which case there would be no advantage in incorporating income taxes, and (2) since
many of the decisions that affect income taxes are made at headquarters, it is not appropriate to judge profit center
managers on the consequences of these decisions. There are situations, however, in which the effective income tax rate does
vary among profit centers. For example, foreign subsidiaries or business units with foreign operations may have different
effective income tax rates. In other cases, profit centers may influence income taxes through their installment credit policies,
their decisions on acquiring or disposing of equipment, and their use of other generally accepted accounting procedures to
distinguish gross income from taxable income. In these situations, it may be desirable to allocate income tax expenses
to profit centers not only to measure their economic profitability but also to motivate managers to minimize tax liability.
 'cc
c The quality of decisions may improve because they are being made by managers closest to the point of decision.c
c The speed of operating decisions may be increased since they do not have to be referred to corporate headquarters.
. Headquarters management, relieved of day-to-day decision making, can concentrate on broader issues.c
c Managers, subject to fewer corporate restraints, are freer to use their imagination and initiative.Because profit
centers are similar to independent companies, they provide an excellent training ground for general management.
Their managers gain experience in managing all functional areas, and upper management gains the opportunity to
evaluate their potential for higher-level jobs. c
c

c c
c Profit consciousness is enhanced since managers who are responsible' for profits will constantly seek ways to
increase them. (A manager responsible for marketing activities, for example, will tend to authorize promotion
expenditures that increase sales, whereas a manager responsible for profits will be motivated to make promotion
expenditures that increase profits.).c
c Profit centers provide top management with ready-made information on the profitability of the company's individual
components. . Because their output is so readily measured, profit centers are particularly responsive to pressures to
improve their competitive performance.
  '
c Decentralized decision making will force top management to rely more on management control reports than
on personal knowledge of an operation, entailing some loss of control.
c If headquarters management is more capable or better informed than the average profit center manager, the
quality of decisions made at the unit level may be reduced.
c Friction may increase because of arguments over the appropriate transfer price, the assignment of common
costs, and the credit for revenues that were formerly generated jointly by two or more business units working
together.
c Organization units that once cooperated as functional units may now be in competition with one another. An
increase in profits for one manager may mean a decrease for another. In such situations, a manager may fail
to refer sales leads to another business unit better qualified to pursue them; may hoard personnel or
equipment that, from the overall company standpoint, would be better off used in another unit; or may make
production decisions that have undesirable cost consequences for other units.
c Divisionalization may impose additional costs because of the additional management, staff personnel, and
record keeping required, and may lead to task redundancies at each profit center.

'0c&c c( 
c cc   c  
cc3/c ) cD&c(c  c  
c c   cè


c
  c
(c3( 
cc3/c ) c

 
c! cc   c  
'c
3
c/
c  
'c
In a diversified firm one of the important tasks of senior management is resource deployment, that is, make decisions
regarding the use of the cash generated from some business units to finance growth in other business units. Several planning
models have been developed to help corporate level managers of diversified firms to effectively allocate resources. These
models suggest that a firm has business units in several categories, identified by their mission; the appropriate strategies for
each category differ. Together, the several units make up a portfolio, the components of which differ as to their risk/reward
characteristics just as the components of an investment portfolio differ. Both the corporate 'office and the business unit
general manager are involved in identifying the missions of individual business units. Of the many planning models, two of
the most widely used are Boston Consulting Group's two-by-two growth-share matrix and General Electric Company/McKinsey
& Company's three-by-three industry attractiveness-business strength matrix. While these models differ in the methodologies
they use to develop the most appropriate missions for the various business units, they have the same set of missions from
which to choose: build, hold, harvest, and divest.
Gc ccccccc3 ('cThis mission implies an objective of increased market share, even at the expense of short-termcearnings and
cash flow (e.g., Merck's bio-technology, Black and Decker's handheld electricctools).c
Gc D('cThis strategic mission is geared to the protection of the business unit's market share and competitive position
(e.g.: IBM's mainframe computers). c
Gc D ) 'ccThis mission has the objective of maximizing short-term earnings and cash flow, even at the expense of
market share (e.g., American Brands' tobacco products, General Electric's and Sylvania's light bulbs)c
Gc  ) 'cThis mission indicates a decision to withdraw from the business either through a process of slow liquidation
or outright sale. While the planning models can aid in the formulation of missions, they are not cook books. A business unit's
position on a planning grid should not be the sole basis for deciding its mission.c
c
c

c c
Gc 3
c /
c "   ) c ()
 'c Every business unit should develop a competitive advantage in order to
accomplish its mission. Three interrelated questions have to be considered in developing the business unit's competitive=
advantage. First, what is the structure of the industry in which the business unit operates? Second, how should the business
unit exploit the industry's structure? Third, what will be the basis of the business unit's competitive advantage?c
Gc 
( c 
 'c Research has highlighted the important role industry conditions play in the performance of
individual firms. Studies have shown that average industry profitability is, by far, the most significant predictor of firm
performance. According to Porter, the structure of an industry should be analyzed in terms of the collective strength of five
competitive forces.c
1. The intensity of rivalry among existing competitors. Factors affecting direct rivalry are industry growth, product
differentiability, number and diversity of competitors, level of fixed costs, intermittent overcapacity, and exit barriers.
2. The bargaining power of customers. Factors affecting buyer power are number of buyers, buyer's switching costs, buyer's
ability to integrate backward, impact of the business unit's product on buyer's total costs, impact of the business unit's
product on buyer's product quality/ performance, and significance of the business unit's volume to buyers.
3. The bargaining power of suppliers. Factors affecting supplier power are number of suppliers, supplier's ability to integrate
forward, presence of substitute inputs, and importance of the business unit's volume to suppliers.
4. Threat from substitutes. Factors affecting substitute threat are relative price/performance of substitutes, buyer's switching
costs, and buyer's propensity to substitute.
5. The threat of new entry. Factors affecting entry barriers are capital requirements, access to distribution channels,
economies of scale, product differentiation, technological complexity of product or process, expected retaliation from existing
firms, and government policy.
Gc . c* c c  ) 
c& c  (cc c
( c
 'c
1. The more powerful the five forces are, the less profitable an industry is likely to be. In industries where average
profitability is high (such as soft drinks and pharmaceuticals), the five forces are weak (e.g., in the soft drink industry, entry
barriers are high). In industries where the average profitability is low (such as steel and coal), the five forces are strong (e.g.,
in the steel industry, threat from substitutes is high).
2. Depending on the relative strength of the five forces, the key strategic issues facing the business unit will differ from one
industry to another.
3. Understanding the nature of each force helps the firm to formulate effective strategies. Supplier selection (a strategic
issue) is aided by the analysis of the relative power of several supplier groups; the business unit should link with the supplier
group for which it has the best competitive advantage. Similarly, analyzing the relative bargaining power of several buyer
groups will facilitate selection of target customer segments.
Gc
c"   ) c()
 'c
The five-force analysis is the starting point for developing a competitive advantage since it helps to identify the opportunities
and threats in the external environment. With this understanding, Porter claims that the business unit has two generic ways
of responding to the opportunities in the external environment and developing a sustainable competitive advantage: low cost
and differentiation.
Gc &c "'c Cost leadership can be achieved through such approaches as economies of scale in production;
experience curve effects, tight cost control, and cost minimization (in such areas as research and development, service, sales
force, or advertising). Some firms following this strategy include Charles Schwab in discount brokerage, Wal-Mart in discount
retailing, Texas Instruments in consumer electronics, Emerson Electric in electric motors, Hyundai in automobiles, Dell in
computers, Black and Decker in machine tools, Nucor in steel, Lincoln Electric in arc welding equipment, and BIC in pens.c
Gc  
  
:
The primary focus of this strategy is to differentiate the product offering of the business unit, creating something that is
perceived by customers as being unique. Approaches to product differentiation include brand loyalty (Coca-Cola and Pepsi
Cola in soft drinks), superior customer service (Nordstrom in retailing), dealer network (Caterpillar Tractors in construction
equipment), product design and product features (Hewlett-Packard in electronics), and technology (Cisco in communications
infrastructure). Other examples of firms following a differentiation strategy include BMW in automobiles; Stouffer's in frozen
foods, Neiman-Marcus in retailing, Mont Blanc in pens, and Rolex in wristwatches.
c
c

c c
c
!c7
c .c cc 
  c
 c c
(c 
c( 
c cc$ 
  c"
 c& c*  c

$ 
  c
 'cc

A responsibility center is an organization unit that is headed by a manager who is responsible for its activities. In a sense, a
company is a collection of responsibility centers. Each of which is represented by box on the on the organization are
responsibility centers for section work shifts or other small organization units. At a higher level are departments or business
units that consist of several of these smaller units plus staff and management people these larger units are also responsibility
center. And from the stand point of senior management and the board of directors, the whole company is responsibility center
although the term is usually used to refer to unit within the company.

 cc 


  c
 c

A responsibility center exist one or more purpose are its objectives. The company as a whole has goals, and senior
management has decided on a set of strategies to accomplish these goals. The objectives of responsibility centers are to help
implement these strategies. Because the organization is the sum of its responsibility centers, if the strategies are sound and if
each responsibility center, if the strategies are sound and if each responsibility center meets its objectives the whole
organization should achieve its goals. A responsibility center uses inputs, and a variety of services. Its work with these
resources and it usually require working capital, equipment, and other asset to do this work. As a result of this work the
responsibility center produces output which is classified either as goods if they are tangible or as services if they are
intangible. Every responsibility center has output that is it does something. In a production plant, the outputs are goods. In
staff units, such as human resources, transportation, engineering, accounting, and administration, the output s are services.
For many responsibility centers, especially staff units, outputs are difficult to measure; nevertheless, they exist. The products
produced by a responsibility center or to the outside marketplace. In the first case, the product are inputs to the other
responsibility center in the latter case, they are output s of the whole organization.

! cc$ 


  c"
 c

"c"
 c

Cost centers are divisions that add to the cost of the organization, but only indirectly add to the profit of the company. Typical
examples include Research and Development, Marketing and Customer service. Companies may choose to classify business
units as cost centers, profit centers, or investment centers. There are some significant advantages to classifying simple,
straightforward divisions as cost centers, since cost is easy to measure. However, cost centers create incentives for managers
to underfund their units in order to benefit themselves, and this underfunding may result in adverse consequences for the
company as a whole (reduced sales because of bad customer service experiences, for example). Because the cost centre has
a negative impact on profit (at least on the surface) it is a likely target for rollbacks and layoffs when budgets are cut.
Operational decisions in a contact centre, for example, are typically driven by cost considerations. Financial investments in
new equipment, technology and staff are often difficult to justify to management because indirect profitability is hard to
translate to bottom-line figures. Business metrics are sometimes employed to quantify the benefits of a cost centre and relate
costs and benefits to those of the organization as a whole. In a contact centre, for example, metrics such as average handle
time, service level and cost per call are used in conjunction with other calculations to justify current or improved funding.

è  c"
 c
c

c c
A responsibility centre is called a profit centre when the manager is held responsible for both costs (inputs) and
revenues (outputs) and thus for profit. Despite the name, a profit centre can exist in nonprofits organizations
(though it might not be referred to as such) when a responsibility centre receives revenues for its services. A
profit centre is a big segment of activity for which both revenues and costs are accumulated: A centre, whose
performance is measured in terms of both - the expense it incurs and revenue it earns, is termed as a profit
centre. The output of a responsibility centre may either be meant for internal consumption or for outside
customers. In the latter case, the revenue is realized when the sales are made. That is, when the output is
meant for outsiders, then the revenue will be measured from the price charged from customers. If the output is
meant for other responsibility centre, then management takes a decision whether to treat the centre as profit
centre or not. In fact, any responsibility centre can be turned into a profit centre by determining a selling price
for its outputs. For instance, in case of a process industry, the output of one process may be transferred to
another process at a profit by taking into account the market price. Such transfers will give some profit to that
responsibility centre. Although such transfers do not increase the Company¶s assets, they help in management
control process.

c

) 
c"
 c

An investment centre goes a step further than a profit centre does. Its success is measured not only by its income but also by
relating that income to its invested capital, as in a ratio of income to the value of the capital employed. In
practice, the term investmentccentrecis not widely used. Instead, the term profitccentrecis used indiscriminately
to describe centers that are always assigned responsibility for revenues and expenses, but may or may not be
assigned responsibility for the capital investment. It is defined as a responsibility centre in which inputs are
measured in terms of cost / expenses and outputs are measured in terms of revenues and in which assets
employed are also measured. A responsibility centre is called an investment centre, when its manager is
responsible for costs and revenues as well as for the investment in assets used by his centre. He is responsible
for maintaining a satisfactory return on investment i.e. asset employed in his responsibility centre. The
investment centre manager has control over revenues, expenses and the amounts invested in the centre¶s
assets. The manager of an investment centre is required to earn a satisfactory return. Thus, return on
investment ($#) is used as the performance evaluation criterion in an investment centre. He also formulates
the credit policy, which has a direct influence on debt collection, and the inventory policy, which determines the
investment in inventory. The Vice President (Investments) of a mutual funds company may be in charge of an
Investment Centre. In the Investment Centre, the manager in charge is held responsible for the proper
utilization of assets. He is expected to earn a satisfactory return on the assets employed in his responsibility
centre. Measurement of assets employed poses many problems. It becomes difficult to determine the amount of
assets employed in a particular responsibility centre. Some of the assets are in the physical possession of the
responsibility centre while for some assets it may depend upon other responsibility centers or the Head Office of
the company. This is particularly true of cash or heavy plant and equipment. Whether such assets should be
included in the figure of assets employed of the responsibility centre and if included, at how much value, is a
difficult question. On account of these difficulties, investment centers are generally used only for relatively large
units, which have independent divisions, both manufacturing and marketing, for their individual products.

c  c
-c 
c  c   c c ) 
c c $ 
  c "
 c  c 
c  c c 
 c & c  c  c c
  
55 
 cc c   .

è  cc ) 
cc$ 
  c"
 c

1.c The organization is divided into various responsibility centers. Each responsibility centre is put under the charge of a
responsibility manager.

2.c The targets or budgets of each responsibility centre are set in consultation with the manager of responsibility centre,
so that he may be able to give full information about his department. The manager of responsibility centre should
know as what is expected of him - each centre should have a clear set of goals. The responsibility and authority of
each centre should be well defined.

3.c Managers are charged with the items and responsibility, over which they can exercise a significant degree of direct
control.

4.c Goals defined for each area of responsibility should be attainable with efficient and effective performance.
5.c The actual performance is communicated to the managers concerned. If it falls short of the standards, the variances
are conveyed to the top management. The names of persons responsible for the variances are also conveyed so that
responsibility may be fixed.

The purpose of all these steps is to assign responsibility to different individuals so that their performance is improved and
costs are controlled. The personal factor in Responsibility Accounting is most important. The management may prepare the
best plan or the budget and put up before its staff, but its success depends upon the initiative and the will of the workers to
execute it

 cc$ 


  c"

The Sarva Shiksha Abhiyan emphasizes quality improvement in elementary education for
which it deems necessary that resource groups and responsibility centers from national to sub-district levels are identified.
These groups would oversee the policy, planning, implementation and monitoring of all quality related interventions. Their
major role would be to advise and assist at various levels in curriculum development, pedagogical improvement, teacher
education/training and activities related to classroom transaction. In order to facilitate a decentralized mode of education,
these groups would need to be constituted at various operational levels, namely - national, state, district and sub district. The
following could be involved in the groups:c

National level - NCERT, NIEPA, Universities, NGOs, experts and eminent educationists.

State level - SCERT, SIEMAT, Universities, IASEs/CTEs, NGOs, experts and eminent educationists.

District level - DIETs, representatives from DPEP District Resource Group, higher educational institutions, innovative teachers
from the districts, NGOs.

Sub-district - BRC/BEO, representatives from CRCs, innovative teachers.

0c 3 c ( 
c    
 c 
 c "
 :c 

(c 
 c "
 :c è  c "
 c 
(c 
) 
c
"
 cD&c c ( c  (c
c   
 c
 c"
 c


(c 
 c
 'c

Engineered expense center have the following characteristics:

- Their inputs can be measured in monetary terms.


- Their output can be measured in physical terms.
- The optimal dollar amount of input required to produce one unit of output can be established.

Engineered expense center usually are found in manufacturing operations. Warehousing, distribution, trucking
and similar units in the marketing organization also may be engineered expense center and so many certain
responsibility center within administrative and support department. Examples are accounts receivable account
payable and payroll section in the controller department personnel record and cafeteria in the human resource
department shareholder record in the corporate secretary department and the company motor pool. Such units
perform repetitive task for which standard cost can be developed. In an engineered expense center the output
multiplied by the standard cost of each unit produced represents what the finished product should have cost.
When this cost is compared to actual costs, the difference between the two represents the efficiency of the
organization unit being measured. We emphasize that engineered expense centers have other important tasks
not measured by cast alone. The effectiveness of this aspect of performance should be controlled.
c c
For example expenses center supervisor are responsible for the quality of good and for the volume of
production in addition to their responsibility for cost efficiency. Therefore the type and amount of production is
prescribed and specific quality standards are set so that manufacturing costs are not minimized at the expense
of quality. Moreover manager of engineered expense center may be responsible for activities such a training
that are not related to current production judgment about their performance should include an appraisal of how
well they carry out these responsibilities. There are few if any responsibility center in which all cost items are
engineered. Even in highly automated production department the amount of indirect labor and of various
services used can vary with management discretion. Thus, the term engineered costs center refers to
responsibility center in which engineered cost predominate but in does not imply that valid engineering
estimates can be made for each and every cost item.

   
 c 
 c
 'c

The output of discretionary expenses center cannot be measured in monitory terms. They include
administration and support units research and development organization and most marketing activities. The
term discretionary does not mean that management judgment is capricious or haphazard. Management has
decided on certain policies that should govern the operation of the company. Whether to match exceed or
spend less than the marketing effort of its competitor; the level of service that the company provides to the
customer. The appropriate amount of spending for R & D, financial planning public relation and many other
activities. One company may have a small headquarter staff another company of similar size and in the same
industry may have a staff that is 10 times as large. the management of both companies may be concerned that
they made the correct decision on staff size but there is no objective way judging which decision was actually
better manager are hired and paid to make such decision. After such a drastic change the level of discretionary
expenses generally has a similar pattern from one year to the next. The difference between budgeted and
actual expense is not a measure of efficiency in a discretionary expense center it is simply the difference
between the budgeted input and the actual input. It in no way measures the value of the output. if actual
expense do not exceed the budget amount, the manager has µlived within the budget µ however ,because by
definition the budget does not purport to measure the optimum amount of spending we cannot say that living
within the budgeted is efficient performance.

è  c"
 c
c
A responsibility centre is called a profit centre when the manager is held responsible for both costs (inputs) and
revenues (outputs) and thus for profit. Despite the name, a profit centre can exist in nonprofits organizations
(though it might not be referred to as such) when a responsibility centre receives revenues for its services. A
profit centre is a big segment of activity for which both revenues and costs are accumulated: A centre, whose
performance is measured in terms of both - the expense it incurs and revenue it earns, is termed as a profit
centre. The output of a responsibility centre may either be meant for internal consumption or for outside
customers. In the latter case, the revenue is realized when the sales are made. That is, when the output is
meant for outsiders, then the revenue will be measured from the price charged from customers. If the output is
meant for other responsibility centre, then management takes a decision whether to treat the centre as profit
centre or not. In fact, any responsibility centre can be turned into a profit centre by determining a selling price
for its outputs. For instance, in case of a process industry, the output of one process may be transferred to
another process at a profit by taking into account the market price. Such transfers will give some profit to that
responsibility centre. Although such transfers do not increase the Company¶s assets, they help in management
control process.


) 
c"
 c

An investment centre goes a step further than a profit centre does. Its success is measured not only by its
income but also by relating that income to its invested capital, as in a ratio of income to the value of the capital
employed. In practice, the term investmentccentrecis not widely used. Instead, the term profitccentrecis used
indiscriminately to describe centers that are always assigned responsibility for revenues and expenses, but may
or may not be assigned responsibility for the capital investment. It is defined as a responsibility centre in which
inputs are measured in terms of cost / expenses and outputs are measured in terms of revenues and in which
assets employed are also measured. A responsibility centre is called an investment centre, when its manager is
responsible for costs and revenues as well as for the investment in assets used by his centre. He is responsible
for maintaining a satisfactory return on investment i.e. asset employed in his responsibility centre. The
investment centre manager has control over revenues, expenses and the amounts invested in the centre¶s
assets. The manager of an investment centre is required to earn a satisfactory return. Thus, return on
investment ($#) is used as the performance evaluation criterion in an investment centre. He also formulates
the credit policy, which has a direct influence on debt collection, and the inventory policy, which determines the
investment in inventory.

c c
3( cè   
c

The decision that management make about a discretionary expense budget are different from the decisions that
it makes about the budget for an engineered expense center. For the latter management decides whether the
proposed operating budget represent the cost of performing task efficiently for the coming period. management
is not so much concerned with the magnitude of the task because this is largely determined by the actions of
other responsibility centers, such as the marketing departments ability to generate sales. In formulating the
budget for a discretionary expense center, however management principal task is to decide on the magnitude of
the job that should be done. These tasks can be divided generally into two types continuing and special.
Continuing task are those that continue from year to year for example financial statement preparation by the
controller¶s office. Special tasks are one shot project for example developing and installing a profit budgeting
system in a newly acquired division. The technique management by objective is often used in preparing the
budget for a discretionary expense center. Management by objective is a formal process in which a budget
purposes to accomplish specific tasks and state a mean for measuring whether these tasks have been
accomplished. There are two different approach to planning for the discretionary expense center increment
budgeting and zero based review.

!cIc

c  c

c(   c  
cc  c
 cc c
(c()
 c  c
/
( c& c  
c  
cc  c
 c c
c()   c

/
( c& c  
c  
cc  c
 c c
c()   c
Decentralized decision making will force top management to rely more on management control reports than on personal
knowledge of an operation, entailing some loss of control. If headquarters management is mere capable or better informed
than the average profit center manager, the quality of decisions made at the unit level way be reduced. Friction may increase
because of arguments over the appropriate transfer price, the assignment of common costs, and the credit for revenues that
were formerly generated jointly by two or more business units working together.

Organization units that once cooperated as functional units may now be in competition with one another. An increase in
profits for one manager may mean a decrease for another. In such situation a manager may fail to refer sales leads to
another business unit better qualified to pursue them; may hoard personnel or equipment that, from the overall company¶s,
standpoint, would be better off used in another unit; or may make production decisions that have undesirable cost
consequences for other units.

Divisionalization may impose additional costs because of the additional management, staff personnel, and record keeping
required, and may lead to task redundancies at each profit center.Competent general managers may not exist in a functional
organization because there may not have been sufficient opportunities for them to develop general management competence.
There may be too much emphasis on short-run profitability at the expense of long-run profitability. In the desire to report
high current profits, the profit center manager may skimp on R&D, training programs, or maintenance. This tendency is
especially prevalent when the turnover of profit center managers is relatively high. In these circumstances, managers may
have good reason to believe that their actions may not affect profitability until after they have moved to other jobs. There is
no completely satisfactory system for ensuring that optimizing the profits of each individual profit center will optimize the
profits of the company as a whole.

-.c c c


 c (c
c 
c   c )  c ) ( (cc3
c/
cccc 
cc@  c

 Ac

3
c/
ccè  c"
 c
Most business units are created as profit centers since managers in charge of such units typically control product
development, manufacturing, and marketing resources. These managers are in a position to influence revenues and costs and
as such can be held accountable for the "bottom line." However, as pointed out in the next section, a business unit manager's
authority may be constrained in various ways, which ought to be reflected in a profit center's design and operation.

"
 
c
c3
c/
c c
To realize fully the benefits of the profit center concept, the business unit manager would have to be as autonomous as the
president of an independent company. As a practical matter, however, such autonomy is not feasible. If a company were
divided into completely independent units, the organization would lose the advantages of size and synergy. Furthermore in
delegating to business unit management all the authority that the board of directors has given to the CEO, senior
management would be abdicating its own responsibility.
Consequently, business unit structures represent trade-offs between business unit autonomy and corporate constraints. The
effectiveness of a business unit organization is largely dependent on how well these trade-offs are made.
c

c c
"
 
c c# c3
c/
c
One of the main problems occurs when business units must deal with one another. It is useful to think of managing a profit
center in terms of control over three types of decisions:
(1) The product decision (what goods or services to make and sell),
(2) The marketing decision (how, where, and for how much are these goods or services to be sold?), and
(3) The procurement or sourcing decision (how to obtain or manufacture the goods or services). If a business unit manager
controls all three activities, there is usually no difficulty in assigning profit responsibility and measuring performance. In
general, the greater the degree of integration within a company,
the more difficult it becomes to assign responsibility to a single profit center for all three activities in a given product line; that
is, if the production, procurement, and marketing decisions for a single product line are split among two or more business
units, separating the contribution of each business unit to the overall success of the product line may be difficult.
c
 c  c  c c
The constraints imposed by corporate management can be grouped into three types:
(1) Those resulting from strategic considerations,
(2) Those resulting because uniformity is required, and
(3) Those resulting from the economies of centralization.

Most companies retain certain decisions, especially financial decisions, at the corporate level, at least for domestic activities.
Consequently, one of the major constraints on business units results from corporate control over new investments. Business
units must compete with one another for a share of the available funds. Thus, a business unit could find its expansion plans
thwarted because another unit has convinced senior management that it has a more

Attractive program. Corporate management .also imposes other constraints. Each business unit has a "charter" that specifies
the marketing and/or production activities that it is permitted to undertake, and it must refrain from operating beyond its
charter, even though it sees profit opportunities in doing so. Also, the maintenance of the proper corporate image may
require constraints on the quality of products or on public relations activities.

Companies impose some constraints on business units because of the necessity for Uniformity. One-constraint is that
business Units must conform to corporate accounting and MCS This constraint is especially troublesome for units that have
been acquired from another company and that have been accustomed to using different systems.
c

0c.  c c c

c
c â c3 (c3( 
c

-c 

c"
 Jc

â c3 (c3( 
''câero-based budgeting is a technique of planning and decision-making which reverses the working
process of traditional budgeting. In traditional incremental budgeting, departmental managers justify only increases over the
previous year budget and what has been already spent is automatically sanctioned. No reference is made to the previous level
of expenditure. By contrast, in zero-based budgeting, every department function is reviewed comprehensively and all
expenditures must be approved, rather than only increases.[1] âero-based budgeting requires the budget request be justified
in complete detail by each division manager starting from the zero-base. The zero-base is indifferent to whether the total
budget is increasing or decreasing.c

The term "zero-based budgeting" is sometimes used in personal finance to describe the practice of budgeting every
dollar of income received, and then adjusting some part of the budget downward for every other part that needs to be
adjusted upward. It would be more technically correct to refer to this practice as "active-balanced budgeting".

Advantages of âero-Based Budgeting:

1.c Efficient allocation of resources, as it is based on needs and benefits.


2.c Drives managers to find cost effective ways to improve operations.
3.c Detects inflated budgets.
4.c Municipal planning departments are exempt from this budgeting practice.
5.c Useful for service departments where the output is difficult to identify.
6.c Increases staff motivation by providing greater initiative and responsibility in decision-making.
7.c Increases communication and coordination within the organization.
8.c Identifies and eliminates wasteful and obsolete operations.
9.c Identifies opportunities for outsourcing.
10.c Forces cost centers to identify their mission and their relationship to overall goals.

c c
Disadvantages of âero-Based Budgeting:

1.c Difficult to define decision units and decision packages, as it is time-consuming and exhaustive.
2.c Forced to justify every detail related to expenditure. The R&D department is threatened whereas the production
department benefits.
3.c Necessary to train managers. âero-based budgeting must be clearly understood by managers at various levels to be
successfully implemented. Difficult to administer and communicate the budgeting because more managers are
involved in the process.
4.c In a large organization, the volume of forms may be so large that no one person could read it all. Compressing the
information down to a usable size might remove critically important details.
5.c Honesty of the managers must be reliable and uniform. Any manager that exaggerates skews the results



c"
 'c

Internal control is defined as a process affected by an organization's structure, work and authority flows, people and
management information systems, designed to help the organization accomplish specific goals or objectives.[1] It is a means
by which an organization's resources are directed, monitored, and measured. It plays an important role in preventing and
detecting fraud and protecting the organization's resources, both physical (e.g., machinery and property) and intangible
(e.g., reputation or intellectual property such as trademarks). At the organizational level, internal control objectives relate to
the reliability of financial reporting, timely feedback on the achievement of operational or strategic goals, and compliance
with laws and regulations. At the specific transaction level, internal control refers to the actions taken to achieve a specific
objective (e.g., how to ensure the organization's payments to third parties are for valid services rendered.) Internal control
procedures reduce process variation, leading to more predictable outcomes

`  cc  c

Internal controls may be described in terms of: a) the objective they pertain to; and b) the nature of the control activity itself.

Objective categorization

Internal control activities are designed to provide reasonable assurance that particular objectives are achieved, or related
progress understood. The specific target used to determine whether a control is operating effectively is called the control
objective. Control objectives fall under several detailed categories; in financial auditing, they relate to particular financial
statement assertions,[5] but broader frameworks are helpful to also capture operational and compliance aspects:

1.c Existence (Validity): Only valid or authorized transactions are processed (i.e., no invalid transactions)
2.c Occurrence (Cutoff): Transactions occurred during the correct period or were processed timely.
3.c Completeness: All transactions are processed that should be (i.e., no omissions)
4.c Valuation: Transactions are calculated using an appropriate methodology or are computationally accurate.
5.c Rights & Obligations: Assets represent the rights of the company, and liabilities its obligations, as of a given date.
6.c Presentation & Disclosure (Classification): Components of financial statements (or other reporting) are properly
classified (by type or account) and described.
7.c Reasonableness-transactions or results appear reasonable relative to other data or trends.

Activity categorization

Control activities may also be described by the type or nature of activity. These include (but are not limited to):

c Segregation of duties - separating authorization, custody, and record keeping roles to limit risk of fraud or error by
one person.
c Authorization of transactions - review of particular transactions by an appropriate person.
c Retention of records - maintaining documentation to substantiate transactions.
c Supervision or monitoring of operations - observation or review of ongoing operational activity.
c Physical safeguards - usage of cameras, locks, physical barriers, etc. to protect property.
c Analysis of results, periodic and regular operational reviews, metrics, and other key performance indicators (KPIs).
c IT Security - usage of passwords, access logs, etc. to ensure access restricted to authorized personnel. S

1c=
cè)c(:ccc  (c
c c*
c) c cc
 
c
c4c cD
(
c
 ) c.c
 c
c c
 c c&(cc  c
(c&c

Since Veena is a small multiproduct company it would require changes in control system which would be related to transfer
pricing a, as this company would generally provide inputs to HUL. Thus the domestic operations generally involve transfer of
goods and services only In view of this difference many other considerations, in addition to the criteria used in domestic
operations for the determination of transfer price, are involved. These include:

c c
(a) Fair Price: This is an important factor one needs to consider while determining the transfer price for foreign operations.
Companies that enter into joint ventures must ensure that the transfer price charged is fair. If such companies charge a
higher transfer price, it would reduce the profits of the joint venture and as a result reduce the foreign partner's share of
profits.

(b) Government Regulations: All countries have a regulatory framework under which business units operate. Where
government rules and regulations regarding transfer prices are lenient, the parent company should fix a higher transfer price
for all transfers to countries with high income tax rates. This approach would enable the parent company to minimize taxes in
such countries.

(c) Exchange Control Restrictions: Every country has foreign exchange control regulations.
These regulations impose a limit on the amount of foreign exchange available for the import of certain goods. To
accommodate the foreign subsidiary the parent company may have a lower transfer price so that the subsidiary is able to
import a larger quantity of required goods.

(d) Income Tax Regulations: The rates of income tax vary from country to country. To overcome this difference the transfer
price should be so fixed that countries with low tax rates show profits while others end up with a loss. This helps the parent
company to reduce its taxes on a global basis.

(e) Desire to accumulate funds: A company that wishes to accumulate funds in a particular country may fix the transfer prices
in such a manner that it facilitates shifting of funds into that country.

(I) Tariffs- and Duties: No country likes high imports. In order to restrict imports countries impose restrictions such as
quantitative restrictions, high duties and tariffs and banning import of products. The general practice is to charge import
duties as a percentage of the value of products imported, although a lower tariff may be levied if the import value is lower. It
is seen that the impact of tariffs on the profitability of foreign operations is generally the reverse of the incidence of income
taxes in transfer pricing. As such a low transfer price would lead to low import duties on transfer, the profit arising in that
country would be high. This results in high income taxes in that country. It is therefore advisable that companies must
compute the net effect of these factors while determining transfer prices.

In designing performance evaluation systems for acquired Veena company,HUL could use the following guidelines

Subsidiary managers should not be held responsible for translation effects. The simplest way to achieve this objective is to
compare budgets and actual results using the same metric and isolate inflation-related effects through variance analysis. It is
pointless for managers to worry about the appropriate metric. The MNE should choose whatever metric is more convenient.

Transaction effects are best handled through centralized coordination of the MNE's overall hedging needs. This is likely to be
cheaper and simpler, and it prevents the subsidiary manager from becoming a foreign exchange rate forecaster and
speculator.

The subsidiary manager should be held responsible for the dependence effects of exchange rates resulting from economic
exposure.

Evaluation of the subsidiary as a basis for a decision to locate operations in a country or to relocate operations from a country
should reflect the consequences of translation. Transaction and economic exposures.

c c
!c6c

cWhat are the Special Characteristics of Professional Service Organization?c


& 'c
c
c
A dominant goal of a manufacturing company is to earn a satisfactory profit, specifically a satisfactory return on
assets employed. A professional organization has relatively few tangible assets; its principal asset is the skill of
its professional staff, which 'doesn't appear on its balance sheet. Return on assets employed, therefore, is
essentially meaningless in such organizations. Their financial goal is to provide adequate compensation to the
professionals.

In many organizations, a related goal is to increase their size. In part, this reflects the natural tendency to
associate success with large size. In part, it reflects economies of scale in using the efforts of a central
personnel staff and units responsible for keeping the organization up-to-date. Large public accounting firms
need to have enough local offices to enable them to audit clients who have facilities located throughout the
world.

è   
cc
Professional organizations are labor intensive, and the labor is of a special type. Many professionals prefer to
work independently, rather than as part of a team. Professionals who are also managers tend to work only part
time on management activities; senior partners in an accounting firm participate actively in audit engagements;
senior partners in law firms have clients. Education for most professions does not include education in
management, but quite naturally stresses the skills of the profession, rather than management; for this and
other reasons, professionals tend to look down on managers.

Professionals tend to give inadequate weight to the financial implications of their decisions; they want to do the
best job they can, regardless of its cost. This attitude affects the attitude of support staffs and nonprofessionals
in the organization; it leads to inadequate cost control.
c
#c
(c
c  
c
The output of a professional organization cannot be measured in physical terms, such as units, tons, or gallons.
We can measure the number of hours a lawyer spends on a case, out this is a measure of input, not output.
Output is the effectiveness of the lawyer's work, and this is not measured by the number of pages in a brief or
the number of hours in the courtroom. We can measure the number of patients a physician Teats in a day, and
even classify these visits by type of complaint; but this is by no means equivalent to measuring the amount or
quality of service the physician has provided. At most, what is measured is tl1e physician's efficiency in treating
patients, which is of some use in identifying slackers and hard workers. Revenues earned is one measure of
output in some professional organizations, but these monetary amounts, at most, relate to the quantity of
services rendered, not to their quality (although poor quality is reflected in reduced revenues in the long run).

Furthermore, the work done by many professionals is non repetitive. No two consulting jobs or research and
development projects are quite the same. This makes it difficult to plan the time required for a task, to set
reasonable standards for task performance, and to judge how satisfactory the performance was. Some tasks
are essentially repetitive: the drafting of simple wills, deeds, sales contracts, and similar documents; the taking
of a physical inventory by an auditor; and certain medical and surgical procedures. The development of
standards for such tasks may be worthwhile, although in using these standards, usual circumstances that affect
a specific job must be taken into account. Some professionals, notably scientists, engineers, and professors, are
reluctant to keep track of how they spend their time, and this complicates the task of measuring performance.
This reluctance seems to have its roots in tradition; usually, it can be overcome if senior management is willing
to put appropriate emphasis on the necessity of accurate time reporting. Nevertheless, difficult problems arise
in deciding how time should be charged to clients.

If the normal work week is 40 hours, should a job be charged for 1140th of a week's compensation for each
hour spent on it? If so, how should work done on evenings and weekends be counted? (Professionals are
"exempt" employees-that is, they are not subject to government requirements of overtime payments.) How to
account for time spent reading literature, going to meetings, and otherwise keeping up-to-date?
c
c + c
With a few exceptions, such as some law firms and accounting firms, professional organizations are relatively
small and operate at a single location. Senior management in such organization can personally observe what is
going on and personally motivate employees. Thus, there is less need for a sophisticated management control
system, with profit centers and formal performance reports. Nevertheless, even a small organization needs a
budget, a regular comparison of performance against budget, and a way of relating compensation to
performance.

c c
D&c c * 
c(
c
c c
In a manufacturing company there is a clear dividing line between marketing activities and production
activities; only senior management is concerned with both. Such a clean separation does not exist in most
professional organizations. In some, such as law, medicine, and accounting, the profession's ethical code limits
the amount and character of overt marketing efforts by professionals (although these restrictions have been
relaxed in recent years). Marketing is an essential activity in almost all organizations, however. If it can't be
conducted openly, it takes the form of personal contacts, speeches, articles, conversations on the golf course,
and so on. These marketing activities are conducted by professionals, usually by professionals who spend much
of their time in production work-that is, working for clients.
In this situation, it is difficult to assign appropriate credit to the person responsible for "selling" a new customer.
In a consulting firm, for example, a new engagement may result from a conversation between a member of the
firm and an acquaintance in a company, or from the reputation of one of the professionals as an outgrowth of
speeches or articles.
Moreover, the professional who is responsible for obtaining the engagement may not be personally involved in
carrying it out. Until fairly recently, these marketing contributions were rewarded subjectively-that is, they were
taken into account in promotion and compensation decisions. Some organizations now give explicit credit,
perhaps as a percentage of the project's revenue, if the person who "sold" the project can be identified.
c
D&c(c& c ) c cè  
 c  cc
As noted earlier in regard to teachers, at the extremes the performance of professionals is easy to judge.
Appraisal of the large percentage of professionals who are within the extremes is much more difficult. For some
professions, objective measures of performance are sometimes unavailable: The recommendations of an
investment analyst can be compared with actual market behavior of the securities; the accuracy of a surgeon's
diagnosis can be verified by an examination of the tissue that was removed; and the doctors' skill can be
measured by the success ratio of operations. These measures are, of course, subject to appropriate
qualifications, and in most circumstances the assessment of performance is finally a matter of human judgment
by superiors, peers, self, subordinates, and clients. Judgments made by superiors are the most common. For
these, professional organizations increasingly use formal systems to collect performance appraisals as a basis
for personnel decisions and for discussion with the professional. Some systems require numerical ratings of
specified attributes of performance and provide for a weighted average of these ratings. Compensation may be
tied, in part, to these numerical ratings. In a matrix organization, both the project leader and the head of the
functional unit that is the professional's organizational "home" judge performance. "

Appraisals by a professional's peers, or by subordinates, are sometimes part of a formal control system. In
some organizations, individuals may be asked to make a self-appraisal. Expressions of satisfaction or
dissatisfaction from clients are also an important basis for judging performance, although such expressions may
not always be readily forthcoming.

The budget can be used as the basis for measuring cost performance, and the actual time taken can be
compared with the planned time. Budgeting and control of discretionary expenses are as important in a
professional firm as in a manufacturing company.

Such financial measures are relatively unimportant in assessing a professional's contribution to the firm's,
profitability, however. The professional's major contribution is related to quantity and above all quality of work,
and its appraisal must be largely subjective. Furthermore, the appraisal must be made currently; it cannot wait
until one learns whether a new building is well designed, a new control system actually works well, or a bond
indenture has a flaw.

In some professions, internal audit procedures are used to control quality. In many accounting firms, the report
of an audit is reviewed by a partner other than the one who is responsible for it, and the work of the whole firm
is "peer reviewed" by another firm. The proposed design of a building may be reviewed by architects who are
not actively involved in the project.

c
c c
-c.c cc
c5cè  c# 
+ 
cD&c c c  
 cc c 
+ 
c ) (c


& 'c
c

 ( 
c
A nonprofit organization, as defined by law, is an organization that cannot distribute assets or income to, or for
the benefit of, its members, officers, or directors. The organization can, of course, compensate its employees,
including officers and members, for services rendered and for goods supplied. This definition does not prohibit
an organization from earning a profit; it prohibits only the distribution of profits. A nonprofit organization needs
to earn a modest profit, on average, to provide funds for working capital and for possible ³rainy days.´

è  
 c ) 
cc

  c 
+ 
c
For any organization, the most important reasons to measure performance are to improve effectiveness and to
acquire information that will allow the organization to drive its agenda forward. If the motivation for doing
evaluation remains outside an organization, the evaluation will have limited impact. To do performance
assessment effectively, an organization must commit to adopting a culture of measurement, because
acceptance must come from senior management, staff, funders, and board members alike.

Gc 3 (c 5 ) 


c
Members of the Board of Directors should regularly evaluate the quality of their activities on a regular basis.
Activities might include staffing the Board with new members, developing the members into well-trained and
resourced members, discussing and debating topics to make wise decisions, and supervising the CEO. Probably
the biggest problem with Board self-evaluation is that it does not occur frequently enough. As a result, Board
members have no clear impression of how they are performing as members of a governing Board. Poor Board
operations, when undetected, can adversely affect the entire organization.
Gc c
(c)
 c4
( ) (c  
 c ) 
c
Most of us are familiar with employee performance appraisals, which evaluate the quality of an individual¶s
performance in their position in the organization. Ideally, those appraisals reference the individual¶s written job
description and performance goals to assess the quality of the individual¶s progress toward achieving the
desired results described in those documents. Continued problems in individual performance often are the
results of poor strategic planning, program planning and staff development. If overall planning is not done
effectively, individuals can experience continued frustration, stress and low morale, resulting in their poor
overall performance. Experienced leaders have learned that continued problems in performance are not always
the result of a poor work ethic ± the recurring problems may be the result of larger, more systemic problems in
the organizations.
Gc è  c ) 

Program evaluations have become much more common, particularly because donors demand them to ensure
that their investments are making a difference in their communities. Program evaluations are typically focused
on the quality of the program¶s process, goals or outcomes. An ineffective program evaluation process often is
the result of poor program planning ± programs should be designed so they can be evaluated. It can also be the
result of improper training about evaluation. Sometimes, leaders do not realize that they have the responsibility
to verify to the public that the nonprofit is indeed making a positive impact in the community. When program
evaluations are not performed well, or at all, there is little feedback to the strategic and program planning
activities. When strategic and program planning are done poorly, the entire organization is adversely effected.
Gc ) 
cc 5
 
c   c
Cross-functional processes are those that span several systems, such as programs, functions and projects.
Common examples of major processes include information technology systems and quality management of
services. Because these cross-functional processes span so many areas of the organization, problems in these
processes can be the result of any type of ineffective planning, development and operating activities.
Gc # 
+ 
c ) 
c
Ongoing evaluation of the entire organization is a major responsibility of all leaders in the organization. Leaders
sometimes do not recognize the ongoing activities of management to actually include organizational evaluations
± but they do. The activities of organizational evaluation occur every day. However, those evaluations usually
are not done systematically. As a result, useful evaluation information is not provided to the strategic and
program planning processes. Consequently, both processes can be ineffective because they do not focus on
improving the quality of operations in the workplace.
c c
0cc. c )   (c
c9cè c

 
c(c c
cc c( )  
ccc cc c
&
c
c
 c
c&c 
 
c
 c
c c
(c
 c  ( c
c c
  (c.cc
c

& 'c
As, we Pritam International is a well diversified company. Sometimes, excessive diversification and that too in
unrelated lines of business causes failure in the business operations. One of the major reason for failures of
many Mergers and Diversification is excessive diversification. As, excessive diversification is ominous especially,
in unrelated lines of business. As, there may be no advantage of operating synergy. Neither through:

I)c Sharing common resources nor


II)c Sharing common core competencies
Therefore, it may be a strategic decision by the promoters and directors of the company to sell one of its
divisions. As, this may be impacting their core business. Sometimes, your core business tends to get neglected
mainily due to excesive diversification. As, the division is being sold to its own company managers. There,
might not be major changes in management control and systems. As, most of its managers will be the same.
But, they will have more autonomy to take decisions independently after acquisition. Now there will be less red
tapism and managers can take more risk. The managers will manage the firm in their own style. As, they are
not answerable to their superiors.

!cc

.c 3
 c  c " (c c 
 ( (c   c c  c  (c c è  
 c   c
è  c3
 c c" (c c
c 
+ 
cc c   c& c

.c c c3


 (c  (
The rationale for the development of the Balanced Scorecard was a growing dissatisfaction with traditional,
financial measures of performance. These measures suffer from a number of serious drawbacks in that they
take a short-term, lagged (i.e., historic) view of performance. The shift towards flexible, lean production/service
systems in many firms has strengthened the requirement for performance measurement systems to become
more broadly based, incorporating both non-financial and external measures of performance. According to
Kaplan and Norton, the Balanced Scorecard provides a better assessment of performance as it "enables
companies to track financial results while simultaneously monitoring progress in building the capabilities and
acquiring the intangible assets they need for future growth".

The original scorecard designed by Kaplan and Norton contained four key groupings of performance measures.
These four groupings, called µperspectives¶ by Kaplan and Norton, were considered sufficient to track the key
drivers of both current and future financial performance of the firm. The perspectives focused on the
achievements of the firm in four areas: namely the financial, customer, internal business process and
innovation/learning perspectives. The four perspectives can be represented as an interlinked hierarchy. The
firm¶s strategy underlies the whole scorecard, as the measures for each of the four perspectives are drawn from
this strategy.

c  c
To obtain a satisfactory overview of performance, the scorecard will require a mix of lagging and leading
(forward looking) measures. Financial measures tend to be lagged and consequently, the measures chosen for
the other perspectives will need to include leading measures. In general, outcome measures tend to be lagged,
for example, current market share is the result of past decisions and consequently is a lagging measure. Thus
the challenge in designing a Balanced Scorecard is to choose driver measures which lead changes in the
outcome measures in the non-financial perspectives and which ultimately drive the financial measures.

Once the firm¶s objectives have been agreed and the appropriate outcome and driver measures chosen for each
of the perspectives, firm and managerial performance is assessed by comparing actual attainment on each
measure with the target set for that measure.

Objective Measure Target Actual

3
 c c(
c c3
 (c  (c c
There are several benefits from implementing a Balanced Scorecard. Originally the Balanced Scorecard was
seen as a useful tool for performance measurement. In this role, the Balanced Scorecard was seen as
integrating financial/non-financial, internal/external and leading /lagging information on firm performance in a
coherent fashion.

Later it was realised that the Balanced Scorecard could play a pivotal role in the strategic management process.
Because the Balanced Scorecard requires management to clarify and obtain consensus on the strategic
objectives of the firm, it can assist in the communication of the chosen strategy, consequently aligning the
efforts both of individuals and of departments. In this role, there is a clear link between the Balanced Scorecard
and management by objectives (MBO). Effective implementation of a Balanced Scorecard project will generally
involve the development of a series of hierarchical (cascaded) scorecards. Given the overall corporate
scorecard, supporting scorecards can be developed for each department within the firm. Within each
department, a scorecard can be developed for each manager (or perhaps even for each individual member of
staff) which links the objectives on each perspective for that manager back to the objectives for each
perspective outlined in the scorecard for the department and finally, back to the objectives listed in the firm¶s
overall scorecard.

The Balanced Scorecard could be used to assist in corporate restructuring. In recent years, many firms have
migrated away from a traditional hierarchical structure to a flatter, team-based organisational structure. The
Balanced Scorecard can support such changes, as it can help clarify the objectives and the critical success
factors for the newly formed teams.
c c
Apart from the communication and co-ordination roles of the Balanced Scorecard in strategic implementation,
the Balanced Scorecard can be used to link strategy to specific critical success factors in the customer, internal
business process and growth/learning perspectives. By setting both short and long-term targets for driver and
outcome measures and by comparing actual attainment against target, feedback is obtained on how well the
strategy is being implemented and on whether the strategy is working.

Building on the Balanced Scorecard¶s use as a strategic management tool, it has been suggested that the
Balanced Scorecard can play a role in the investment appraisal process(5). Traditional methods of investment
appraisal such as discounted cash flow do not cope well with investments which generate indirect rather than
direct financial returns. Examples of these include investments which enhance the future µflexibility¶ of a firm or
investments in the firm¶s infrastructure, such as an enhanced management information system. The Balanced
Scorecard can assist management¶s investment appraisal decisions as it provides managers with a mechanism
to incorporate the strategic aspects of the investment into the appraisal process. This could be achieved by
using a weighting system developed from a firm¶s Balanced Scorecard measures to evaluate new projects. An
index score would be calculated for each investment opportunity and projects would then be ranked and
selected based on this score.

3
 c c" (cc" ( c" (c"


c c
c-c


c "
c c &c  )  
'c c <c 3c $ 
c 
c 
) 
c  c c ( )  
c c -6Gc   c  c  )
c
&'5c

è   c  )cc  )c3c

 )  
c c 1666666c IC66666c

 )  
c
) 
c -666666c 0-66666c

è  c 166666c C16666c


 c
(c
c
c( )  
c  
 cc c

.$c

cccè  c 
c>cè  ccK66c c cccccccccccc
 c

Profit Margin for Division µA¶= 4,00,000 /40,00,000 *100 = 10%

Profit Margin for Division µB¶ = 6,40,000/ 96,00,000 *100 = 6.6%

!
) cc
) 
c>c cKc66c

ccccccccccccccccccccccccccccccccccccccccccc
) 
c

Turnover of Investment for Division µA¶ = 40,00,000/20,00,000 = 2 times

Turnover of Investment for Division µB¶ = 96,00,000/32,00,000 = 3 times

As Return on investment for both Divisions A and B is 20%.

"#!'5c

 )  
cLMc9cAlthough µA¶ has more profit margin than Division µB¶ that is 10% as compared to 6.6% of µB¶, so it has more
profitability but inspite of it, division µA¶ has lower turnover of investment that its assets management is bad than Division µB¶,
it can be improved by increased sales or reducing investment.

 )  
c L3Mc 9 Needs to improve profit margin by increasing sales and reduce variable cost and sales at same price or by
reducing salesprice and increase the volume of sales so that its profit would improve. As it has good assets management
shown by its turnoverof Division µB¶ that is 3 times which is better than Division µA¶. So it can become profitable organisation
by improving Profit Margin.

c c
1 c
(c   c( 
 c
(c    c &
cc

- Strategy Formulation and Management Control

- Management Control and Task Control

 c 
 
c &
c  c£  
c
(c
 
c"
 cc

"   c   c£  


cc 
 
c"
 c

a) Focus of plan On one aspect at a time On entire organisation

b) Complexities Many variables hence complex Less complex

c) Nature of information Tailor-made for the issue, more Integrated, more internal and
external and predictive, less historical, more accurate.
accurate.

d) Structure Unstructured and irregular, each Rhythmic, definite pattern, set


problem being different procedure

e) Communication of information Relatively simple Relatively difficult

f) Purpose of estimates Show expected results Lead to desired result

g) Persons involved Staff and top management Line and top management

h) No. of persons involved Small Large

i) Mental activity Creative, analytical Administrative, persuasive

j) Planning and control Planning dominant but some Emphasis on both planning and
control control

k) Time horizon Tends to be long Tends to be short

l) End result Policies and precedents Action within policies laid

m) Appraisal of job done Extremely difficult Less difficult

c c 
 
c &
c
 
c"
 c
(c!*c"
 c

"   c !*c"


 c 
 
c
 c

a)cFocus of planc Single task or transaction On entire organisation

b)cNature of informationc Tailor-made to operation, specific, Integrated, more internal and


often non- financial, real time historical, more accurate

c) Persons involvedc Supervisors Line and top management

d) Mental activity Follow directives or none as in case Administrative, persuasive


of machines or set objectives

e) Time horizon Day to day Tends to be short

c
c
c
c
c
c
c
c
c
c
c
c
c c
c
c
c
c
!c-c

-c c(c4 c4"5-66Ec

(a)c Define profit in this case and prepare a statement for both divisions and overall company.

Solution:

cè    c 
cc )  
c'5c

Particulars Amount(Rs.)
Selling price p.u. 35
Variable Cost p.u. 11
Contribution p.u. 24

Contribution p.u. Expected sales Total contribution Total Fixed cost Net profit (Rs.)
(no. of units) (Rs.)
24 2000 48000 60000 (12000)
24 3000 72000 60000 12000
24 6000 144000 60000 84000

cè    c 
cc )  
c3'5c

Selling p.u. Total Contribution Expected Total Total Fixed Net profit
variable cost p.u. sales (no. of contribution cost (Rs.) (Rs.)
p.u. units)
90 42 48 2000 96000 90000 6000
80 42 38 3000 114000 90000 24000
50 42 8 6000 48000 90000 (42000)
[Note: Total Variable cost p.u. = Variable cost p.u. (Rs.7) + Transfer price of intermediate product (Rs.35)]

cè    c 
cc"
ccc& '5c

Expected sales Net profit of division A Net profit of Division B Total Net profit
(Rs.) (Rs.)
2000 (12000) 6000 (6000)
3000 12000 24000 36000
6000 84000 (42000) 42000

(b)c State the selling price which maximizes profits for division B and company as a whole. Comment on why
the latter price is unlikely to be selected by division B.

Solution:

As per the calculation in part (a), selling price p.u. of Rs.80 maximizes profit for division B whereas selling
price p.u. of Rs.50 maximizes profit for the Company as a whole. However, if Division B opts for selling
price p.u. of Rs.50 in order to maximize Company¶s profit, it would suffer a loss of Rs.42000. Therefore,
Division B would not select Selling price p.u. of Rs.50.

c c
0c 
c ( 
c  
+ 
c c "
c 
c c c  ( c ) c   + 
c
c
   c
c
Although we often refer to the goals of a corporation, a corporation does not have goals; it is an artificial being
with no mind or decision-making ability of its own. Corporate goals are determined by the chief executive officer
(CEO) of the corporation, with the advice of other members of senior management, and they are usually ratified
by the board of directors. Incmany corporations, the goals originally set by the founder persist for generations.
Examples are Henry Ford, Ford Motor Company; Alfred P. Sloan, General Motors Corporation; Walt Disney, Walt
Disney Company; George Eastman, Eastman Kodak; and Sam Walton, Wal-Mart.


 c c
Shareholder's value, Earning per share and Market value, all relate to maximizing shareholder's value, which is
not a desirable goal, because what is 'maximum' is difficult to determine. Although optimizing shareholder value
may be one goal, but there are other stakeholders in the business also such as customers, employees,
creditors, community and so on. Again, shareholder value is usually equated with the market value of the
company's stock.
But market value is not an accurate measure of the worth of shareholders' investments. Besides, such value
can be obtained only when the share is traded in the stock exchange.
It is interesting to note that Henry Ford's operating philosophy was 'satisfactory profit', not 'maximum
profit'. He said, "A reasonable profit is right, but not too much. So, it has been my policy to force the price of
the car down as fast as production would permit and give the benefit to the user and laborers, with resulting
surprisingly enormous benefit to ourselves"
Other goals such as adding new products, or product-line or new business actually indicate normal
organizational growth.
 c c
However, every organization has its share of responsibility towards the local community where it is situated,
and the public at large. It is very difficult to incorporate in Management Control System such goals as taking
pride in an organization which cares for the society and renders service to the public. Of course, any concrete
structural programme indicating its operational expenses, methods of providing service, personnel involved in
rendering service and the nature of the service in details can, however, be mentioned through an appropriate
system.

è    c
In a business, profitability is usually the most important goal.

Return on investment can be found by simply dividing profit (i.e., revenues minus expenses) by investment,
but this method does not draw attention to the two principal components: profit margin and investment
turnover.
In the basic form of this equation, "investment" refers to the shareholders' investment, which consists of
proceeds from the issuance of stock, plus retained earnings.

One of management's responsibilities is to arrive at the right balance between the two main sources of fi-
nancing: debt and equity. The shareholders' investment (i.e., equity) is the amount of financing that was not
obtained by debt, that is, by borrowing. For many purposes, the source of financing is not relevant;
"investment" thus means the total of debt capital and equity capital.
"Profitability" refers to profits in the long run, rather than in the current quarter or year. Many current
expenditure (e.g., amounts spent on advertising or research and development) reduce current profits but
increase profits over time.
Some CEOs stress only part of the profitability equation. Jack Welch, former CEO of General Electric
Company, explicitly focused on revenue; he stated that General Electric should not be in any business in which
its sales revenues were not the largest or the second largest of any company in that business. This does not
imply that Welch neglected the other components
of the equation; rather, it suggests that in his mind there was a close correlation between market share and
return on investment.
Other CEOs, however, emphasize revenues for a different reason: For them, company size is a goal. Such a
priority can lead to problems. If expenses are too high, the profit margin will not give shareholders a good
return on their investment. Even if the profit margin is satisfactory, the organization may still not earn a good
return if the investment is too large.

c c
  +
c ( c= c
In the 1980s and 1990s the term shareholder value appeared frequently in the business literature. This concept
is that the appropriate goal of a for-profit corporation is to maximize shareholder value. Although the meaning
of this term was not always clear, it probably refers to the market price of the corporation's stock. We believe,
however, that achieving satisfactory profit is a better way of stating a corporation's goal, for two reasons.
First, "maximizing" implies that there is a way of finding the maximum amount that a company can earn. This
is not the case. In deciding between two courses of action, management usually selects the one it believes will
increase profitability the most. But management rarely, if ever, identifies all the possible alternatives and their
respective effects on profitability. Furthermore, profit maximization requires that marginal costs and a demand
curve be calculated, and managers usually do not know what these are. If maximization were the goal,
managers would spend every working hour (and many sleepless nights) thinking about endless alternatives for
increasing profitability; life is generally considered to be too short to warrant such an effort.
Second, although optimizing shareholder value may be a major goal, it is by no means the only goal for most
organizations. Certainly a business that does not earn a profit at least equal to its cost of capital is not doing its
job; unless it does so, it cannot discharge any other responsibilities. But economic performance is not the sole
responsibility of a business, nor is shareholder value. Most managers want to behave ethically, and most feel
an obligation to other stakeholders in the organization in addition to shareholders.
Example: Henry Ford's operating philosophy was satisfactory profit, not maximum profit. He wrote let me
say right here that I do not believe that we should make such an awful profit on our cars. A reasonable
profit is right, but not too much. So it has been my policy to force the price of the car down as fast as
production would permit, and give the benefits to the users and laborers-with resulting surprisingly
enormous benefits to ourselves.
By rejecting the maximization concept, we do not mean to question the validity of certain obvious principles.
A course of action that decreases expenses without affecting another element, such as market share, is sound.
So is a course of action that increases expenses with a greater than proportional increase in revenues, such as
expanding the advertising budget. So, too, are actions that increase profit with a less than proportional increase
in shareholder investment (or, of course, with no such increase at all), such as purchasing a cost-saving
machine. These principles assume, in all cases, that the course of action is ethical and consistent with the cor-
poration's other goals.
An organization's pursuit of profitability is affected by management's willingness to take risks. The degree of
risk-taking varies with the personalities of individual managers. Nevertheless there is always an upper limit;
some organizations explicitly state that management's primary responsibility is to preserve the company's
assets, with profitability considered a secondary goal. The Asian .financial crisis during 1996-1998 is traceable,
in large part, to the fact that banks in Asia's emerging markets made what appeared to be highly profitable
loans without paying adequate attention to the level of risk involved.

  c* ( c c


Organizations participate in three markets: the capital market, the product market, and the factor market. A
firm raises funds in the capital market, and the public stockholders are therefore an important constituency. The
firm sells its goods and services in the product market, and customers form a key constituency. It competes for
resources such as human capital and raw materials in the factor market and the prime constituencies are the
company's employees and suppliers and the various communities in which the resources and the company's
operations are located.
The firm has a responsibility to all these multiple stakeholders-shareholders, customers, employees,
suppliers, and communities. Ideally, its management control system should identify the goals for each of these
groups and develop scorecards to track performance.
Example: In 2005, the Acer Group, headquartered in Taiwan, was one of the largest computer companies
The Company subscribed to the multiple stakeholder approach and managed its internal operations to
satisfy the needs of several constituencies. To quote Stan 'Shih,-the founder, "The customer is number 1,
the employee is number 2, the shareholder is number 3. I keep this message consistent with all my
colleagues. I even consider the company's banks, suppliers, and others we do business with are our
stakeholders; even society is stakeholder. I do my best to run the company that way."
Lincoln Electric Company is well known for its philosophy that employee satisfaction was more important than
shareholder value. James Lincoln wrote: "The last group to be considered is the stockholders who own stock
because they think it will be more profitable than investing more in any other way. The absentee stockholder is
not' of any value to the customer or to the worker, since he has no knowledge of nor interest in the company
other than greater dividends and advance in the price of his stock." Donald F. Hastings, chairman and chief
executive officer, emphasized that this was still the company's philosophy in 1996.
c c
1c
c
(c   c& c
c  c( 
 c &
c0c cc

c( 5c£

 :c
#  
c<c
 
c
c
£

 c( 5c
Financial Audit is a historically oriented, independent evaluation performed by
internal auditor or external auditor for the purpose of attesting to the fairness, accuracy and reliability of the
financial data, providing protection for the entity's assets; evaluating the adequacy and accomplishment of the
system (internal control) designed,
provide for the aforementioned Fairness and Protection, Financial data, while not being the only source of
evidence, are the primary evidential source. The evaluation is
performed on a planned basis rather than a request".
Institute of Internal Auditor:-
Financial audit takes care of the protective aspect of the business and it does not
normally carry out constructive appraisal function of the business operations. It helps in detection and
prevention of fraud. It also verifies whether documentation and flow of activities arc in conformity with the
internal control system introduced and developed within the organization. It helps coordinating with statutory
auditor to help them in proper discharge of their function. Besides, financial audit also ensures compliance with
statutory laws especially in financial and accounting matters.
# ;  )  c£

 c( '
-To see that established accounting systems and procedures have been complied with
-To see that proper records have been maintained for the fixed assets of the Concern to look into correctness
of the financial data and records along with correctness of the accounting procedure followed.
-To see whether scrap, salvage and surplus materials have been properly accounted for etc.
-To see that internal control system has been working properly.
-To see that any abrupt variation in sales, purchases etc.; with respect to immediate previous year are not due
to any irregularity
-To see that the credit control has been strictly followed.
-To see that all payments have been made with proper authorization and approval. .
-To see that preparation of salary and wage pay roll has been properly done.
budgetary control system, if any scope and performance of internal audit, if any, suggestions for improvements
in performance, if any, and improved inventory policies.
The opinion expressed by the auditors shall be based on verified data, reference to ich shall also be made here
and, if practicable, included after the company has been
forded on opportunity to comment on them.
c

 
c( 
It is a complex task closely related with the process of management. It is highly result oriented. It requires
inter/multi-disciplinary approach as it involves examination, review and appraisal of various policies and actions
of management on the basis of certain norms/standards.

It undertakes comprehensive and critical review of all organizational activities with wider perspective.
It goes beyond conventional audit and audits the efficacy of the management itself.
 
 
'c
It's a comprehensive and constructive examination of an organization, the structure of a company,
institution or branch of government or of any components thereof, such as division or department and its plans,
objectives, its means of operations and its use of human and physical facilities.
c c
.  cèc 
 (c
It's an investigation of a business from the higher level downwards in order to ascertain whether sound
management prevails throughout, thus facilitating the most effective relationship with the outside world and
the most efficient and smooth running internally.
  cD& (c
It is an audit performed with the object of examining the efficacy of the institution/control systems,
management procedures towards the achievement of enterprise goals.
"  c<c" c

It is an objective and independent appraisal of the effectiveness of managers and the effectiveness of the
corporate structure in the achievement of company objectives and policies. Its aim is to identify existing
and potential management weaknesses within an organization and to recommend ways to rectify these
weaknesses.
"  (c
  cc
 
c

c
(
c

Thus it can be seen that management audit is an examination, review and appraisal of the various policies
and actions of the management. It is a tool for the evaluation of methods and performance in all the areas
of the enterprise.
# ;  ) 'c

1. To ascertain the provision of proper control at different levels, their effectiveness I in accomplishing
management goals.

2. Ascertain objectives of the organization are properly communicated and understood at all levels.
3. To reveal defects or irregularities in any of the elements examined and to indicate what improvements
are possible to obtain the best results of the operations of the company.
4. To assist the management to achieve the most efficient administration of its operations.
5. To suggest to the management the ways and means to achieve the objectives if the management of the
organization itself lacks the knowledge of efficient management.
6. It aims to achieve the efficiency of management and assess the strength and weaknesses of the organization
structure, its management team and its corporate culture.
7. To ascertain the provision of proper control at different levels, their effectiveness in
accomplishing management goals.

8. Ascertain objectives of the organization are properly communicated and understood at all levels.
9. To reveal defects or irregularities in any of the elements examined and to indicate what improvements are
possible to obtain the best results of the operations of the company.
10. To assist the management to achieve the most efficient administration of its operations.
11. To suggest to the management the ways and means to achieve the objectives if the
management of the organization itself lacks the knowledge of efficient management.
12. It aims to achieve the efficiency of management and assess the strength and weaknesses of the
organization structure, its management team and its corporate culture.
13. To help the management at all levels in the effective and efficient discharge of their duties and
responsibilities.
The auditor must apprise managerial performance at all levels of the organization. The audit starts right at
the top level of the management. It studies the managerial performance at all the levels of management. The
audit has to study the decision-making system of the organization and also the level of autonomy granted to
the managers at different levels of the organization. The authority and responsibility given at the different levels
of the management. One of the most important things that the audit must study is that the mangers at various
levels use the authority.

c c
"
(
c
 
c( c
Management audit requires an interdisciplinary approach since it involves a review of all aspects of
management functions. It has to be conducted by a team of experts because this requires 3 varieties of skills,
which one individual may not possess.

The team may consist of management experts, accountants, and the operation research specialists, the
industry experts and even social scientists.

The auditors must have analytical mind and ability to look at a management function form the point of view
of the organization as a whole. They therefore have to be properly trained in this aspect. They need to have
through knowledge of the management science and they should be acquainted with the salient features of
various functional areas.
Under financial audit, the entire emphasis is on macro-aspect, the individual transactions being- scrutinized for
check of the aggregates. It is concerned with examination of transactions recorded in the books of account. It
reviews the procedure and internal checks, and scrutinizes individual transactions for the purpose of
verification, of Profit and Loss Account and Balance Sheet. Financial audit is not concerned with ~ avoidance of
profiteering motive. It indicates the financial position and over~ performance of the business, regardless of its
performance in various segments. Financial audit is applicable to all classes of companies and industries
irrespective of size and Dan of operations.
Instead of serving the interest of the management and the Government, it serves interest of shareholders.
Financial audit is organization - oriented. It is conducted under Sections 224 - 232 of the Companies Act 1956.

cccccccccccccccccc£

 c(  
 
cc( 
It is concerned with the review of the past
It is concerned with financial aspects of
Performance to ascertain whether it is in tune
business transactions of the year under with the objectives, policies and procedures of the
enterprise.
audit
The auditor examines the past financial The management auditor reports on performance
of the management during a particular period and
records to report his opinion on the truth
suggest ways to remedy the deficiencies,
and fairness of the representations made in the including modification of objectives, policies etc.
financial statements. Examination of the
performance of the management is
beyond his scope

Past year '(Financial) transactions are No limit as to the period to be covered


Covered Enterprises such as companies, trust and
societies etc.

There is legal compulsion as regards


Financial audit is compulsory in the case of certain management audit.
enterprises such as companies, trust and societies
etc.
The auditor reports to the owner, i.e. The auditor reports to the management
shareholders in the Case of a company

c  c
2c
c c) c cc
 
c
 c  c 
c
c  cc c
Management control process involves communication of information to the managers at various levels of
hierarchy and their interactions arising out of them. These communications aim towards attaining the
organization's goals. But individual managers have their personal goals also. For example, a young manager
with good education, experience, personality and social background joins a company like Britannia Industries or
Reliance. The company finds him fit for the position as per job specifications, appoints him and makes him
aware of what the company expects of him. The young manager sets his goals of gaining rich experience for his
career progress besides adequate compensation packages. Naturally, his actions will be directed towards
achieving his own objectives and goals while serving the company. Thus, his self-interest and the best interest
of the organization are apparently in conflict. But the best results can be achieved by perfectly matching the
two interests and this is called 'goal congruence'.

It is quite apparent that perfect congruence between the goals of the individual and the organization individual's
goals and the organization's goals can never happen. Yet, the main purpose of a management control system is
to assure goal congruence between the interest of the individual and the organization as far as practicable.
c

 
c
 c c
£ c
(c
 c"
 
c
As mentioned earlier, all the communication of information may be either formal or informal. The formal
communication system involves strategic plan, budgets, standards and reports whereas the informal
communication is made through letters and memos, verbally or even by facial expression.
Formal communicationsc are all documented and addressed to the responsible managers for their information
and actions, if necessary. However, the actions depend on the perception of the individual managers.
Informal communication c on the other hand, relates to some external factors-work ethics, management style
and culture. Added to these factors is the existence of an informal organization within the structured formal
organization.
Informality refers to the relaxation of sharp differentiation and explicit description of behavior as indicated
in the hierarchy and thereby, moving away from superior/subordinate relationship. However, such relations
depend on the personal capabilities of the manager such as education, experience, expertise, trust and
cooperation. For example, Accounts Manager of Nasik Plant (see the organization chart in the diagram 3.2)
reports to the General Manager of the Plant. While visiting the Corporate Office for attending a Training Course,
he meets other colleagues, parallel officers and even the Finance Director. The latter communicates some
important matter to him verbally and wants action thereon. Accounts Manager carried out the instructions so
given. As per the organization chart, he should inform his General Manager, but it depends on his own
perception of the situation, and he mayor may not report to the General Manager.

. *c :c
 
c c
(c" c
External factors like work ethics vary from place to place. Therefore, organization work culture depends on the
general behavior of the people in the society where the organization situates. Work culture generally differs
because of the life style and the attitude towards the work. For example, people of Mumbai lead very fast life.
Time has more value at Mumbai as compared to Kolkata, where people take things easily and leisurely.
Japanese and Korean people have reputation for their excellent work culture.
However, the most important internal factor is the organization's culture and climate. The culture refers to the
set of common beliefs, attitudes, norms, relationships and assumptions that are explicitly or implicitly accepted
and evidenced throughout the organization. The writer joined Union Carbide as an Assistant just three days
before Christmas Eve. On the very second day, when he attended Christmas lunch, his table was shared by
none other than the General Sales Manager Dr. W.R. Correa. He kept us amused with various stories of his
recent tour abroad and recited Urdu 'shairies', even sharing jokes. Such a situation was unthinkable in Jessop &
Co., where sharp differences were maintained at every level of hierarchy.

 
c
 c 
Climate is used to designate the quality of the internal environment that conditions the quality of cooperation,
the development of individuals, the extent of members' dedication or commitment to organizational purpose
and the efficiency with which that purpose is translated into results. Climate is the atmosphere in which
individuals work help, judge, and reward, constrain and find out about each other. It influences moral-the
attitude of the individual towards his/her work and environment.

c c
Culture differs between the organizations, but cultural norms are extremely important. They are not written
like formal communication. But the existence of a good culture can be felt from the behavior of the members of
the organization. Once the writer landed up with his family at Hyderabad in the early morning to discover that
nobody had come to receive them at the station. His visit was arranged through non other than the Director of
the company himself. His unit being new, telephone directory did not include any number of his unit, but the
parent organization's telephone number was located. When an executive of the parent company was contacted,
he immediately sent an officer of the company with a car to pick us up to their Guest House, entertain with
coffee and then put up in a Hotel. What subsequently happened is a different matter, but the attitude and
treatment of that member of organization speak volumes about their excellent culture.
In any organization, the culture remains unchanged as long as the Chief Executive remains in position.
When a new executive replaces him, there is likelihood of some change in the culture, unless the new Chief
follows the footsteps of his predecessor and maintains it. Generally, if higher positions are filled in through
promotion of internal executives, the culture remains unchanged and the traditions are maintained.
The other important internal factor which influences management control system is management style-that
is the attitude of the superior to his subordinates and the latter's reaction through their perception of the
attitude of their superiors. Again, the attitude ultimately stems from the temperament of the Chief Executive,
who controls the entire organization. That is why R. W. Emerson said "an institute is the lengthened shadow of
a man".
 
 cc
 c"
 

An organization indulges in informal control process when encountering non-routine decision-making or when
seeking new information to increase understanding of some problem areas. During a very critical period in an
organization, the writer found that the Chief Executive used to call managers informally at his residence or club
to extract information in a relaxed manner rather than in a tense situation prevailing in the factory.

£ c"
 cè  c
c
Formal communication system is structured as per the 'hierarchy outlined in the organization chart. The system
has the following four components:
(a) Strategic plan and programme
(b) Budgeting
(c) Operations and measurement in responsibility centers (d) Reporting

(ac   cè
c
(cè  
The foundation of management control process lies in the organization's goals and its strategies for attaining
these goals. A strategic plan is prepared in order to implement the strategies, after carefully considering
opportunities and threats in the external environment as well as the strengths and weaknesses in the internal
environment. Thus, a strategic plan and programme is prepared as a guideline to budgeting.
4 c3( 
c
The strategic plan is converted to an annual budget incorporating planned expenditure and revenues for
individual responsibility centers. Expenses and revenues are marked for each responsibility centre period wise,
say monthly, quarterly, half yearly, and annually.
4c#  
c
(c  
c
Responsibility centers operate within the framework of the budget, established standards, standing instructions,
practices and operating procedures embodied in 'rules', and 'manuals'. Thus, besides budget, the responsibility
centers are also guided by a large number of rules. They record the resources actually used and revenue
earned. They also classify the data by programmes as well as by responsibility centers for performance
measurement.
4(c$  
c
Actual performance is analyzed, measured and reported against plan, indicating variances and highlighting
areas of weaknesses. If the performance is satisfactory, feedback information is sent to the responsibility centre
concerned for praise or reward. If the same is unsatisfactory feedback communication is sent to the
responsibility centre concerned for corrective action. If such action requires to be included in the budget, then
the latter is revised to give effect to the changed position. If required, then the plan itself can be revised and a
new basis of control may be established.

c c
The aforesaid formal control process has been presented in the following diagram:

= c=cccc=
c  c


cc

c
   c
c=c= c


ccc

c c c 
c
c cc
c
c ccc 
cc 
c  cc
c c 
c  


c c 

c
c c c c c c  
c 
c c 

c
ccc
 ccc

cc  cccc 
c c
c  c


ccc
c cc
c cc ccc
c ccccc
c


cc ccc!
 c


c
cc"
c 
cc
c
 cc
c #


cc

c
ccc 
cc
Oc cc
c
Oc 
cc
c


cccc
 c
Oc 

ccc


 c
c

c !c
cc

ßc
c
c cc
c c#
ccßc
c"
c$c
c
c cc


ccc
 
 c

c c c
% c &
c 
c 
c c
c
 
c 'c 
c c c 
c


cccccccc 
c ccc
( c )cc c 


c
c$ccccccc cc
c 


 cc
* c )c c  

c c c

c c )c c  c 


c  c 
c


c
c 

c c 
+
c  c c 
c c
c c $c
 c c
 c 

cc

cc$c 

 c
, c  
cc c
c c 
ccccc ccc

c

c$
c c 
c c
 cc c
c

c 

c$c

c
cc 


c c
- c c.
ccccc
c
c
cccc
cc  
c

c 
 c /c c c
c c c c  


c c c "c c c

c c

c c
c
! 
ccc
c0cc 
cc ccc
cc
cc
c


ccc1"cc
c cc
cccc+ccc
cc"cccc cc  c c
c


 "c=  " c c
ccccccc
ccc cccccc
c

 cc 
cc

c cc$
c
cc c c


ccccc
cc c c

Èc #c$ccccc"cc*2c3c
 c
c
ccc"c cccc c

ccc 
cc
c$"c c)cc4 ß
cc5

ccc
 c
cc
c6
c

cc cc6
ccc$"ccccccc cc
c  cc
c
c$c
c(cc*cc"cc*cccccccccc
c
c
c$"cc
cccccc 
c
c6
c
c cc$" c
Èc ! #c#cc&
c
c
c
cc
 c

cccc-2c ccc
c
 
c

ccc  
 cc

cc"ccc
cccc
c c c c c c   
c $
c c
c  c 
c c c
 c
"$c c cc   
 c )cc c
c c 6
c c

cc c 
c c 4 c
ß
cc c"c
c6
c c$ccccc c$
c
cc cc%223c

ccccc7c"$c$c6
c c
 c

cc

cc
c


c$c$cccc cc 
c
ccccc
$c
cc18 c89c 

cc1:
c!9ccc"c 

cccc
c
c 
c
c
c 
c$
c$cccc c

c cc
cc
c6
ccccc
c
ccccccc$cccc
 cccc"c
cc  
c
c
cc
cccc
 
cc6
cc

c c
cc c
c6
c
c 
ccc
0
cccc
c  
cccc
c c c c c c 
c c c
c c c c c c
c c
c c c c 

 c c c

cccc
ccc"$cc
cccccc
cc4 c"
ccc
cc ccccc

ccc cccccc 
 cccc ccc4 cß
cc1cccc
$c$cccc
c
c


 cc$c"$c
c
c9c

Üc c #!cc#%%c!c& cc'# c&#$c!ccc%(c

c c 
c c c  c c c c  c c 
c 
c c c 

 c c
 
cc c
c


 cc

c
ccc c
c
c
ccc
c;cc
c c
7c

c c
ccc
cc
cc$ccc
cc
ccc c$
c
c
c c c
cc
c
c
cc
+
 cc

)
c

 c
c cccc ccc  cc c
c cc

&c
c
cccc$
cc


c


ccccc 
cc
c
ccccc c
$
ccc c
cc$cccc
c   c

§c cccc$
c c cccc c
ccc
c
c
c
ccccc
§c c
c "cc 0

ccc c7cc$
cccc

 c
§c 

 cc 
ccc$c
c 

cc$
cc ccc


c c
c5c
c
ccc$cc
cc 
cccc c
§c 

 c
cc
cccc
ccccccc
cc
cccccc

cc
 
cccccc cc
c
c cccc

c
cccc

cc
cccc
 c
ccccc

c
c
c$"c
c

c c
§c  
c
c
ccc

c$cc


c ccc
c5 
ccc
c
5 
c 
c c c c c c  
c c 
 c c c  
c 
c c 
c c
5 
ccccc


cccc c 
c
cccc 
 c
§c ccccccccc
cc$"cccc  cc
cccc
 c
cc
c
cc
ccc
cc+0c
 ccccc

c
cccc
c
ccc$"ccc cc
cc
 
 c
c
§c &c
cccc

cc
cc
c 
+ccc
ccc$cccc5 c
cc
c
ccc 
+cc
0
cccccc$ccccc
ccc"$ccc$"cccc
 c$
c
c
c
cccc$
c$
cc

 c
§c ccc ccccc$ccccccc"$cccc 

ccc
c
cccc ccc cc
c7cc"$cccc
 cc  c

c


cc
c ccc
c
c
c
c
ccc "c


c$
cc
ccc


ð  ð  
! %c

cc 
cc
ccc
 
c 
cc
cc cc"
ccc
$
c
cc 
cc
c  cc&c%<=,cc$cc,22cc c
ccc
c

c c
 cc cc
cccc>c,<,%c

ccc
 c$c>c(*?c

 cc
cc$cc(=(222 c

c
c%<-2cc 
c


cc 
c 
+
cc
cc


cc
c
c  ccc
c
cc
+
c c c ccc c 
+cc
c
c
 
+
c$cccc 
c
c$ccc
cccc  cc&cc
 
+ccc
ccc c$c6
cccc  cccc
cc
c
ccc5 cc cc
c
+
c
cc cc
c5  ccc

c" $c )%cc

cc5 c$c
c
c%<-( cc#


cc5 c$c
cc 
c
c
cc c 
c
c


 cccc
+
c
c


c cc
c
$cccc cc
c5  cc

ccc
cc5 c$ccccc

cc$
c5 c$ccc c
!
c c

c$ccc$0c

% c 4c $c c c 


c c c  c c 
+
c c c
c c
( c 4c $c c
c c
 c c  c c c c c c
  cc!c c#cc#cc
c
cc
 cc c ccc
 cccc
ccc
c
 c
cc%(3ccccc(-3cc
c cc
* c 4c$ccc
cc
cc5c
c
cc cccc
5 c
, c 4c $c c 
c c $
c c c  c c $c c c  c
c
cc

c
 cc c$ccccc c
- c 4c$ccc cccc

c  
c 
ccc
c c 
 c

c
c c
c'##c$ $c)%c*c!'!!ccc$)c &)%+c

% c /
ccccc c c
( c ! 
ccc$"cc

c 
c"
cc
 cccc

c
ccc cc
+
 c
* c 4cc$"cc
cc c
c
+
c

cc
c
 c

c
cc
c5 cc
cc$cccc$cc$
c6
cc

cc

ccc$
cccc
c
c
c
cccccc
 c6
ccc cc
c 
ccc cc
@cc

&c"
cc$cc
c6
c
+
cc c
cccccc
c"
cc$"c
cc c 
c
c$
ccc

ccc 
c5 
cc$cc

c
 cc
$cc
cccccM ,c #c-.c

/.0ccc c!%&!1c*c #!c&#,c&c!!(c c!c #!c&c%%c cccc


2 'c$ c%c&c% %!c$c(c

*+c &#,c
ccc
ccc cccc c
cc
c
 c
 c.
c$
cc ccc
 cccc
 c
cc
Acccccc
 c
cccc
cc c c


cccc

c cc cc
ccc#c/c&c
6

cc$c

ccc
c ccc
cc


+cccc"cc
c"
c


ccc
cc
cc ccc
c 

c
cccc c
6c"c c

$c'&#c%# !ccc& cc 


cc
cc. c c
0cB
c0cc c c"ccc 


cc
ccc  c
0c)
c0c ccc
 cc
c
cccc
cccc cc

=%c c"!#cccccccc
Ac&#,;ccc6
c
c  cc

ccc
c

cccccc
ccccc$
c
cc
c

ccc

c cc

 cc2 ,c0ccccc
Ac


;c
c 
c
cc
c
c
c ccc cccc
c
c
cc
cc
c$ cc

c 'c0cc
ccccc


ccc
;ccc
cc
c
cc

ccc cc

%#c 'ccccc cc$


cc
ccc
c
cc 
c ;cc
cc cc
cccc
cc
 cBc
c  cc


cccc6
 cc

.


c#c0c"
cc
cccccc


cc c c
cc
cc c"c
cc c"
c c c

3'c
c$cc

c$cc cccc
ccccc&#,cc

cccc cc

!

c


c cc"c
ccc

c 
c
ccc
cc
cc
c

c
  c!

c
c
c
cc
c cc
c

c
7c"ccc
 c

,c #cccc##*+c

0.c &#,c
4.c "3cc
5.c ! %',c
6.c ! %c#!c
7.c #c!'#$c
8.c $#,c !c
9.c &#%c&#,c
:.c #%c&*c+c!c#+c#.c

c c
&#,c

c"c
 cccc 
ccc


c$c1cc
c
 9 cc#
c
 c$c
c
cc
cc ccc 
c  ccc 
c c$c 
c cccc
cc;c

c ccc
c
c  ccc 

ccc
ccc
 ccc



c$c cc$0c

% c c
 cc 
+c 

cc 
c
cc
 c
( c c
 cc 
+c$cc$"ccc 
cc
 c
* c c
 c c 
+c c 1c  c c
9c Cc  c c 
c $
cc  
7c c
c
+
c5 
 Dc
, c c
 cccc"c
c

cc


cc
c 7cc
 c

&c cc 
cc
cccc


cc
+
c 
ccc

c
 c
 
ccccc
c
cc
cccc
cc
ccc ccc

3ccccc!%c*c!ccc*,+c


cccc
 ccc
c 
c
c

c 
+ccc"c

c
cc
cc
cc
ccc$
ccc
c cc
c
c
ccc
 ccc

 cc
cc
cc$"ccccccc
c
 c cc c
cc cc
c
$"c
c cc c  
cc
cc
cccc
c
cc
 cc
)
c$
ccc
ccc
c$
ccc"c

ccc c

c$
ccc
c

7c c
c$
c 

ccc
c
 cc
ccc
 
cc$cc 
c c

cc
 
ccc
c
cc$ c

/.4ccc,c!c%ccc&#,c!!cc!c$ $c,$(c ;c$%ccc


%c#%!cc&#,c$ cccc#,c% %!(c

*+c

c%cc3,c #cc*%c!!cc!c$ $c,$ccc##*+c

"3c c

. c
c
c"ccc$cc
ccccc"c
c
c
cc
 ccc
c$c cc cc
c
c
c" ccc

c
c5c 

c
c

c"c

cc
c

cc$c 
cc c
cc
$c 
c"cc c c
cc
 ccc c
ccc
cc
cc c
c  c
$
ccccc
c$cc
 cc
c


c
cc$c

c
cc c
cccc
c 


c cc  "cc
ccc$c 


c ccc  "c"
 c  "ccc  " cc
. c
ccccc
c c cc
 c
cc c
 cc c c
cc
 cc c
ccc c


 c

ccc$
c

c
c

cc$c 
c"cc
c c
 ccc
c
c
ccc
 cc
c
c ccc


c c
cc$0c

% c c"c
 ccc
 c 
c ccc

 c 
c cC 
c
cc 
cc 
cccccccEc 
cc
c Dc
( c c"c
 cccc
c 
cccc
ccc

 c
* c c"c
 c
c
c
ccc 
c
c
c" c
, c c"c
 c cc 
cc c

&c

ccc"c

ccccc c
cc
c$c 
c c
c
c$c 
c" cc.cc
cc
c
cc
c
c
c
c
"
c$cc
ccc$c
 cc
c cc
cc


ccc cc 

 c

c c
! %',c

 cc 

c
c
cc

cccccc
cc c c
c
c 
 c!c

c ccc$c
ccc 
cc"cc 

c
ccccccc
ccc
 cc&c
c
cc
ccc c 

cc ccc ccccc
 cc$c 
c
$c
0c

% c )c
c c c c c  
+c c 
c c $c c c 
c c
c
c
 

 cc
( c 

cccc
c 
cc
cc
 c

/cc
cc 

c#
cc 
c
ccc
 cc/c c
c$cccc
 c

/c&c
cc 
cccc

c c.cc$c
 c
c
ccc
ccccc
c cc
 c
cc"
c ccc c$c5cc c
c
cc
cc c
c
c
c$ccc cc c
c 

 c

! %c !c

&c
c
cc1

cc
ccc
c
c
c


cc
cc c


cc 
c
"$c
c

c 
c"
c
cccc$c ccc
c
c6
cc
cc

c  cc. c$c
cc

c

c cc c cc

% c $c
c c c c$
c 

cc$
c c@c
( c c$

c c$c c  cc$
c c$c@c


c c$ccc6

ccc6

c 

 cc&c$ccc cccc


c"
c 
c ccc

#c'#$c

&c
cc1
c

cccc

cc
cccccc cc
ccc



c$cc
ccc

c c$cc 
9 cc4ccc 
c
c
ccc
 
c1c
c c
cc 
cc cc

c 
c
"
cc
 cccc
cc
cc
cc

cc
c ccc
c
cc
c 
c$cc
ccc
c

ccc
cc 
cc
cc


cc c

 ccc

cc$c
c
ccc c
c
c"cc0c

%Dc c 


c c c 
c c c  
6c 
c c c  c c c
cc
cc$c
 c
(Dc c
c$c"cc
ccc
ccc$cc
c6


cc"c

cc
ccc
c$

c c
*Dc  
c$
c$
cc  c
cccc$c c

!
cc 
cc5c
c6 cc$
cc
cc 0c

% $c+c$ccc5c$c
cc
c 
cc 
ccc @c

cc<&cc>c cc$c
cc

c 
cc

c

cc"$cc
cc ccc5c
ccc c

ccc$cccc5cc$
@c

'*c!c #c>c ccc



cc

c
$
c cccc
 
c$
cc


c

cc
c$"cc

ccc 
ccc


c
c

c@c

#%$+c c$c
ccccc 
c$c c
c5c c$
c
c
cc


cc
c

c$cccc
c ccc$c
c@c

c c
=%!cc$c 
c$
c
cc
cccc

c  ccc c$c

 c

cc

c

c "cc cc
c
 cc&

ccc$c

c
c
c 
cc
c ccc
ccc$c
cccccc
c
c
ccc
cccccM$&#-.ccc cc$c
c
c$
cc
c cc
c  ccc  
cc c c
$c  c
ccc
cc$c+cc

c
ccc$c c$c 
ccccD&
ccDc  c Dc

 c

$#,c !c

!ccc
ccc c
c
c"cccc
cc
cc1c
+c

cc
$c$c5 cccc$
c
ccc
 cc
9 cc&cc$c
c c
ccc
 cc!
c
ccc

c

c cccc
ccc

 ccc
c 
  cc  c c
cc

ccccc
6

 c

='#c%$c*c%!ccc%cc

%Dc c

c

c
c
ccc

cc
c  c
(Dc &cc

ccccccccc c
*Dc ) cc

c$c

c
c$
cc 
cccc


+c

c
 ccc c


cc
ccc 
 c
,Dc c c
c 
c c c

c 
c c c c  c  c c  c c c
ccccc 
cc c

&#%c &#,c
c"cccccc 
7c 

cc
c

cc  c
ccc


+c$

c
c 
c
c$
c$cc
c
c
c   cc#


cc 
c
c c
c$c 
c


cccc cccccc c
$
cc
cc 
c"cc cc ccc
c
 cc$c 7c
cc
c c

+ ccc

c cccc$c cccc


ccc
cc


ccc

cc
cc  ccc 


cccc$
c cc
c 
 c

cc
ccccc
c 
 c

/cccc 
ccc 
c
c
c 7c
cc
c


 cc
c cc

c

cc c
c$c
cc c
cc"c

c
c$cc

 cc!c

c
c$
cc$c  c

=*cccccc 
cc "c$ccc1c 9ccc
c
c c
c$c
c


cc
cc
c1c
c
9cc$9c
c
c  c

?!1c#1c!c& c


cc$c
ccc$cccc

cc
c
c
cc7c 
c
c
 
c
c
cc c$
c 
ccc$ccc cc8cc

c$c
"$
cc
c

$cc
cc
cc$cc
c

c$c
c
c

 c

#c$$ ,c


c 
cc$cccc
cc


c
 cc 
cc ccc


c
$ccc 
 ccc 
c
 cc

cc$ccccccccc
c
cccc cc 
c$ccccc

c 
ccc c
c c 

cc c 

cc


c
c  c
c
+
 c

#%c&*c=+c c!c + c#


c c$cccccc"ccc
ccc

+c
 cc0c
cc$cc  cc cccc cccc c

ccc
 c c-c%2c%-cc
cc
 cccc c
c c$cc
 cc c
c"ccc$c
c cc
cccc0cc0c

 ccc

cc c$c0cc0c

cccc

c


cc"ccc
0ccc c$cc 

cc0c
 cc.ccc 
c
c cc$ccc

c
cccc0cc0c


 c

c c
#c $#$c

c 7c
c
c
cc 
cc
 ccc c"ccc
cEc

%Dc #
$
c c
cc cc
(Dc 
ccc cc
*Dc .
cc 
cc
,Dc .

c
cccc 
 c


 c
cc"ccccc


c"c

c 

c c
cccc
c
c
c
cc$cccc
c
c 
c
c
c cc)

cc
c
 
 c
c
c"ccccccc cccc 
ccc
 cc 
c
ccc 

c
c c

ð c
F&c&c $c c 
c c  c c $c c >,G2c c Fc 
c :c )cc c)
 c
.c Ac c 8


c F&Ac  
c c c c c c c c c c c

c
c
ccc
c0 c6
 c&Acc
cc$"cc
cc6
ccc
c
c
c&ccc$cc c
c5cc>,*2cc>,-2cc
ccc>,G2c&Acc
c$c
c
c&Acc$"
cccc
c cc


c Acc$c$cc
cc
c
c
ccAc
c cc
cccc ccc
ccc
 Fc

)
 c.cc$cc
0
+cc
cc c
c$
cc"ccc
 c c 
c c
c c c $c c
c c  c c c c 8


c
$
c c 
c c c c 
c  c c c   c & 
c c c c c c

c


ccc
c


c$
c
cccc Acc6
 c

!ccc c


ccc5c
ccc
cc
c
cccc
 c
c c c c $"
c c 
c  
c c c c 
c c  
+
c 


c c

ccc

c cc
ccc c
 cc Acc

c
cc

ccc$ccc  cc 


+
ccc
c cccc Ac
c
c 

c

c

cc
 c

cHc8


cc
cc 
c
c cccc
cc
c 5 
c$
ccc
8


c $
c $c 6
c c "c c  c Ac c c  "c 
c c c c
cc 
cc
c 
ccc
ccc c) cccc c
c c c $c c c  c  
c c c c $c c $c 


c c c
8


c$c
cccHc8


ccc cc
c
ccc$" c

c c c 




c $c c c Hc 8


c "c c 
c c c  c c c c
8


ccc$c
c 
 c c


cc$cccccc$c
c
c$
ccccc$

cc c


c$c  cccc
c"c
c
cc
$cc
 c

8
c 
c 
c c 
c 
c c  c c c c c 8


c $c 
c 6+c
c c 
c c c 

c c c
c c c
c  
c $c c 
c c c
c
c
c   c c
c cc c
c
cc c )
 c 


c c cAc c
$ccc
c  c&ccccccHc
cc$cc
c
cc

c
ccc c8


cc)
  cc$ccc c c 
cc
cc

c
cc
c$

ccc 
c
 c
c  c
cc ?2c  c c Ac 00 "c c >,22c c c c c c c c 
c c

c
 c c c c c 
c $c 
c c c  c
c
c 6c c
"c
c$
ccc
c$"cccccc c&c00 "c ccc
c
c 
c
c$cc=2c ccc
c
 c

cHc8


c 
c
ccc cc>,G2cccccc8


c>,*2cc
cc c.ccc>,*(cccc
c.cB c
c.ccccc
)
 c c 
c 
c $
c c 
c 

c c c
c c $c c
c $c

c 
c c

c
 cc 
c 
c $c c 
c c c c 8


c c c 
c 6
c c><2c c
c  c c
c $c c 
c c >*2c c c c c c 8


 c /c c >*2c c >(-c
$cc00 "c  c


 c 
c 

c c c c c 
c c

c ßc c c c Hc 8


c

 cc$
c
  cc
c$
c)
 Ac 
c
c
 cccc
c
cF cc

ccc 

c"c$c
c c ccc c$c c$cc  cc$cc c

cccc
c
c$ccccc
cccc%2c ccc
c"@Fc

ß$
cc4 c)cc 
c
ccc$ccccccccc
8


cc 
c
ccccc
c
cc4 c)c$cccc(2c cc
c 
c c c 
c 00 "c  c c c $c "c c 
c 4 c )Ac $c $c c
ccccc

ccc  cc$ccccc
cccc$"c
c c  c c 
c  
c c 
c cc c c 
c c c c"c cc  
c c c  c

 c

c
c
c ccc c ccc
c


 cccc c
c c c c c c c 
c c $"c c c c  c c c $
c $c 
c c c



c ccc
ccc ccc$ cc"$cc
cc cc

cccc
c 4 c ßc $c  c c $c 
c $
c $c c c c c .c c c >,*2 c
$c
c$cc
ccccccc  ccc
c
cc

c$c
 c  
 c c c c c c c c  
c
c 6
c $c c c -c  c c c
ccccc


c
cc 
c$c  
c
c

cc c

c c
c

Ê c %c&!c #!cc'c%%ccccc&ccc%c c$,(c


c
=#c 
ccc
ccc c

 c   c
" c

c .c%222c c

= c   c
c/cc "ccc)
 c c @65Ac

 c   1c cc c >,*(c

c Bccc c
cC><2I,23Dc >*=c

c c c
cC>*20>(-Dc cccc-c cc>,%c

c c /cc "c cc)


 c c c c @5B0c


" =
c c c c c c c c >,G2c

c Bc c
cC>(G2I,23Dc c c >%%(c

c c c
cC>,G20>,22Dc c c ccccG2c >%<(c

c c /cc "c cc)


 c c c c @4::c

c$c
cc  
ccHcc cc
ccc


cc
ccc
$c c
ccc
c$

cc c$c  cc  c& 
cc$c c$cc
c)
 c.ccccc
c
c
 c

c
c c
D c = #!c".c c%%cc&!(c,cc*,c(c
c

=#cc
cc
c
c4 cßccc
cc cc
cc
c
cHc



Ac$c
 ccc
c
c
c
cc
ccc$
c
c

cc
cc
c

 
 c&cc
ccccc
 c
ccc

c c$
c
cc


cc
ccc
cc
 c

4 cßccccHc


ccc cc
ccc


 ccc

c c c 


c c .c c c 
c .c c c >,G2c >,*2c c >,*(c
 
 cc 
cc
ccc


c$cc 
cc
c
ccccc
C
c D c 
c $c c
c c $c 
 c c c Hc 


c
c c c c
c
c
c
c5c
ccc
cc
c
cccc
 cc4 cßccc c
c
ccc


 c

&c
c
c c c 0c  
c
c c c Hc 8


c c  c c 
c c c .c
cc
c
c$
c
cHc8


c
c
c
 ccccc
c
cc)
 c
.c
cccc"c
cC5ccc
c
c

‰ c = #!cc'%c!cc%c c$,c3c,c%(c


c

=#cccc
cc
cc
c$cc
c c
c
c
cc
c
c c cc
cc
c cc
c

cc 
cc
c
c c c
ccc
c6
cc
c

cc

 cc"c
cccc
c ccccc
cc c

 c
&c
c cc


ccc
c$c 
cc c


ccc c
 c
cc
cc>,G2cccc


ccccc
c cc
c

c
c c
c c c "cc c  c 
 c c c c 
c c >,G2c
c 
c c c "c

c$
c
cc>,*2 c8

cccc
c$
cc
 cc  ccc

c
cc

 c


c c

c c c 
c 
c c )
 c .c c c Hc 


c $c c
c  c c $c 
c c c .c  c 
c 
c c
c c 
c 
c c Hc



cc
c
cc
ccc
cc)
 c.c cc 
cc
ccc


c$c
c c cc $c c 


 c c c c $c c c cc c c c c c
Hc


c$
c
c>-2cC>,G20>,*2D c

c 
c 
c c 
c 

c c c Hc 


c c  c c 
c c c



 c $c c c  
c
c 
c c c c c -3c c c c c c c c 


c

c
c
ccc
ccc
c
cc
cc 
c$c

cc
 c

c c
  c c c %',c !%&!1c*1cc c ##1c c c c %c ,$c !, %#(c c c
&#$c%##cc$c%1cc%1cccc%c#%,ccc'##c$(c c1c
*c%%c%c!c, c (c
c

=#cc&c$cc
cc


cc"c
cc


ccc
 c$ccc c
0c c 
c
 c 0c c 
c c c c c $c c "c 
c
c c

 c c c $
c )
 Ac c 

c c
c c c $c  c 


c c c 
c $c 
c
c$
c
c

c$
cc Ac
cc 
+c


cc
 c c c


c c
c 
c $c 
c 
c c 
c c  c c c 6c 
c c  c 


c c "c


ccc
c
c$c


cccc ccc$ c

!
c$c"cccc
ccc6 c&c
cc>-2cccc"c
 c
c
$cc
c
c
cc 

c cc
c
c$c 
ccc
c6cc
c
$
c
c
ccc"c
 ccc
c$cc
ccc>,G2cc$
ccccc
cc
ccc
ccc
cccc
cc
cc ccHc)cc
c c c 
c c c c" c 
c
c
 
c $
c c   
c c c 


ccc c
"c
c
cc$cc
 c4"c
ccccc
c c$c$c
cc


c
cc
cc 
 c

c c c c 


c c c 
c c c
 c
c c 
c 
c c $c c c c

c c cc
c
cAc
c$c  ccHc
ccc$

cc c

c$c
cc

+c c
 c cc
c$

cc$cc"c
c
c cc
c$0c

cc  c
cc
 cc
c
ccccHccc
c
cccc
c
cc
c cc
ccccc
 c

&c 
c $c c $
c c c 
c 5
c c 
c 
c c c Hc  c c c

cc c 
 c)cc c
cccc
 c$
cH c

&c$c
cc c6cc"c
ccccccc
cc
 c 
 c
c$
ccccccc"c
c ccccc

ccc
"cc
c5c
ccccc cc ccc"c
ccc cc
c$
cccccccc
ccc
c ccc$
c$cc"c
c c

c
ccccc
cc ccc c


c
c5ccc
c
c c c
 c 
c c c 


c c 0
+c c 
c c  c c

   c c




c
cc 

c0c
cccc"cc
cc Ac c&c
c

cc 
cc
c
c
cc
cc

+cc Ac
ccc$ c
c$
c cc c
0c
 c

 cc ccc5cc c




cccc

c
c
"c
cccc

 cc ccccc cc0



c
c
"c6
cccc
cc



Ac
cc
 cc&c

cc cc$cc


c  cc
c$c
cccccc ccc 

ccc5c "
cc


cccc"c

 c
c c
c

c c

You might also like